Random Process Characteristic Equation NOTES
Random Process Characteristic Equation NOTES
Random Process Characteristic Equation NOTES
PART - A
Problem 4. Define nth order stationary process, when will it become a SSS process?
Solution:
A random process X (t) is said to be stationary to order n or nth order stationary
if its nth order density function is invariant to a shift of time origin.
i.e., f X ( x1, x2 ,..., xn , t1, t2 ,..., tn ) = f X ( x1, x2 ,..., xn , t1 + δ , t2 + δ ,..., tn + δ ) for all
t1, t2 ,..., tn & h .
1
Unit.3. Classification of Random Processes
2
Unit.3. Classification of Random Processes
Problem 6. When are the process {X (t )} & {Y (t )} said to be jointly stationary in the
wide sense?
Solution;
Two random process {X (t )} & {Y (t )} are said to be jointly stationary in the
wide sense, if each process is individually a WSS process and RXY ( t1 , t2 ) is a function
of (t2 − t1 ) only.
∴ P { X (t ) = n} = n!
, n = 0,1, 2,...
3
Unit.3. Classification of Random Processes
ij }
= C X (ti ) , X (t j ) and Λ ij = Cofactor of λij in Λ.
⎛0 1⎞
Problem 15. If the transition probability matrix of a markov chain is ⎜ 1 1⎟ find the
⎜ ⎟
⎝2 2⎠
steady-state distribution of the chain.
4
Unit.3. Classification of Random Processes
Solution:
Let π = (π 1 , π 2 ) be the limiting form of the state probability distribution on
stationary state distribution of the markov chain.
By the property of π , π P = π
⎛0 1 ⎞
i.e., ( π 1 , π 2 ) ⎜ 1 1 ⎟⎟ = ( π 1 , π 2 )
⎜
⎝2 2 ⎠
1
π 2 = π1 ------------------------------ (1)
2
1
π + π 2 = π 2 ------ (2)
1
2
Equation (1) & (2) are one and the same.
Consider (1) or (2) with π1 + π 2 = 1 , since π is a probability distribution.
π1 + π 2 = 1
1
Using (1) , π 2 + π 2 = 1
2
3π 2
=1
2
2
π=
2
3
2 1
π = 1− π = 1− =
1 2
3 3
1 2
π 2 = 1− π1 = 1− =
3 3
1 2
∴π = & π = .
1 2
3 3
PART-B
Problem 16. a). Define a random (stochastic) process. Explain the classification of
random process. Give an example to each class.
Solution:
RANDOM PROCESS
A random process is a collection (orensemble) of random variables {X (s, t )} that are
functions of a real variable, namely time t where s S (sample space) and t T
(Parameter set or index set).
CLASSIFICATION OF RANDOM PROCESS
Depending on the continuous on discrete nature of the state space S and parameter set T ,
a random process can be classified into four types:
(i). It both T & S are discrete, the random process is called a discrete random sequence.
Example: If X n represents the outcome of the nth toss of a fair dice, then { X n, n ≥ 1} is a
discrete random sequence, since T = 1, 2, 3,...} and S = 1, 2, 3, 4, 5, 6} .
5
Unit.3. Classification of Random Processes
E ⎡ ⎣ X (t )⎤ ⎦ = ∫ X (t ) f ( 0 )d 0
π
−
2
∫ cos (t + 0 ).
π2
1 d0
=
−
π π
2
π
2
= ∫ cos (t + 0 ) d 0
π
−
2
π
1
= ⎡⎣ sin ( t + 0 )⎤⎦ π 2
π −
2
2
= cost ≠ Constant.
π
Since E ⎡ ⎣ X (t is a function of t , the random process X (t )} is not a stationary
)⎤ ⎦
process.
6
Unit.3. Classification of Random Processes
Problem 17. a). Show that the process X (t )} whose probability distribution under
⎧ ( at ) n−1 ,
⎪ n−1 n = 1, 2,...
⎪ (1+ at )
certain conditions is given by P { X ( t ) = n} = ⎨ is evolutionary.
⎪ at
⎪ , n=0
⎩ 1+ at
Solution:
The probability distribution is given by
X (t) = n : 0 1 2 3 . . . .
(at )
2
at 1 at
P ( X (t) = n) : . . . .
(1+ at ) (1+ at ) (1+ at )
2 3 4
1+ at
∞
E ⎡ ⎣ X (t )⎤ ⎦ = ∑ npn
n=0
3 ( at )
2
1 2at
= 2 + 3 + 4 + ...
(1+ at ) (1+ at ) (1+ at ) 2
= ⎧
1 2 ⎪⎨1+ 2 ⎜⎛ at ⎟⎞+ 3 ⎜ ⎛ at ⎟ ⎞ ⎫⎪
1+ at 1+ at + ...⎬
(1+ at ) ⎪⎩ ⎝ ⎠ ⎝ ⎠ ⎪⎭
2
1 at
= 2 1
(1+ at) 1+ at
E ⎡ ⎣ X (t )⎦ ⎤ = 1 = Constant
∞
E ⎡ ⎣ X 2 (t )⎤ ⎦ = ∑ n2 pn
n=0
( at ) ( at )
n −1
=∑ = ∑∞ ⎡ n ( n +1) − n⎤
n−1
∞ n2
⎣ ⎦
(1+ at ) (1+ at )
n+1 n+1
n=1 n=1
Var X (t )} = 2at
∴The given process X (t )} is evolutionary
7
Unit.3. Classification of Random Processes
b). Examine whether n the Poisson process {X (t )} given by the probability law
e−λt ( λt )
P { X (t ) = n} = , n = 0,1, 2,... is evolutionary.
n!
Solution:
∞
E ⎡ ⎣ X (t )⎤ ⎦ = ∑ npn
e− λ t ( λ t )
n=0 n
∞
= ∑n
n=0 n!
e−λt ( λ t )
∞ n
=∑
n=1 ( n −1)!
( λt )
∞ n−1
= ( λt ) e −λt ∑
n=1 (n −1)! 2
⎡ λt ( λt ) ⎤
= ( λ t ) e−λt ⎢1+ + + ...⎥
⎢⎣ 1! 2! ⎥⎦
= ( λt ) e e
−λ t λt
E ⎡ ⎣ X (t )⎤ ⎦ = λt
E ⎡ ⎣ X (t )⎦ ⎤ ≠
Constant.
Hence the Poisson process { X (t )} is evolutionary.
Problem 18. a). Show that the random process X (t ) = A cos(ω t+ θ ) is WSS if A & ω
are constants and θ is uniformly distributed random variable in (0, 2π ) .
Solution:
Since is uniformly distributed random variable in (0, 2π )
⎧1
⎪ , 0 < 0 < 2π
f (θ ) = ⎨2π
⎪⎩0 , elsewhere
2π
E ⎡⎣ X ( t ) ⎤⎦ = ∫ X ( t ) f (θ ) dθ
0
2π
1
= ∫ 2π Acos (ωt + θ ) dθ
0
2π
= A cos ( ω t + θ ) dθ
2π 0∫
A 2π
= ⎡⎣ sin ( ωt + θ ) ⎤⎦
2π 0
8
Unit.3. Classification of Random Processes
A
= ⎡ sin ( ωt + 2π ) − sin ( ωt ) ⎤
⎣ ⎦
2π
A
= ⎡⎣ sin ( ωt ) − sin ( ωt ) ⎤⎦ ⎡⎣ sin ( 2π + θ ) = sinθ ⎤⎦
2π
E ⎡ ⎣ X (t )⎤ ⎦ = 0
RXX (t1, t2 ) = E X (t1 ) X (t2 )⎤ ⎦
= E ⎡ ⎣ A2cos (ωt + θ ) cos (ωt + θ )⎤ ⎦
1 2
⎡ cos ( ω ( t1 + t2 ) + 2θ ) + cos ( ω ( t1 − t2 ) ) ⎤
= A2 E
⎢
⎣⎢ 1 2 ⎦⎥
A2 2π ⎡ ⎤
cos ( ω ( t1 + t2 ) + 2θ ) + cos ( ω ( t1 − t2 ) ) ⎦ dθ
2 ∫0 2π ⎣
=
2π
A2 ⎡ sin ⎡⎣ ω ( t1 + t 2 ) + 2θ ⎤⎦ cos ω t ⎥
⎤
= ⎢ +θ
4π ⎢ ⎣ 2 ⎥⎦ 0
A2
=
4π
[2πcosωt]
A2
= cosωt = a function of time difference
2
Since E ⎡ ⎣ X (t )⎦ ⎤ = constant
RXX (t1, t2 ) =a function of time difference
∴{X (t )} is a WSS.
b). Given a random variable y with characteristic function φ (ω ) = E (eiωy ) and a random
process define by X (t ) = cos (λt + y ) , show that { X (t )} is stationary in the wide sense
if φ (1) = φ ( 2 ) = 0 .
Solution:
Given 0 (1) = 0
⇒ E [cosy + isiny] = 0
∴ E [cosy] = E [sin y] = 0
Also 0 ( 2 ) = 0
⇒ E [cos2 y + isin2 y] = 0
∴ E [cos2 y] = E [sin 2 y] = 0
E { X ( t )} = E ⎡⎣ cos ( λt + y ) ⎤⎦
= E [cosλtcosy − sinλtsiny]
9
Unit.3. Classification of Random Processes
E ⎡ ⎣ X (t )⎤ ⎦ = ∫ X (t ) f ( y ) dy
0
2π
1
=
2π ∫ sin (ωt + y ) dy
0
1 2π
= ⎡⎣ −cos ( ωt + y ) ⎤⎦
2π 0
1
=− ⎡ cos ( ωt + 2π ) − cosωt ⎤ = 0
⎣ ⎦
2π
RXX ( t1 , t2 ) = E ⎡⎣ sin ( ωt1 + y ) sin (ωt2 + y )⎤⎦
⎡ cos ( ω ( t1 − t2 ) ) − cos ( ω ( t1 + t2 ) + 2 y ) ⎤
=E⎢ ⎥
⎢ 2
⎦⎥
) )⎤⎦ − E ⎡⎣ cos (ω ( t + t ) + 2 y )⎤ ⎦
1
⎣
= E ⎣ cos ( ω ( t − t
1
⎡
10
Unit.3. Classification of Random Processes
1 2 1 2
2 2
11
Unit.3. Classification of Random Processes
1 1 2π 1
= cos ( ω ( t1 − t2 ) ) − ∫ cos ( ω ( t1 + t2 ) + 2 y ) dy
2 2 0 2π2π
1 1 ⎡ sin ( ω ( t + t ) + 2 y ) ⎤
= cos (ω (t − t )) −
1 2
⎢ ⎥
4π ⎢ ⎣
1 2
2 2 ⎦⎥ 0
= cos ( ω ( t − t ) ) − ⎡⎣ sin ( ω ( t + t ) + 2π ) − sinω (t + t
1 1
)⎤ ⎦
8π
1 2 1 2 1 2
2
1
= cos (ω (t − t ) ) is a function of time difference.
1 2
2
∴{X (t )} is WSS.
b). Verify whether the sine wave random process X (t ) = Y sin ωt, Y is uniformly
distributed in the interval ( 1,1) is WSS or not
Solution:
Since y is uniformly distributed in( 1,1),
1
f ( y ) = , −1 < y < 1
2
1
=
2 ∫
−1
ydy
Sinωt
= (0) = 0
2
RXX (t1, t2 ) = E X (t1 ) X (t2 )⎤ ⎦
= E ⎡ ⎣ y2sinωt 1sinωt ⎤ 2⎦
⎡ 2 cosω (t1 − t2 ) − cosw (t1 + t2 ) ⎤
=
E ⎢⎣ y ⎥⎦
2
cosω (t1 − t2 ) − cosw (t1 + t2 )
= E ( y2 )
cosω (t − t ) −2cosw(t + t ) 1
=
∫−1 y2 f ( y ) dy
1 2 1 2
2
cosω (t − t ) − cosw(t + t ) 1 1
= 1 2 1 2
∫ y2dy
2 2 −1
12
Unit.3. Classification of Random Processes
cosω ( t − t ) − cosw ( t + t ) ⎛ y 3 ⎞
1
= 1 2 1 2
⎜ ⎟
4 ⎝ 3 ⎠−1
cosω (t1 − t2 ) − cosw (t1 + t2 ) ⎛ 2 ⎞
= ⎜ ⎟
4 ⎝3⎠
cosω (t1 − t2 ) − cosw(t1 + t2 )
=
6
RXX (t1, t2 ) a function of time difference alone.
Hence it is not a WSS Process.
Problem 20. a). Show that the process X (t ) = A cos λt + B sin λt (where A & B are
random variables) is WSS, if (i) E ( A) = E ( B) = 0 (ii) E ( A2 ) = E ( B 2 ) and (iii)
E ( AB ) = 0 .
Solution:
Given X (t ) = Acosλt + Bsinλt , E ( A) = E ( B) = 0 , E ( AB ) = 0 ,
E ( A2 ) = E ( B 2 ) = k (say)
E ⎡ ⎣ X (t )⎦ ⎤ = cosλt E ( A) + sinλt E ( B )
E ⎡ ⎣ X (t )⎦ ⎤ = 0 = is a constant. E ( A) = E ( B) = 0
R ( t1 , t2 ) = E { X ( t1 ) X ( t2 )}
= E {( Acosλt1 + Bsinλt1 )( Acosλt2 + Bsinλt2 )}
= E ( A2 )cosλt cos
1 λt +2 E ( B )sinλt sin1λt + E
2
2 ( AB )[sin λt cosλ1t 2 + cosλt sin
1 λt ]2
= E ( A )cosλt cosλt + E ( B ) sinλt sinλt + E ( AB ) sinλ (t + t )
2 2
1 2 1 2 1 2
= k (cosλt1cosλt2 + sinλt1sinλt2 )
= kcosλ (t1 − t2 ) = is a function of time difference.
∴{X (t )} is WSS.
b). If X (t ) = Ycost + Zsint for all t & where Y & Z are independent binary random
2 1
variables. Each of which assumes the values – 1 & 2 with probabilities &
3 3
respectively, prove that {X (t )} is WSS.
Solution:
Given
Y=y : 1 2
2 1
P( Y = y ) :
3 3
13
Unit.3. Classification of Random Processes
2 1
E ( Y ) = E ( Z ) = −1× + 2 × = 0
3 3
E (Y 2 ) = E ( Z 2 ) = ( −1) × + ( 2 ) ×
2 2 2 1
3 3
E (Y 2 ) = E ( Z 2 ) = + = = 2
2 4 6
3 3 3
Since Y & Z are independent
E (YZ ) = E (Y ) E (Z ) = 0 ---- (1)
Hence E ⎡ ⎣ X (t )⎦ ⎤ = E [ ycost + zsint ]
= E [ y]cost + E [z]sint
E ⎡ ⎣ X (t )⎦ ⎤ = 0 = is a ⎡⎣ E ( y ) = E ( z ) = 0⎤⎦
constant.
RXX (t1, t2 ) = E ⎡ ⎣ X (t1 ) X (t2
)⎤ ⎦
= E ⎣ ⎡ ( ycost1 + zsint1 )( ycost2 + zsint2 )⎤ ⎦
= E ⎡ ⎣ y2cost1 2
+ yzcost1 sint 2 + zysint1cost 2 + z2sint 1sint 2⎤ ⎦
cost
= E ( y2 )cost1 cost 2 + E [ yz]cost1 sint 2 + E[zy]sint1co2st 2 + E ⎡⎣ z ⎤ ⎦ sint1 sint2
= E ( y2 )cost1cost 2+ E ( z 2 ) sint 1sint 2
= 2 [ cost cost + sint sint ] ⎡⎣ E ( y 2 ) = E ( z 2 ) = 2⎤⎦
1 2 1 2
Problem 21. a). Check whether the two random process given by
X (t ) = A cosωt + B sin ωt & Y (t ) = B cosωt − Asin ωt . Show that X (t) & Y (t) are
jointly WSS if A & B are uncorrelated random variables with zero mean and equal
variance random variables are jointly WSS.
Solution:
Given E ( A) = E ( B) = 0
Var ( A) = Var ( B ) = σ 2
∴ E ( A2 ) = E ( B 2 ) = σ 2
As A & B uncorrelated are E (AB) = E ( A)E (B) = 0 .
E ⎡ ⎣ X (t )⎦ ⎤ = E [ Acosωt + Bsinωt ]
= E ( A) cosωt + E ( B ) sinωt = 0
E ⎡ ⎣ X (t )⎦ ⎤ = 0 = is a constant.
RXX (t1, t2 ) = E ⎣ ⎡ X (t1 ) X (t2 )⎤ ⎦
14
Unit.3. Classification of Random Processes
15
Unit.3. Classification of Random Processes
= 2
cosw (t 1 t )2 [ E ( A2 ) = E ( B 2 ) = σ 2 & E ( AB ) = E (BA) = 0]
RXX (t1, t2 ) =is a function of time difference.
E ⎡⎣ Y ( t ) ⎤⎦ = E [ Bcosωt − Asinωt ]
= E ( B) cosωt − E ( A) sinωt = 0
RYY (t1, t2 ) = E ⎡ ⎣ ( Bcosωt1 − Asinω t1 )( Bcosωt2 − Asinω t2 )⎤ ⎦
= E ⎡⎣ B 2 cosω t cos
1 ωt 2
– BAcosωt sin
1
ωt −2 AB sinωt c2os
1
ωt
+ A sinωt sin
1 ωt ⎤ 2⎦ 2
1 2
16
Unit.3. Classification of Random Processes
geometric with parameter p starts from 0. The random variables mi , i =1, 2,... are
mutually independent.
The geometric distribution is given by p (1 p ) , i = 1, 2,...
i 1
(iv) The binomial distribution of the process approaches poisson when n is large and p
is small.
Problem 22. a). Describe Poisson process & show that the Poisson process is Markovian.
Solution:
If { X (t )} represents the number of occurrences of a certain event in (0, t) then
the discrete random process {X (t )} is called the Poisson process, provided the
following postulates are satisfied
(i) P ⎡⎣ 1 occumence in ( t, t + Δt ) ⎤⎦ = λΔt + o ( Δt )
(ii) P ⎡⎣ no occurrence in ( t, t + Δt ) ⎤⎦ = 1− λΔt + o ( Δt )
(iii) P ⎡⎣ 2 or more occurrences in ( t, t + Δt ) ⎤⎦ = o ( Δt )
(iv) X (t) is independent of the number of occurrences of the event in any interval prior
and after the interval(0, t) .
(v) The probability that the event occurs in a specified number of times (t0 , t0 + t )
depends only on t , but not on t0 .
Consider
P⎡ ⎣ X = n / X (t P ⎣ ⎡ X (t1 ) = n1, X (t2 ) = n2 , X (t3 ) = n3 ⎤ ⎦
(t ) = n , X (t ) = n )⎤ ⎦ =
P ⎡ X (t ) = n , X (t ) = n ⎤
3 3 2 2 1 1
⎣ 1 1 2 2⎦
e− λt λ n t n1 ( t 2 1 ) (t – t )n −n
n2 −n1
3 3 1 –t 3 2
= 3 2
n1 !(n2 − n1 )!(n3 − n2 )!
e− λt λ n t n1 ( t 2 – t1 ) n −n
2 2 1 2 1
=
n1 !(n2 − n1 )!
− λ ( t 3 −t 2 ) n3 −n2
=e λ t( 3 − t2 n3 −n 2
)
(n 3 − n 2 )!
P ⎡⎣ X ( t3 = n3 / X ( t2 ) = n2 , X ( t1 ) = n1 ) ⎤⎦ = P ⎡⎣ X ( t3 ) = n3 / X ( t2 ) = n2 ⎤⎦
This means that the conditional probability distribution of X (t3 ) given all the past
values X (t1) = n1 , X (t2 ) = n2 depends only on the most recent values X (t2 ) = n2 .
17
Unit.3. Classification of Random Processes
= eux P X (t ) = x
x=0
( t)
t x
e
= eux
x=0 x! 1
eu ( t ) e 2u ( t )
2
= e 1+ t
+ +...
1! 2!
=e e t teu
t (e 1) u
M X (t) (u ) = e
Let X1 (t) and X 2 (t) be two independent Poisson processes
Their moment generating functions are,
t (e 1) u
t (e 1) u
1 2
=e
1 2
= (λ t + λ t )1 + (λ t + λ t ) −2 2 (λ t )( λ1 t )
2 2
1
2 2
2 2
= (λ + λ ) t + (λ − λ ) t2
2
1 2 1 2
≠ ( λ – λ ) t + (λ − λ )
2
t2
1 2 1 2
18
Unit.3. Classification of Random Processes
E {X 2 (t )} = λt + λ 2t2
∴ X1 (t ) − X 2 (t ) is not a poisson process.
(iii). The inter arrival time of a poisson process i.e., with the interval between two
successive occurrences of a poisson process with parameter λ has an exponential
1
distribution with mean .
λ
Proof:
Let two consecutive occurrences of the event be Ei & Ei+1 .
Let Ei take place at time instant ti and T be the interval between the occurrences of Ei
Ei+1 .
Thus T is a continuous random variable.
P (T > t ) = P{Interval between occurrence of Ei and Ei+1 exceedst}
= P { Ei +1 does not occur upto the instant ( ti + t )}
= P No event occurs in the interval (ti ,ti + t )}
= P { X (t ) = 0} = P0 (t )
= e− λ t
∴The cumulative distribution function of T is given by
F ( t ) = P {T ≤ t} = 1− e−λt
∴The probability density function is given by
f ( t ) = λe −λt , ( t ≥ 0 )
1
Which is an exponential distribution with mean .
λ
19
Unit.3. Classification of Random Processes
Pn (t + Δt ) − Pn (t )
= λ {P n−1 ( ) n ( )}
t −P t
∴
Δt
Taking the limits as t 0
P (t ) = λ {P (t ) − P (t )} ------- (1)
d
dt n n−1 n
P n( t ) eλt = ∫ λ P ( t )eλt
n−1
-------------------------
(2)
0
Now, we have,
P0 (t + t ) = P 0 occurences in ( 0, t + t )
= P 0 occurences in (0, t ) and 0 occurences in (t, t + t )
= P0 (t)[1 t]
P0 (t + t) P0 (t) = P0 (t)( t)
P0 (t + t ) P0 (t)
= P (t)0
t
Taking limit t 0
Lt P0 (t + t) P0 (t)
= P (t)0
t 0 t
dP0 (t)
= P ( t)
0
dt
dP0 (t)
= dt
P 0 ( t)
log P0 (t) = t +c
P0 (t ) = e t+c
P0 (t) = e tec
P0 (t) = e t A ----------------------(4)
Putting t = 0 we get
P0 (0) = e0 A = A
i.e., A =1
(4) we have
P0 (t) = e t
substituting in (3) we get
t
e t P 1(t) = e te t dt
0
20
Unit.3. Classification of Random Processes
= dt = t
0
P1 (t) = e t t
Similarly n = 2 in (2) we have,
t
= e t
te tdt
0
t2
= 2
2
P (t)e t = e ( t )
t 2
2
2!
Proceeding similarly we have inn general
e −λt ( λt )
Pn (t ) = P{ X (t ) = n} = , n = 0,1,...
n!
Thus the probability distribution of X (t ) is the Poisson distribution with parameter λt .
E ⎡ ⎣ X (t )⎤ ⎦ = λt .
.b). Find the mean and autocorrelation and auto covariance of the Poisson process.
Solution:
The probability law of the poisson process { X (t )} is the same as that of a poisson
distribution with parameter λt
∞
E ⎡ ⎣ X (t )⎦ ⎤ = ∑ nP ( X (t ) = n)
e− λ t ( λ t )
n=0 n
∞
= ∑n
n=0 n!
λt ( λt )
∞ n−1
E X{ (t )} = ∑ n2 P ( X (t ) = n)
2
n=0
e−λt ( λt )
n
∞
n=0
(
= ∑ n −n+n 2
) n!
21
Unit.3. Classification of Random Processes
e−λt ( λ t ) e−λt ( λt )
∞ n ∞ n
= ∑ n ( n −1) +∑n
n=0 n! n=0 n!
e−λt ( λ t )
∞ n
= ∑ n ( n −1) + λt
n=0 n!
⎡ ( λt )2 ( )
λ t
3
( λt )
4
⎤
=e − λt
⎢ + + + ...⎥ + λt
⎣⎢ 1 1! 2! ⎥⎦
= ( λ t ) e−λt eλt + λt
2
= (λt ) + λt
2
E {X 2 (t )} = λt + λ 2t2
∴Var ⎡ ⎣ X (t )⎤ ⎦ = λt
RXX ( t1 , t2 ) = E { X ( t1 ) X ( t2 )}
{
= E X ( t1 ) ⎣⎡ X ( t2 ) − X ( t1 ) + X ( t1 )⎤⎦ }
{
= E X (t ) ⎡ X (t ) − X (t )⎤ + E ⎡ X 2 (t )⎤ }
1 ⎣ 2 1 ⎦ ⎣ 1 ⎦
= E ⎡ X (t )⎤ E ⎡ X (t ) − X (t )⎤ + E ⎡ X 2 (t )⎤
⎣ 1 ⎦ ⎣ 2 1 ⎦ ⎣ 1⎦
= λ 2t t + λt if t ≥ t
1 2 1 2 1
R ( t ,t ) = λ 2 t t + λ min ( t ,t )
XX 1 2 1 2 1 2
Auto Covariance
C XX ( t1 , t2 ) = RXX ( t1 , t2 ) − E { X ( t1 )} E { X ( t2 )}
= λ 2t , t + λt − λ 2t t
1 2 1 1 2
= λt1, if t2 ≥ t1
= λ min (t1, t2 )
Problem 24. a). Prove that the random process X (t ) = A cos(ω t+ θ ) . Where A, ω are
constants θ is uniformly distributed random variable in (0, 2π ) is ergodic.
Solution:
Since θ is uniformly distribution in (0, 2π ) .
1
f (θ ) = , 0 < θ < 2π
2π
Ensemble average same,
2π
1
E ⎡⎣ X ( t ) ⎤⎦ = ∫ cos ( ωt + θ ) dθ
2π 0
22
Unit.3. Classification of Random Processes
2π
1
=
2π ∫ ( cosωt cosθ − sinωt sinθ ) dθ
0
1
[cosωt sinθ + sinωt cosθ ]
2π
=
2π 0
= 1 [sinωt − sinωt] = 0
2π
E ⎡ ⎣ X (t )⎤ ⎦ = 0
RXX (t1, t2 ) = E X (t1 ) X (t2 )⎤ ⎦
= E ⎣⎡ cos ( ωt1 + θ ) cos ( ωt 2 + θ ) ⎤⎦
1
= E ⎡ cos ( ωt + ωt + 2θ ) + cos ( ωt − ωt ) ⎤
2 2π ⎣ 1 2 1 2⎦
1 1
= ∫ ⎡ cos ( ωt1 + ω t2 + 2θ ) + cos ( ωt1 − ωt 2 ) ⎤⎦ dθ
2 0 2π ⎣
1 ⎡ sin (ωt + ωt + 2θ ) + θ cos ( ω ( t − t ) ) ⎤ 2π
= 1 2 1 2
⎥
2 ⎥⎦ 0
⎢
4π ⎢ ⎣
2π
= cos (ω (t − t ))
4π 1 2
R (t , t ) = cos (ω (t − t ))
1
XX 1 2 1 2
2
The time average can be determined by
lim 1 T
X (t ) =
T → ∞ 2T −T ∫ cos (ωt + θ ) dt
= lim 1 ⎢ ⎡ sin ( ωt + θ ) ⎤ ⎥
T
T → ∞ 2T ω
⎣ ⎦ −T
lim 1
= ⎡ sin ( ωT + θ ) − sin ( −ωT + θ ) ⎤
T → ∞ 2Tω ⎣ ⎦
X (t ) = 0 [ As T − > ∞]
The time auto correlation function of the process,
Lt 1 T
T → ∞ 2T ∫ X ( t ) X ( t + τ ) dt
−T
T
lim 1
=
T → ∞ 2T ∫ cos (ωt + θ )cos (ωt + ωτ + θ ) dt
−T
lim 1 T ⎡ cos ( ωt + ωτ + 2θ ) + cosωτ ⎤
= T → ∞ 2T
∫ ⎢⎣
−T
2
⎥ dt
⎦
23
Unit.3. Classification of Random Processes
T → ∞ 2T ∫ X ( t )X ( t + τ ) dt = RXX (τ )
−T
RXX (τ ) = E ⎣⎡ X ( t ) X ( t + τ ) ⎤⎦
= E ⎡⎣ 10 cos (100t + θ ) 10 cos (100t +100τ + θ ) ⎤⎦
⎪⎧ ⎡ cost ( 200t +100τ + 2θ ) + cos (100τ ) ⎤ ⎫⎪
= E ⎨100 ⎢ ⎥⎬
2
⎪⎩ ⎣ ⎦ ⎪⎭
π
1
= 50 ∫ ⎡ cos ( 200t +100τ + 2θ ) + cos (100τ ) ⎤⎦ dθ
−π
2π ⎣ π
= 50 ⎢⎡ sin ( 200t +100τ + 2θ ) ⎤
2π 2 + θ cos (100τ ) ⎥
⎣ ⎦ −π
RXX (τ ) = 50 cos (100τ )
Lt 1 T
T → ∞ 2T −T∫ X ( tT) X ( t + τ ) dt
Lt 1
= (100t + θ )10cos (100t +100τ + θ )⎤⎦ dt
T → ∞ 2T T−T ∫ ⎡10cos
⎣
Lt 25
=
T → ∞ T −T ∫ ⎡⎣ cos ( 200t +100τ + 2θ ) + cos (100τ ) ⎤⎦ dtT
Lt 25 ⎡ sin ( 200t +100τ + 2θ ) ⎤
=T →∞ T ⎢ 200 + t cos (100τ ) ⎥
⎣ ⎦ −T
lim 50
lim
XT = Tcos (100τ )
T →∞ T →∞ T
= 50 cos (100τ ) = is a function of time difference
24
Unit.3. Classification of Random Processes
Problem 25. a). If the WSS process { X (t )} is given by X (t ) = cos (ωt + φ ) where φ is
uniformly distributed over ( −π , π ) prove that { X ( t )} is correlation ergodic.
Solution:
To Prove { X (t )} is correlation ergodic it is enough to show that when
Lt 1 T
∫ X ( t )X ( t + τ ) dt = RXX (τ )
T → ∞ 2T −T
RXX (τ ) = E ⎡⎣ X ( t ) X ( t + τ ) ⎤⎦
2 ∫−π 2π ⎣
t+ − 20 + cos ⎦ d 0
1 ⎡ sin (2ωt + ωτ + 20 )
π
⎤
= + 0 cosωτ ⎥
4π ⎢⎣ 2 ⎥⎦ −π
1
RXX (τ ) = cos (ωτ )
2
Consider, T
Lt 1
T → ∞ 2T ∫ X ( t ) X ( t + τ ) dt
−T T
Lt 1
=
T→
Lt∞ 2T1 ∫ cos (ωt + 0 )cos (ωt + ωτ + 0 ) dt
∫ ⎣⎡ cos ( 2ωt + ωτ + 20 ) + cos (ωτ ) ⎤ dt⎦
−T
=
T → ∞ 4T
T
−T
T → ∞ 2π ∫ X ( t ) X ( t + τ ) dt =
−T 2
cos ( ωτ ) = R (τ )
25
Unit.3. Classification of Random Processes
b). A random process{X (t )} defined as{X (t )} = A cosωt + B sin ωt , where A & B are
the random variables with E ( A) = E ( B) = 0 and E ( A2 ) = E ( B 2 ) & E ( AB ) = 0 . Prove
that the process is mean ergodic.
Solution:
To prove that the process is mean ergodic we have to shoe that the ensemble mean is
same as the mean in the time sense.
Given E ( A) = E (B) = 0 ------------------ 1
E (A2 ) = E (B2 ) = k (say) & E (AB) = 0 ----- 2.
Ensemble mean is
E ⎡ ⎣ X (t )⎦ ⎤ = E [ Acosωt + Bsinωt ]
= E ( A) cosωt + E ( B ) sinωt = 0 Using (1)
Time Average
lim 1 T
X (t ) =
T → ∞ 2T −T ∫ ( Acosωt + B sinωt ) dt
lim 1 ⎡ Asinω t Bcosω t ⎤
T
= –
T → ∞ 2T ⎢ ⎣ ω ω ⎥ −T
lim 1 ⎡ ⎛ Asinωt Bcosωt ⎞ ⎛⎟ Asin
= T → ∞ 2T ⎢ ⎜ ⎜ ω (−T ) − Bcosω (−T ) ⎞⎤ ⎟⎥
ω − ω − ω ω
⎝ ⎦ ⎠
⎢⎣ ⎝ ⎠⎥ ⎦
lim 1 ⎡ Asinωt Bcos ωt Asin ωT BcosωT ⎤
= − + +
⎢
T → ∞ 2T ⎣ ω ω ω ω ⎥⎦
lim 1 ⎡ 2 AsinωT ⎤
=
T → ∞ 2T ⎢ ⎣ ω ⎥⎦
A lim ⎡ sinωT ⎤ = 0 .
=
ω T → ∞⎢ ⎣ T ⎥ ⎦
The ensemble mean =Time Average
Hence the process is mean ergodic.
26.a). Prove that in a Gaussian process if the variables are uncorrelated, then they are
independent
Solution:
Let us consider the case of two variables X t & X t
1 2
26
Unit.3. Classification of Random Processes
⎛σ 2 0⎞
Matrix of co-factors = ∑ = ⎜
2
⎟
ij
⎝0 σ 21 ⎠
∴ ∑ = σ 2σ 12 − 02 = σ 2σ 2 1 2
Eij
Now ( X − μ ) ∑ −1 ( X − μ ) = ( X − μ ) ( X − μ)
1 1
∑
Let us consider
( X − μ ) Eij ( X − μ ) 2
1
⎡σ 0 ⎤ ⎡ x1 − μ ⎤1 ⎡ x1 − μ1 ⎤
[X − μ , X – μ ]⎢ 2 ⎥ = ⎡ ⎣ ( – μ )σ + 0 + 0 + ( x – μ )σ 2 ⎤ ⎦ ⎢
2
⎥⎢ ⎥
x
1 1 2 2 0 σ2 ⎣ x −μ 1 1 2 2 2 1 x −μ
⎣ 1 ⎦ 2 2⎦ 2 2 ⎦
⎣
= (x − μ ) σ 2 +(x − μ ) σ 2
2 2
1 1 2 2 2 1
( 2π )
1 2 2/2
σ σ12 2
2
⎡
−1 ⎛ x − μ ⎞
2
⎛ x −μ ⎞ ⎤
2
⎢ ⎜ 1 1 ⎟ +⎜ 2 2 ⎟ ⎥
1 2 ⎢ ⎝ σ1 ⎠ ⎝ σ2 ⎠ ⎥⎦
= e ⎣
( 2π ) σ 1σ 2 2 2
1 ⎛ x −μ ⎞ 1 ⎛ x −μ ⎞
− ⎜ 1 1⎟ – ⎜ 2 2⎟
= 1 2 ⎝ σ1 ⎠ 2⎝ σ2 ⎠
e e
2π σ1σ 2
2 2
1 ⎛ x −μ ⎞ 1⎛ x −μ ⎞
− 2⎜ 1 1 ⎟ – ⎜ 2 2⎟
= 1 e ⎝ σ1 ⎠ 1 2⎝ σ2 ⎠
e
σ1 2π σ2 2π
f ( x1, x2 ) = f ( x1 ) f ( x2 )
∴ X t& X t are independent.
1 2
27
Unit.3. Classification of Random Processes
28
Unit.3. Classification of Random Processes
= P{ Z ≤ 0.7137}
= 2× 0.2611 = 0.5222 .
Problem 27. a). Define a Markov chain. Explain how you would clarify the states and
identify different classes of a Markov chain. Give example to each class.
Solution:
Markov Chain: If for all n,
P { X n = an / Xn−1 = an−1, X n−2 = an−2 ,..., X 0 = a0} = P { X n = an / Xn−1 = an−1} then the
process X n } , n = 0,1, 2,... is called a Markov Chain.
Classification of states of a Markov chain
Irreducible: A Markov chain is said to be irreducible if every state can be reacted from
every other state, where Pij(n) > 0 for some n and for all i & j.
⎡ 0.3 0.7 0 ⎤
Example: ⎢ 0.1 0.4 0.5⎥
⎢ ⎥
⎢⎣ 0 0.2 0.8⎥⎦
Period: The Period di of a return state i is defined as the greatest common division of all
m such that Pij(m) > 0
i.e., d i = GCD m : p(ijm) > 0 }
State i is said to be periodic with period di if di >1 and a periodic if di =1.
Example:
⎡ 0 1⎤
So states are with period 2.
⎢ 1 0⎥
⎣ ⎦
⎡ 1 3⎤
⎢ 4 4⎥
⎢ ⎥ The states are aperiodic as period of each state is 1.
⎢ 1 1⎥
⎢⎣ 2 2
⎥⎦
Ergodic: A non null persistent and aperiodic state is called ergodic.
Example:
29
Unit.3. Classification of Random Processes
⎡0 0 1 0⎤
⎢0 0 0 1⎥
⎢ ⎥
⎢0 1 0 0 ⎥ Here all the states are ergodic.
⎢ ⎥
⎢ 1 1 1 1⎥
⎢⎣ 2 8 8 4⎥ ⎦
b). The one-step T.P.M of a Markov chain { X n ; n = 0,1, 2,...} having state space
⎡ 0.1 0.5 0.4 ⎤
⎢ 0.2 0.2 ⎥ and the initial distribution is π 0 = (0.7, 0.2, 0.1) .
S = {1, 2, 3} is P = ⎢ 0.6 ⎥
⎢⎣ 0.3 0.4 0.3⎥⎦
Find (i) P ( X 2 = 3 X 0 = 1) (ii) P ( X 2 = 3) (iii) P ( X 3 = 2, X 2 = 3, X1 = 3, X 0 = 1) .
Solution:
(i) P ( X 2 = 3 / X 0 = 1) = P ( X 2 = 3 / X1 = 3) P ( X1 = 3 / X 0 = 1) + P ( X 3 = 3 / X1 = 2) P ( X1 = 2 / X 0 = 1)
+P ( X 2 = 3 / X1 = 1) P ( X1 = 1/ X 0 = 1)
= (0.3)(0.4) + (0.2)(0.5) + (0.4)(0.1) = 0.26
⎛ 0.43 0.31 0.26 ⎞
⎜ ⎟
P = P.P = 0.24 0.42 0.34
2
⎜ ⎟
⎜ 0.36 0.35 0.29 ⎟
⎝ ⎠
3
(ii). P ( X 2 = 3) = ∑ P ( X 2 = 3 / X 0 = i ) P ( X 0 = i )
i=1
= P ( X 2 = 3 / X 0 = 1)P ( X 0 = 1) + P ( X 2 = 3 / X 0 = 2 ) P ( X 0 = 2)
+ P ( X 2 = 3 / X 0 = 3)P ( X 0 = 3)
= P 2P ( X = 1) + P2 P ( X = 2) + P2 P ( X = 3)
13 0 23 0 33 0
30
Unit.3. Classification of Random Processes
Problem 28. a). Let { X n ; n = 1, 2, 3,.....} be a Markov chain with state space S = {0,1, 2}
⎡⎢ 0 1 0 ⎤⎥
1 1 1
and 1 – step Transition probability matrix P = ⎢ (i) Is the chain ergodic?
⎥
⎢ 4 2 4⎥
Explain (ii) Find the invariant probabilities. ⎢ 0 1 0⎥
Solution: ⎣ ⎦
⎡ 1 1 1⎤
⎢ ⎥
⎢⎡ 01 11 10 ⎥⎤ ⎢⎡ 01 11 01 ⎤⎥ ⎢ 14 23 14 ⎥
P2 = P.P = ⎢ ⎥⎢ ⎥ =⎢ ⎥
⎢⎢ 4 2 4 ⎥⎥ ⎢⎢ 4 2 4 ⎥⎥ ⎢⎢ 8 4 8 ⎥⎥
0 1 0 0 1 0 1 1 1
⎣ ⎦⎣ ⎦ ⎢ ⎥
⎢ ⎣ 4 2 4⎥ ⎦
⎛1 1 1 ⎞ ⎛1 3 1⎞
⎜ 4 2 4 ⎟⎛⎜ 0 1 0 ⎟⎞ ⎜ 8 4 8 ⎟
1 3 1 1 1 1 ⎜ 3 5 3⎟
P =P P=
3 2 ⎜ ⎟ ⎜ ⎟ = ⎜ ⎟
⎜ 8 4 8 ⎟⎜ 4 2 4 ⎟ ⎜ 16 8 16 ⎟
⎜ ⎟ ⎜ ⎟
1 1 1 ⎜⎝ 0 1 0 ⎟⎠ ⎜ 18 34 18 ⎟
⎜4 2 4 ⎟
⎝ ⎠ ⎝ ⎠
P11(3) > 0, P13(2) > 0, P21(2) > 0, P22(2) > 0, P33(2) > 0 and all other Pij(1) > 0
Therefore the chain is irreducible as the states are periodic with period 1
i.e., aperiodic since the chain is finite and irreducible, all are non null persistent
The states are ergodic.
⎡⎢ 0 1 0 ⎤⎥
1 1 1
[π 0 π 1 π 2 ] ⎢ 4 2 4 ⎥ = [π 0 π1 π2 ]
⎢ ⎥
⎢ 0 1 0⎥
⎣ ⎦
π1
= π – − − − − − − − − − − −(1)
0
4
π
π + 1 + π 2 = π – − − − − − − − − − − −(2)
0 1
2
π1
= π – − − − − − − − − − − −(3)
2
4
π 0 + π1 + π 2 = 1− − − − − − − − − − − −(4)
π π
From (2) π + π = π − 1 = 1
0 2 1
2 2
∴π 0 + π1 + π 2 = 1
π1
+π =1
1
2
31
Unit.3. Classification of Random Processes
3π1
=1
2
2
π=
1
3
π1
From (3) =π
2
4
1
π=
2
6
2 1
Using (4) π + + = 1
0
3 6
4 +1
π + =1
0
6
5 1
π + = 1⇒ π =
0 0
6 6
1 2 1
∴π 0 + , π 1 = & π 2 = .
6 3 6
⎛0 1 0⎞
⎜1 1 ⎟⎟ and
b). Find the nature of the states of the Markov chain with the TPM P = ⎜ 0
⎜2 2⎟
⎜1 0 0 ⎟⎠
⎝
the state space(1, 2, 3).
Solution:
⎛1 1⎞
0
⎜ 2 ⎟⎟
⎜2
P = ⎜0 1 0 ⎟
2
⎜1 ⎟
⎜ 0 1⎟
⎝2 2⎠
⎛ 0 1 0⎞
⎜1 ⎟
P3 = P2.P = ⎜ 0 1 ⎟= P
⎜2 2⎟
⎜0 1 0 ⎟
⎝ ⎠
⎛1 1⎞
0
⎜2 2 ⎟⎟
⎜
P = P .P = ⎜ 0 1 0 ⎟ = P2
4 2 2
⎜1 ⎟
⎜ 0 1 ⎟
⎝2 2⎠
∴P = P & P
2n 2 2 n+1
=P
Also P002 > 0, P011 > 0, P022 > 0
P101 > 0, P112 > 0, P121 > 0
32
Unit.3. Classification of Random Processes
Problem 29. a). Three boys A, B and C are throwing a ball to each other. A always
throws to B and B always throws to C, but C is as likely to throw the ball to B as to A.
Find the TPM and classify the states.
Solution:
A B C
⎛ ⎞
A⎜ 0 1 0⎟
P= ⎜ ⎟
B⎜0 0 1⎟
C⎜1 1 ⎟
⎜ 0⎟
⎝2 2 ⎠
⎛ ⎞
⎜0 0 1⎟
⎜ ⎟
P = P× P = 1
2 ⎜ 1 0 ⎟
⎜2 2 ⎟
⎜ 1 1 ⎟
⎜0 ⎟
⎝ 2 2⎠
⎛1
⎜ 1 ⎞
2 2 0 ⎟
⎜ ⎟
1 1⎟
P3 = P2 × P = ⎜ 0 2 2⎟
⎜
⎜ ⎟
1 1 1
⎜ ⎟
⎝4 4 2⎠
For any i = 2, 3
Pii2 Pii3,... > 0
G.C.D of 2, 3, 5,... = 1
The period of 2 and 3 is 1. The state with period 1 is aperiodic all states are ergodic.
b). A man either drives a car or catches a train to go to office each day. He never goes 2
days in a row by train but he drives one day, then the next day he is just as likely to drive
again as he is to travel by train. Now suppose that one the first day of the week, the man
tossed a fair dice and drove to work iff a 6 appeared. Find the probability that he takes a
train on the third day and also the probability that on the third day and also the
probability that he drives to work in the long run.
Solution:
State Space = (train, car)
33
Unit.3. Classification of Random Processes
Problem 30 a). Three are 2 white marbles in urn A and 3 red marbles in urn B. At each
step of the process, a marble is selected from each urn and the 2 marbles selected are inter
changed. Let the state ai of the system be the number of red marbles in A after i
changes. What is the probability that there are 2 red marbles in A after 3 steps? In the
long run, what is the probability that there are 2 red marbles in urn A?
34
Unit.3. Classification of Random Processes
Solution:
State Space X n } = (0,1, 2) Since the number of ball in the urn A is always 2.
0 1 2
⎛ ⎞
⎜0 1 0 ⎟
0 ⎜ ⎟
P= 1 1 1
1 ⎜ ⎟
⎜6 2 3 ⎟
2 ⎜
2 1⎟
⎜0 ⎟
⎝ 3 3⎠
X n = 0 , A = 2W (Marbles) B = 3R (Marbles)
Xn+1 = 0 P00 = 0
Xn+1 = 1 P01 = 1
Xn+1 = 2 P02 = 0
Xn = 0 , A = 1W &1R (Marbles) B = 2R &1W (Marbles)
1
X =0 P =
n+1 10
6
1
X =1 P =
n+1 11
2
1
X =2 P =
n+1 12
3
Xn = 2 , A = 2R (Marbles) B =1R & 2W (Marbles)
Xn+1 = 0 P20 = 0
2
X =1 P =
n+1 21
3
1
X =2 P =
n+1 22
3
P = (1, 0, 0) as there is not red marble in A in the beginning.
( 0)
35
Unit.3. Classification of Random Processes
⎛ ⎞
⎜0 1 0⎟
⎜ ⎟
1 1 1
( π0 π π ) ⎜
⎜
6 2 3⎟ ( 0 1
⎟ = ππ π )1
2 2
⎜
2 1⎟
⎜0 ⎟
⎝ 3 3⎠
1
π1 = π 0
6
1
π + π + 2 π 2 = π1
0
2 1 3
1 1
π 1 + π 2 = π2
3 3
& π 0 + π1 + π 2 = 1
1 6 3
Solving π 0 = , π 1 = , π 2 =
10 10 10
P {here are 2 red marbles in A in the long run} = 0.3.
b). A raining process is considered as a two state Markov chain. If it rains the state is 0
⎛ 0.6 0.4 ⎞
and if it does not rain the state is 1. The TPM is P = ⎜ ⎟ . If the Initial distribution
⎝ 0.2 0.8 ⎠
is (0.4, 0.6) . Find it chance that it will rain on third day assuming that it is raining today.
Solution:
⎛ 0.6 0.4 ⎞⎛ 0.6 0.4 ⎞
P2 = ⎜ ⎟⎜ ⎟
⎝0.2 0.8 ⎠⎝0.2 0.8 ⎠
⎛ 0.44 0.56 ⎞
=⎜ ⎟
⎝ 0.28 0.32 ⎠
P[rains on third day / it rains today = P[ X 3 = 0 / X1 = 0]
= P002 = 0.44
36
Unit.2. Two Dimensional Random Variables
Part.A
Problem 1. Let X and Y have joint density function f ( x, y ) = 2, 0 < x < y < 1.Find the marginal
density function. Find the conditional density function Y given X = x .
Solution:
Marginal density function of X is given by
∞
f X ( x) = f ( x) = ∫ f ( x, y ) dy
−∞
1 1
= ∫ f ( x, y ) dy = ∫ 2dy = 2 ( y )
1
x
x x
fY ( y ) = f ( y ) = ∫ f ( x, y ) dx
−∞
y
( x ) = f ( x, y ) = 2 = 1 .
0
⎧0 , x < −a
⎪1 ⎛ x ⎞
Problem 2. Verify that the following is a distribution function. F ( x ) = ⎨ ⎜ +1⎟ , −a < x < a .
2 a
⎪ ⎝ ⎠
⎪⎩1 ,x > a
Solution:
F ( x) is a distribution function only if f ( x) is a density function.
d 1
f ( x ) = ⎡⎣ F ( x ) ⎤⎦ = , –a < x<a
dx 2a
∞
∫ f ( x) = 1
−∞
a 1 1 1
a
∴∫
2a
dx =
2a
[ ]−a 2a ⎡⎣ a − ( −a )⎤⎦
x =
−
a
1
Unit.2. Two Dimensional Random Variables
1
=
.2a = 1 .
2a
Therefore, it is a distribution function.
x2
Solution:
( x ) dx = ⎡ F ( x ) ⎤
x2
x2 f
∫
x1
X ⎣ X ⎦ x1
= FX ( x2 ) − FX ( x1 )
= P [ X ≤ x 2 ] − P [ X ≤ x 1]
= P[x1 ≤ X ≤ x2 ]
∫ ∫ f ( x, y ) dxdy = 1.
−∞ −∞
∞y
⇒ ∫ ∫ Ae−xe− ydxdy = 1
0 0
2
Unit.2. Two Dimensional Random Variables
∞ y
⎛ e− x ⎞
⇒ A∫ e− y ⎜ ⎟ dy = 1
0 ⎝ −1 ⎠0
∞
⇒ A∫ ⎡⎣ e− y − e −2 y ⎤⎦ dy = 1
0
−2 y ∞
⎡ e− y e ⎤
⇒ A ⎢− ⎥ =1
⎣ −1 −2 ⎦ 0
⎡ 1⎤
⇒A =1⇒ A = 2
⎢ ⎣ 2⎥ ⎦
Problem 6. Examine whether the variables X and Y are independent, whose joint density
function is f ( x, y ) = xe−x( y+1) , 0 < x, y < ∞ .
Solution:
The marginal probability function of X is
∞ ∞
f X ( x) = f ( x ) = xe− x( y+1)dy
∫ f ( x, y ) dy = ∫
−∞ 0
∞
− x( y +1)
⎡ ⎤
= x⎢ e ⎥ = − ⎡⎣ 0 − e − ⎤⎦ = e − ,
x x
⎣ −x ⎦ 0
The marginal probability function of Y is
∞ ∞
fY ( y ) = f ( y ) = xe − x( y+1)dx
∫ f ( x, y ) dx = ∫
−∞ 0
⎪⎧ e − x( y +1) ⎤ ∞ ⎡ e− x( y +1) ⎤ ⎫ ⎪∞
=x ⎡ −⎢ ⎥
2 ⎬
⎨⎢ ⎥
⎩⎪ ⎣ − ( y +1) ⎦ 0 ⎢ ( y +1) ⎥ ⎪
⎣ ⎦⎭ 0
1
=
( y +1)
2
1
Here f ( x ) . f ( y ) = e−x × ≠ f ( x, y )
(1+ y )
2
3
Unit.2. Two Dimensional Random Variables
fY ( y ) = 2 ye − y , y > 0
2
⎛ π π⎞
Problem 8. If X is uniformly distributed random variable in −⎜ , , Find the probability
2 2⎟
⎝ ⎠
density function of Y = tanX .
Solution:
Given Y = tanX ⇒ x = tan−1 y
dx 1
∴ =
dy 1+ y2
⎛ π π⎞
Since X is uniformly distribution in − ⎜ , ,
2 2⎟
⎝ ⎠
1 1
f ( x) = =
b−a π +π
X
2 2 π
1 π
f ( x) = , − < x <
X
π 2 2 ⎞
Now f ( y ) = f ( x) dx = 1 ⎛ 1 , −∞< y <∞
⎜ ⎟
dy π ⎝ 1+ y2 ⎠
Y X
1
∴ f ( y) = , −∞ < y < ∞
π (1+ y2 )
Y
E ( xy ) = ∫ ∫ xyf ( x, y ) dxdy y
0y
1 1
= 24 ∫ ∫ xy 2 (1− x ) dxdy x= y
0y
1
⎡ 1 y2 y3 ⎤ 4
= 24∫ y ⎢ − 2
+ ⎥ dy = . x
0 ⎣6 2 3⎦ 15
4
Unit.2. Two Dimensional Random Variables
= E ( XY ) − E ( X ) E (Y ) ⎡⎣ E ( X ) = X , E ( Y ) = Y ⎤⎦
Problem 11. If X and Y are independent random variables prove that cov ( x, y ) = 0
Proof:
cov ( x, y ) = E ( xy ) − E ( x) E ( y )
But if X and Y are independent then E ( xy ) = E ( x ) E ( y )
cov ( x, y ) = E ( x) E ( y ) − E ( x) E ( y )
cov ( x, y ) = 0.
Problem 15. i). Two random variables are said to be orthogonal if correlation is zero.
5
Unit.2. Two Dimensional Random Variables
Part-B
Problem 16. a). The joint probability density function of a bivariate random variable ( X ,Y ) is
⎧⎪k ( x + y ) , 0 < x < 2, 0 < y < 2
f ( x, y ) = where ' k ' is a constant.
XY ⎨
⎩⎪0 , otherwise
i. Find k .
ii. Find the marginal density function of X and Y .
iii. Are X and Y independent?
iv. Find fY
X
( y x ) and f ( x y ) .
X
Y
Solution:
(i). Given the joint probability density function of a brivate random variable ( X ,Y ) is
⎪⎧ K ( x + y ) , 0 < x < 2, 0 < y < 2
f XY (x, y ) = ⎨
⎪⎩0 , otherwise
∞ ∞ ∞∞
⇒ K ∫ ( 2 + 2 y ) dy = 1
0
2
⇒ K ⎡⎣ 2 y + y 2 ⎤⎦ 0 = 1
⇒ K [8 − 0] = 1
1
⇒K=
8
(ii). The marginal p.d.f of X is given by
∞
12
( ) ( x + y )dy
8 ∫0
f X ( x) = ∫ f x, y dy =
−∞
2
1⎡ 2
⎤
= 8 xy + y ⎥ = 1+4 x
⎢ 2
⎣ ⎦0
∴ The marginal p.d.f of X is ⎧ x +1
, 0< x<2
⎪
f X ( x) = ⎨ 4
⎪⎩0 , otherwise
The marginal p.d.f of Y is
∞
12
fY ( y ) = ∫ f ( x, y ) dx = ∫ ( x + y ) dx
−∞
80
6
Unit.2. Two Dimensional Random Variables
2
1 ⎢⎡ x2 ⎤
= 8 2 + yx ⎥
⎣ ⎦0
1 y +1
= [2 + 2 y] =
8 4
∴The marginal p.d.f
⎧ y +1
of Y is
, 0< y<2
⎪
fY ( y) = ⎨ 4
⎪⎩0 , otherwise
(iii). To check whether X and Y are independent or not.
( x +1) ( y +1)
f X ( x ) fY ( y ) = ≠ f XY ( x, y )
4 4
Hence X and Y are not independent.
f X ( x)
( x +1) 2 ( x +1)
X x 1
4
( )
⎜ ⎟
y 1⎛x+ y⎞
Y
f X x = , 0 < x < 2, 0 < y < 2
⎛ 1 2 ⎝ x +1 ⎠
0< y< ⎞
2
⎜
(v) P ⎜
⎜
2 ⎟
x = 1⎟ = ∫ fY
⎟ 0 X (
y = 1 dy
x
)
⎝ ⎠
1
1 2 1+ y 5
2 ∫0 2
= dy = .
32
Problem 17.a).⎧1 If X and Y are two random variables having joint probability density function
( 6 − x − y ) , 0 < x < 2, 2 < y < 4
⎪ Find (i) P ( X < 1∩Y < 3)
f ( x, y ) = ⎨ 8
⎪⎩0 , otherwise
(
(ii) P ( X + Y < 3) (iii) P X < 1
Y <3
. )
b). Three balls are drawn at random without replacement from a box containing 2 white, 3 red
and 4 black balls. If X denotes the number of white balls drawn and Y denotes the number of
red balls drawn find the joint probability distribution of ( X ,Y ) .
Solution:
a).
y =3 x=1
7
Unit.2. Two Dimensional Random Variables
y =3 x=1
1
=∫ ∫ 8 (6 − x − y ) dxdy
y =2 x=0
3 1
1
=
8 ∫2 ∫0
(6 − x − y ) dxdy
1
13⎡ x2 ⎤
= ∫ ⎢ 6x − – xy ⎥ dy
82⎣ 2 ⎦0 3
1 3 ⎡ 11 ⎤ 1 ⎡ 11y y 2 ⎤
= ∫ ⎢ − y ⎥ dy = ⎢ − ⎥
8 ⎣ 2 ⎦ 8 ⎣2 2 ⎦
2 2
3
P ( X < 1∩ Y < 3) =
8
1 3− x
1
(ii). P ( X + Y < 3) = ∫ ∫ (6 − x − y ) dydx
0 2
8
11⎡ 3− x
y2 ⎤
= ∫ ⎢ 6 y − xy − ⎥ dx
8 ⎣ 2 ⎦
0 2
11 ( 3 − x) − [12 − 2x − 2]⎤⎥ dx
2
⎡ ( ) ( )
= 8 ∫ ⎢ ⎢6 3 − x − x 3 − x − 2 ⎥⎦
0⎣
1 1 ⎡ (
9 + x2 − 6x ) ⎤
∫
= ⎢18 − 6x − 3x + x − 2
(
− 10 − 2x ⎥ dx )
80 2 ⎥⎦
⎢⎣ 9 x 6x 2
⎤
11⎡
= ∫ ⎢ 18 − 9x + x 2 − − + −10 + 2x ⎥ dx
80⎣ 2 2 2 ⎦
11⎡ 7 x2 ⎤
= ∫ ⎢ − 4x + ⎥ dx
8 2 2
0⎣ ⎦
1
3
1 ⎢⎡ 7x 4x 2 x ⎤ 1⎡ 7 1⎤
= 8 2 – 2 + ⎥ = 8 ⎢ ⎣ 2− 2 + 6 ⎥ ⎦
⎣ 6 ⎦0
1 ⎡ 21−12 +1 ⎤ 1 ⎛ 10 ⎞ 5
= = = .
⎢ ⎥ 8 ⎜ 6 ⎟ 24
8 ⎣ 6 ⎦ ⎝ ⎠
P ( x < 1∩ y < 3)
(
(iii). P X < 1
Y <3 = ) P ( y < 3)
2
8
Unit.2. Two Dimensional Random Variables
1⎡ x2 2
= 8 6x − 2 – yx ⎤⎥
⎢
⎣ ⎦0
1
= [12 − 2 − 2 y]
8
5− y
= , 2 < y < 4.
4
x =1 y =3
∫ ∫ (6 − x − y ) dxdy
1
(
8
P X <1
Y <3 = ) x=0 y =2
y =3
∫ fY ( y ) dy
y =2
3 3
= 3 ⎛ 5 −8y ⎞ = 1 ⎡ 8 y 2 ⎤ 3
∫⎝⎜ 4 ⎠⎟ dy ⎢ 5 y − 2 ⎥
2 4⎣ ⎦2
3 8 3
= × = .
8 5 5
9
Unit.2. Two Dimensional Random Variables
12 ×1× 2× 3 1
= = .
9 × 8× 7 7
P ( X = 1,Y = 1) = P( drawing 1White and 1 red ball)
2× 3
2C1 × 3C1 9× 8× 7 2
= = =
9C3 1× 2 × 3 7
P ( X = 1,Y = 2) = P( drawing 1White and 2 red ball)
2C1 × 3C2 2 × 3× 2 1
= = =
9C3 9 × 8× 7 14
1× 2 × 3
P ( X = 1,Y = 3) = 0 (Since only three balls are drawn)
P ( X = 2,Y = 0) = P( drawing 2 white balls and no red balls)
2C2 × 4C1 1
= =
9C3 21
P ( X = 2,Y = 1) = P( drawing 2 white balls and no red balls)
2C2 × 3C1 1
= =
9C3 28
P ( X = 2,Y = 2) = 0
P ( X = 2,Y = 3) = 0
The joint probability distribution of ( X ,Y ) may be represented as
Y
0 1 2 3
X
1 3 1 1
0
21 14 7 84
1 2 1
1 0
7 7 14
1 1
2 0 0
21 28
Problem 18.a). Two fair dice are tossed simultaneously. Let X denotes the number on the first
die and Y denotes the number on the second die. Find the following probabilities.
(i) P ( X + Y ) = 8 , (ii) P ( X + Y ≥ 8) , (iii) P ( X = Y ) and (iv)
( )
6
P X +Y = Y .
=4
b) The joint probability mass function of a bivariate discrete random variable ( X ,Y ) in given by
the table.
X
Y 1 2 3
1 0.1 0.1 0.2
2 0.2 0.3 0.1
10
Unit.2. Two Dimensional Random Variables
Find
i. The marginal probability mass function of X and Y .
ii. The conditional distribution of X given Y = 1.
iii. P ( X + Y < 4)
Solution:
a). Two fair dice are thrown simultaneously
⎧(1,1)(1, 2 ) ... (1, 6 ) ⎫
⎪ 2,1 2, 2 ... 2, 6 ⎪
⎪( )( ) ( )⎪
S=⎨ ⎬ , n(S ) = 36
⎪ . . ... . ⎪
⎪( 6,1)( 6, 2 ) ... ( 6, 6 ) ⎪
⎩ ⎭
Let X denotes the number on the first die and Y denotes the number on the second die.
1
Joint probability density function of ( X ,Y ) is P ( X = x,Y = y ) = for
36
x = 1, 2, 3, 4, 5, 6 and y = 1, 2, 3, 4, 5, 6
(i) X + Y = { the events that the no is equal to 8 }
= {(2, 6),(3, 5 ) , ( 4, 4 ) , ( 5, 3 ) , ( 6, 2)}
P ( X + Y = 8) = P ( X = 2,Y = 6) + P ( X = 3,Y = 5) + P ( X = 4,Y = 4)
+ P ( X = 5,Y = 3) + P ( X = 6,Y = 2)
1 1 1 1 1 5
= + + + + =
36 36 36 36 36 36
(ii) P ( X + Y ≥ 8)
⎧( 2, 6 ) ⎫
⎪ ⎪
( 3, 5) , ( 3, 6 )
X + Y = ⎪( 4, 4 ) , ( 4, 5 ) , ( 4, 6 )
⎪
⎨⎪ ⎬⎪
( 5, 3) , ( 5, 4 )( 5, 5 ) , ( 5, 6 )
⎪ ⎪
⎪( 6, 2 ) , ( 6, 3) , ( 6, 4 ) , ( 6, 5 )( 6, 6 ) ⎪
⎩ ⎭
∴ P ( X + Y ≥ 8) = P ( X + Y = 8) + P ( X + Y = 9) + P ( X + Y = 10)
+ P ( X + Y = 11) + P ( X + Y = 12)
5
= + 4 + 3 + 2 + 1 = 15 = 5
36 36 36 36 36 36 12
(iii) P ( X = Y )
P ( X = Y ) = P ( X = 1,Y = 1) + P ( X = 2,Y = 2) +...... + P ( X = 6,Y = 6)
1 1 1
= + + .......... + =
6 1
=
36 36 36 36 6
P ( X + Y = 6 ∩Y = 4)
(
(iv) P X + Y = 6
Y = 4 = ) P (Y = 4)
11
Unit.2. Two Dimensional Random Variables
1
Now P ( X + Y = 6 ∩ Y = 4) =
36
6
P (Y = 4) =
36
1
(
∴P X +Y = 6
Y =4
) = 366 = 61 .
36
b). The joint probability mass function of ( X ,Y ) is
X 1 2 3 Total
Y
1 0.1 0.1 0.2 0.4
2 0.2 0.3 0.1 0.6
Total 0.3 0.4 0.3 1
From the definition of marginal probability function
PX ( xi ) = ∑ PXY ( xi , y j )
yj
When X = 1 ,
PX ( xi ) = PXY (1,1) + PXY (1, 2)
= 0.1+ 0.2 = 0.3
When X = 2 ,
PX ( x = 2) = PXY (2,1) + PXY (2, 2)
= 0.2 + 0.3 = 0.4
When X = 3 ,
PX ( x = 3) = PXY (3,1) + PXY (3, 2)
= 0.2 + 0.1 = 0.3
∴ The marginal probability mass function of X is
⎧0.3 when x = 1
⎪
PX ( x ) = ⎨0.4 when x = 2
⎪0.3 when x = 3
⎩
The marginal probability mass function of Y is given by PY ( y j ) = ∑ PXY ( xi , y j )
xi
3
When Y = 2 , PY ( y = 2) = ∑ PXY ( xi , 2)
xi =1
12
Unit.2. Two Dimensional Random Variables
∴ Marginal probability
⎧0.4 mass function of Y is
P ( y) = when y = 1
⎨
⎩0.6 when y = 2
Y
(
When X =1 , P X = 1
Y =1) =
P ( X = 1∩ Y = 1)
P (Y = 1)
PXY (1,1) 0.1
= = = 0.25
PY (1) 0.4
(
When X = 2 , P X = 2 =
PXY (2,1) 0.1
) = = 0.25
Y =1 PY (1) 0.4
(
When X = 3 , P X = 3
Y =1 =
PXY (3,1) 0.2
) = = 0.5
PY (1) 0.4
(iii). P ( X + Y < 4) = P{( x, y ) / x + y < 4 Where x = 1, 2, 3; y = 1, 2}
= P{(1,1), (1, 2),(2,1)}
= PXY (1,1) + PXY (1, 2) + PXY (2,1)
= 0.1+ 0.1+ 0.2 = 0.4
Problem 19.a). If X and Y are two random variables having the joint density function
1
f ( x, y ) = ( x + 2 y ) where x and y can assume only integer values 0, 1 and 2, find the
27
conditional distribution of Y for X = x .
b). The joint probability density function of ( X ,Y ) is given by
⎧ 2 x2
⎪ xy + , 0 ≤ x ≤ 2, 0 ≤ y ≤ 1
f XY ( x, y ) = ⎨ 8 . Find (i) P( X >1) , (ii) P( X < Y ) and
⎪0 , otherwise
⎩
(iii) P ( X +Y 1)
Solution:
a). Given X and Y are two random variables having the joint density function
1
f ( x, y ) = ( x + 2 y ) − − − −(1)
27
Where x = 0,1, 2 and y = 0,1, 2
Then the joint probability distribution X and Y becomes as follows
13
Unit.2. Two Dimensional Random Variables
Y
0 1 2 f1 ( x )
X
1 2 3
0 0
27 27 27
2 3 4 9
1
27 27 27 27
4 5 6 15
2
27 27 27 27
The marginal probability distribution of X is given by f1 ( X ) = ∑ P ( x, y ) and is calculated in
j
(
P Y =0
) = P (PX( X= 0,Y = 0)
=
0
=0
X =0 = 0) 6
27
2
(
P Y =1
) =
P ( X = 0,Y = 1) 27 1
= =
X =0 P ( X = 0) 6 3
27
4
(
P Y =2
X =0 ) =
P ( X = 0,Y = 2) 27 2
= =
P ( X = 0) 6 3
27
1
(
P Y =0
X =1 ) =
P ( X = 1,Y = 0) 27 1
= =
P ( X = 1) 9 9
27
3
(
P Y =1
X =1 ) =
P ( X = 1,Y = 1) 27 3 1
= = =
P ( X = 1) 9 9 3
27
14
Unit.2. Two Dimensional Random Variables
5
(
P Y =2 P ( X = 1,Y = 2) 27 5
X =1 = ) = =
P ( X = 1) 9 9
27
2
(
P Y =0
X =2
=
P ( X = 2,Y = 0) 27 1
) = =
P ( X = 2) 12 6
27
4
(
P Y =1
X =2
=
P ( X = 2,Y = 1) 27 1
) = =
P ( X = 2) 12 3
27
6
(
P Y =2
X =2
=
P ( X = 2,Y = 2) 27 1
) = =
P ( X = 2) 12 2
27
x2
b). Given the joint probability density function of ( X ,Y ) is f XY ( x + y ) = xy 2 + ,
8
0 ≤ x ≤ 2, 0 ≤ y ≤ 1
∞
(i). P ( X > 1) = f X ( x ) dx
∫ 1
1
15
Unit.2. Two Dimensional Random Variables
1
1
3
⎛ y4 y ⎞ ⎡ y5 y 4 ⎤
= ∫ ⎜ + ⎟ dy = ⎢ + ⎥
2 24 10 96 ⎦ 0
0 ⎝ ⎠ ⎣
1 1 96 +10 53
= + = =
10 96 960 480
(iii) P ( X + Y ≤ 1) = f XY ( x, y ) dxdy
∫∫ R3
1− y
1
⎡ 2 2
= ∫ ⎢ ⎜⎛ x y + x ⎞⎤ dy
3
2 ⎟⎥ y =1
y=0 ⎣ ⎝
24 ⎠⎦
0
( )
1 ⎛ 1− y 2 y 2
(1− y ) ⎞
3
= ⎜
∫ ⎜ 2 + 24 ⎟dy ⎟ x+ y =1
y=0 ⎝ ⎠
1 ⎛
( )
1+ y2 − 2 y y2 (1− y )3 ⎞
= ∫⎜ + 24 ⎟⎟dy x
0⎜ 2
⎝ ⎠ 1
2 y ⎤⎥ 1 + (1− y ) ⎥⎤
4
⎡⎡ y 3
y5 2
=⎢⎢ + − 4 2 96 x =1
⎢⎣ ⎣ 3 5 ⎥⎦ 0
⎦
1 1 1 1
= + − + =
13
.
6 10 4 96 480
16
Unit.2. Two Dimensional Random Variables
∂ 2 F ( x, y )
f ( x, y ) =
∂x∂y
−( x+ y )
∂2 ⎡ –x −y ⎤
=
1− e −e +e
∂x∂y ⎣ ⎦
∂ ⎡ −y
= e − e −( x+ y ) ⎤
∂x ⎧⎣ −( x + y ) ⎦
⎪e , x ≥ 0, y ≥ 0
∴
f ( x, y ) = ⎨
⎪⎩0 , otherwise
(ii) The marginal probability function of X is given by
f ( x) = f X ( x )
∞ ∞
= ∫ f ( x, y ) dy = ∫ e−( x+ y )dy
−∞ 0
∞
−( x+ y )
⎡ ⎤
=⎢ e ⎥
−1
⎣ ⎦0
∞
= ⎡ −e−( x+ y ) ⎤
⎣ ⎦0
=e , x>0
−x
= ∫ f ( x, y ) dx
−∞
∞
− ∞
( x+ y )
= ∫e dx = ⎡ −e−( x+ y ) ⎤
⎣ ⎦0
0
= e− y , y > 0
∴ f ( x) f ( y ) = e−xe− y = e−( ) = f ( x, y )
x+ y
= ∫ f ( x)dx × ∫ f ( y ) dy.
1 1
3 2
= ∫ e−xdx × ∫ e− ydy
1 1
3 2
⎡⎤ e− x
⎡ e− y ⎤
=⎢ ⎥ ⎢× ⎥
⎣ − ⎦1 ⎣ − ⎦1
1
( 1
= −e−3 + e−1 −e −2 + e −1 )( )
= e−5 − e−4 − e−3 + e−2
17
Unit.2. Two Dimensional Random Variables
( )
3
(
b). P 1< Y < 3
X =2 )= f y x = 2 dy
1
4
f X ( x) = ∫ f ( x, y ) dy
2
4 6−x− y
⎛ ⎞
= ∫⎜ ⎟dy
⎝ 8 ⎠ 4
2
1⎛ y2 ⎞
= ⎜ 6 y − xy − ⎟
8⎝ 2 ⎠2
1
= (16 − 4x −10 + 2x )
8
6− x− y
( )
f y
x
=
f ( x, y )
f ( x)
= 8
6 − 2x
=
6− x− y
6 − 2x
3
8
(
P 1< Y < 3
X =2 )=
1
( )
f y x = 2 dy
3 4− y
⎛ ⎞
= ∫⎜ ⎟dy
⎝ 2 ⎠
2
3
1⎡ 2
⎤
= 2 4y − y ⎥
⎢ 2
⎣ ⎦2
3
1⎡ y 2
⎤ 1⎡
= 2 4 y − ⎥ = 2 14 − 17 ⎤⎥ 11 .
⎢ 2 ⎢ 2 = 4
⎣ ⎦2 ⎣ ⎦
Problem 21. a). Two random variables X and Y have the following joint probability density
⎧2 − x − y, 0 ≤ x ≤ 1, 0 ≤ y ≤ 1
function f ( x, y ) = ⎨ . Find the marginal probability density function
⎩0 , otherwise
of X and Y . Also find the covariance between X and Y .
6− x− y
b). If f ( x, y ) = , 0 ≤ x ≤ 2, 2 ≤ y ≤ 4 for a bivariate ( X ,Y ) , find the correlation
8
coefficient
Solution:
⎧2 − x − y, 0 ≤ x ≤ 1, 0 ≤ y ≤ 1
a) Given the joint probability density function f ( x, y ) = ⎨
⎩0 , otherwise
∞
= ∫ ( 2 − x − y ) dy
0
18
Unit.2. Two Dimensional Random Variables
1
⎡ 2
⎤
= 2 y − xy − y ⎥
⎢ 2
⎣ ⎦0
1
= 2− x−
2
⎧3
− x, 0 < x ≤ 1
f X ( x) = ⎪⎨2
⎪⎩0 , otherwise
1
E ( XY ) = ∫ ∫ xy f ( x, y ) dxdy
0 0
1 1
= ∫ ∫ xy ( 2 − x − y ) dxdy
0 0
1 1
19
Unit.2. Two Dimensional Random Variables
1 5 5
Cov ( X ,Y ) = × −
6 12 12
1 25 1
= − =− .
6 144 144
E ( XY ) − E ( X ) E (Y )
b). Correlation coefficient ρXY =
σ Xσ Y
Marginal density function of X is 4 6−x− y
∞ ⎛ ⎞ 6 − 2x
f X ( x ) = ∫ f ( x, y ) dy = ∫ ⎜ ⎟ dy =
8 8
−∞ 2⎝ ⎠
Marginal density function of Y is
2 6− x− y
∞ ⎛ ⎞ 10 − 2 y
fY ( y ) = ∫ f ( x, y ) dx = ∫ ⎜ ⎟ dx =
8 8
−∞ 0 ⎝ ⎠
2 2
⎛ 6 − 2x ⎞
Then E ( X ) = ∫ xf X ( x) dx = ∫ x ⎜ ⎟ dx
8
0 0 ⎝ ⎠
2
2 3
= 1 ⎢⎡ 6x − 2x ⎥⎤
8 2 3
⎣ ⎦0
1⎡ 16 ⎤ 1 20 5
= 12 − = × =
8 ⎢⎣ 13 ⎥ ⎦ 48 3 6
4
⎛ 10 − 2 y ⎞ 1 ⎡ 10 y 2 2 y 3 ⎤ 17
E (Y ) = ∫ y ⎜ dy = − =
8 ⎟ 8⎢ 2 3 ⎥ 6
2 ⎝ ⎠ ⎣ ⎦2 2
( )
E X 2 = x2 f ( x) dx
2 2
= x2
⎛ 6 − 2x ⎞
dx = 1 ⎡ 6x 3
−
2x 4 ⎤
=1
∫
40
x
−
∫0 1⎜⎝ ⎡ 108y 3 ⎟⎠ 2 y 4 8⎤ 4⎢⎣ 325 4 ⎥⎦ 0
⎛ 10 2 y ⎞
E (Y 2 ) = ∫ y2 ⎜ ⎟ dy = ⎢ – ⎥ =
2 ⎝ 8 ⎠ 8 ⎣ 3 4 ⎦2 3
Var ( X ) = σ 2 = E ( X 2 ) − ⎡ E ( X )⎤ = 1−
2 ⎛ 5 ⎞ 2 11
=
X ⎣ ⎦ ⎜ ⎟
⎝ 6⎠ 36
Var ( Y ) = σ = E ( Y ) − ⎡ E ( Y ) ⎤ =
2 25 ⎛ 17 ⎞ 2 11
2 2
− ⎜ ⎟=
Y ⎣
⎦ 3 ⎝ 6 ⎠ 36
4 2
⎛6−x− y ⎞
E ( XY ) = ∫ ∫ xy ⎜ ⎟ dxdy
⎝ 8 ⎠
2 0
2
1 4 ⎡ 6x 2 y x 3 y x 2 y 2 ⎤
= 8∫⎢ 2
– − ⎥ dy
2⎣
3 2 ⎦0
4
4 8 2⎞ ⎡ 2 3
⎤
= ∫ ⎜12 y − y − 2 y dy = ⎢ 2 y
1 12 y 2
1 ⎛ 8 y
⎟ – − ⎥
82⎝ 3 ⎠ 8⎣ 2 3 2 3 ⎦2
20
Unit.2. Two Dimensional Random Variables
1⎡ 64 128 16 16 ⎤ 1 ⎡ 56 ⎤
= 96 − − − 24 + + =
8⎢⎣ 3 3 ⎥ ⎢
3 3 ⎦ 8 ⎣ 3 ⎦
⎥
7
E ( XY ) =
3
7 ⎛⎜5 ⎞⎛
⎟⎜17 ⎞ ⎟
−
E ( XY ) − E ( X ) E (Y )
6 6
ρXY 3 ⎝ ⎠⎝ ⎠
= =
σ XσY 11 11
6 6
1
ρ =− .
XY
11
Problem 22 a). Let the random variables X and Y have pdf
1
f ( x, y ) = , ( x, y ) = (0, 0),(1,1), (2, 0) . Compute the correlation coefficient.
3
b) Let X1 and X 2 be two independent random variables with means 5 and 10 and standard
devotions 2 and 3 respectively. Obtain the correlation coefficient of UV where U = 3X1 + 4 X 2
and V = 3X1 − X 2 .
Solution:
a). The probability distribution is
X 0 1 2 P (Y )
Y
1 1
0 3 0 0 3
1 1
1 0 3 0 3
1 1
0 0 0 3 3
P( X ) 1 1 1
3 3 3
⎛ 1⎞ ⎛ 1⎞ ⎛ 1⎞
E ( X ) = ∑ x p ( x ) = 0 × + 1× + 2 × = 1
i i i ⎜ 3⎟ ⎜ 3⎟ ⎜ 3 ⎟
i
⎛⎝ 1 ⎞ ⎠ ⎛ ⎝ 1 ⎞ ⎛⎠ ⎝1 ⎞ 1⎠
E ( Y ) = ∑ y p ( y ) = 0 × + 1× + 0 × = ⎟
i j j ⎜ 3⎟ ⎜ 3⎟ ⎜ 3 3
j ⎝ 1 ⎠ ⎝ 1 ⎠ ⎝1 5⎠
( ) ⎛ ⎞ ⎛
E X 2 = ∑ x 2 p ( x ) = 0 × + 1× + 4 × = ⎟
⎜
⎞ ⎛ ⎞
i i
3⎟ ⎜ 3⎟ ⎜ 3 3
i ⎝ ⎠ ⎝ ⎠ ⎝ ⎠
( )
Var( X ) = E X 2 − ⎡⎣ E ( X ) ⎤⎦ = −1 =
2 5
3
2
3
21
Unit.2. Two Dimensional Random Variables
( )
E Y2 = ∑ y 2 p( y
j
) = ⎛⎜0 × 1 ⎞ +⎟ ⎛1×⎜ 1 ⎞ + ⎟⎛ 0 × 1⎜ ⎞ = 31⎟
j 3
3 3
j ⎠ ⎝ ⎠ ⎝⎝ ⎠
1 1 2
∴V (Y ) = E Y 2 − E ( Y )⎤⎦ = − =
2
( ) 3 9 9
E ( XY ) − E ( X ) E (Y )
Correlation coefficient ρXY =
V ( X )V (Y )
E ( XY ) = ∑∑ xi y j p ( xi , y j )
i j
1 1 1 1
= 0.0. + 0.1.0 +1.0.0 +1.1. +1.2.0 + 0.0.0 + 0.1.0 + 0.2. =
3 3 3 3
1 ⎛1⎞
− (1) ⎜ ⎟
ρXY = 3 ⎝ 3⎠ = 0
2 2
×
3 9
Correlation coefficient = 0 .
b). Given E ( X1 ) = 5, E ( X 2 ) = 10
V ( X1 ) = 4, V ( X 2 ) = 9
Since X and Y are independent E ( XY ) = E ( X ) E (Y )
E (UV ) − E (U ) E (V )
Correlation coefficient =
Var (U )Var (V )
E (U ) = E (3X1 + 4X 2 ) = 3E ( X1 ) + 4E ( X 2 )
= (3× 5) + ( 4 ×10) = 15 + 40 = 55.
E (V ) = E (3X1 − X 2 ) = 3E ( X1 ) − E ( X 2 )
= (3× 5) −10 = 15 −10 = 5
E (UV ) = E ⎡⎣ ( 3X 1 + 4 X 2 )( 3X 1 − X 2 )⎤⎦
= E ⎡⎣ 9X 1 − 3X 1 X 2 +12 X 1 X 2 − 4 X 2 ⎤⎦
2 2
( ) ) − 4E ( X )
= 9E X 12 − 3E ( X 1 X 2) +12E ( X X1 2
2
2
= 9E ( X ) + 9E ( X X ) − 4E ( X )
2
1 1 2
2
2
= 9E ( X ) + 9E ( X ) E ( X ) − 4E ( X )
2
1 1 2
2
2
= 9E ( X ) + 450 − 4E ( X )
2 2
V ( X ) = E ( X ) − ⎡ E ( X )⎤
1 2 2
2
1 ⎣ 1 ⎦ 1
E ( X ) = V ( X ) + ⎡⎣ E ( X ) ⎤ = 4 + 25 = 29
2
2
1 ⎦ 1 1
E ( X ) = V ( X ) + ⎡⎣ E ( X ) ⎤ = 9 +100 = 109
2
2
2
⎦
2 2
22
Unit.2. Two Dimensional Random Variables
Solution:
1 1 1
2 2
⎡ x2 ⎤ 2
a). We have E ( X ) = ∫ xf ( x) dx = ∫ xdx = ⎢ ⎥ =0
−
1
−
1 ⎣2 ⎦ − 1
2 2 2
1
0 x+1 2 1−x
E ( XY ) = ∫ ∫ xydxdy + ∫ ∫ xydxdy
−
1 x 0 −x
2
1
0
⎡ x+1 ⎤ 2
⎡ 1− x ⎤
=∫x⎢ ∫ ydy ⎥ dx + ∫ x ⎢ ∫ ydy ⎥ dx
−1 ⎣ ⎦ 0 ⎣ −x ⎦
x
1
2
12
0
1
=
2 ∫ x ( 2x +1) dx + 2 ∫ x (1 − 2x ) dx
1 0
−
2
1
1 ⎡ 2x3 x2 ⎤ 0 1 ⎡ x2 2x3 ⎤ 2
= 2⎢ 3 + 2 ⎥ + 2⎢ 2 − 3 ⎥ =0
⎣ ⎦ −1 ⎣ ⎦0
2
Since Cov ( X ,Y ) = E ( XY ) − E ( X ) E (Y ) = 0 , the variables X and Y are uncorrelated.
y
b). Regression curve of Y on X is E x ( )
( x ) = ∫ yf ( y x ) dy
∞
E y
−∞
f ( x, y )
f ( y / x) =
fX ( X )
23
Unit.2. Two Dimensional Random Variables
= x ∫ e − x( y +1) dy
0
∞
− x( y+1)
⎡ e ⎤
⎥ = e−x x ≥ 0
= x⎢
−x ,
⎣ ⎦0
f
X ( x )( )
Conditional pdf of Y on X is f y x = f ( x, y ) = xe− − = xe− xy
e –x
xy x
( x ) =∫
∞
E y yxe−xydy
0
∞
⎡ e− xy e− xy ⎤
− x⎢ y
= x2 ⎥
⎣ −x ⎦0
f ( x, y )
f (Y ) =
X f ( x) X
∞
f X ( x) = ∫ f ( x, y ) dy
−∞ 2
⎛ x+ y⎞
2 1⎡ y2 ⎤
=∫⎜
⎟ dy = ⎢ xy + ⎥
3 3 2
0⎝ ⎠ ⎣ ⎦0
2 ( x +1)
=
3
( )
f Y =
f ( x, y ) = x + y
X f X ( x) 2(x +1)
24
Unit.2. Two Dimensional Random Variables
(
2 y x+ y
)
Regression of Y on X = E Y (X ) = ∫ 2 ( x +1) dy
0
2
= 1 ⎢⎡ xy2 + y 3⎥ ⎤
2 ( x +1) 2 3
⎣ ⎦0
1 ⎡ 8⎤
= 2x + = 3x + 4
2 ( x +1) ⎢ ⎣ 3⎥ ⎦ 3 x +1
( )
( Y ) = ∫ xf ( x y ) dx
∞
E X
−∞
f ( x y ) = f f( x,( yy))
Y
∞
fY ( y ) = ∫ f ( x, y ) dx
−∞ 1
⎛ x+ y⎞
1 1 ⎡ x2 ⎤
=∫⎜
⎟ dx = ⎢ + xy ⎥
⎝ 3 ⎠ 3 2
0 ⎣ ⎦0
1⎡ 1 ⎤
= +y
3⎢⎣ 2 ⎥ ⎦
2( x + y)
y( )
f x =
2 y +1
( Y) x+ y
1
Regression of X on Y = E X =∫ dx
0 2 y +1
1
2
= 1 ⎡⎢ x ⎤
2 y +1 2 + xy ⎥
⎣ ⎦0
1
+y
2 1
= = .
2 y +1 2
b).
1. Correlation means relationship between two variables and Regression is a Mathematical
Measure of expressing the average relationship between the two variables.
2. Correlation need not imply cause and effect relationship between the variables. Regression
analysis clearly indicates the cause and effect relationship between Variables.
3. Correlation coefficient is symmetric i.e. rxy = ryx where regression coefficient is not symmetric
4. Correlation coefficient is the measure of the direction and degree of linear relationship
between two variables. In regression using the relationship between two variables we can predict
the dependent variable value for any given independent variable value.
25
Unit.2. Two Dimensional Random Variables
Problem 25. a). X any Y are two random variables with variances σ 2 x and σ 2y respectively and
yσ x
r is the coefficient of correlation between them. If U = X + KY andV = X + , find the
σy
value of k so that U and V are uncorrelated.
b). Find the regression lines:
X 6 8 10 18 20 23
Y 40 36 20 14 10 2
Solution:
GivenU = X + KY
E (U ) = E ( X ) + KE (Y )
σ
V = X + XY
σY
σ
E (V ) = E ( X ) + X E (Y )
σY
If U and V are uncorrelated, Cov (U ,V ) = 0
E ⎡⎣ (U − E (U ) )(V − E (V ) ) ⎤⎦ = 0
⎡ ⎛ σX σX ⎞⎤
⇒
E ⎢ ( X + KY − E ( X ) − KE (Y ) ) × ⎜ X + Y − E( X )− E (Y ) ⎟⎥ = 0
⎣ ⎝ σ Y σ Y ⎠⎦
⎡ σX ⎫
⎤⎪
⇒ E ⎨ ⎡ ( X − E ( X )) + K (Y − E (Y ) ) ⎤ × ⎢( X − E ( X ) ) + (Y − E (Y ) ) =⎥ ⎬0
⎣ ⎦
⎩ ⎣ σ
Y ⎦⎭
⎧ σX σ ⎫
⇒ E ( X − E ( X ) ) + ( X − E ( X ))( Y − E (Y ) ) + K ( Y − E (Y ) )( X − E ( X ) ) + K (Y − E (Y ) ) = 0
2 2
X
⎨ ⎬
⎩ σY Y σ ⎭
σX σX
⇒V (X )+ Cov ( X ,Y ) + KCov ( X ,Y ) + K V (Y ) = 0
σY σY
⎡ σX ⎤ σX
K ⎢ Cov ( X ,Y ) + − (Y )⎥ = −V ( X )
V Cov ( x, y )
⎣ σY ⎦ σY
σ
−V ( X ) − X rσ σ
σY X Y
−σ 2X − rσ 2 X
K= σ =
rσ X σ Y + X
V (Y ) rσ X σ Y + σ X σ Y
σY
−σ 2X (1+ r ) = − σ X .
=
σ X σ Y (1+ r ) σY
26
Unit.2. Two Dimensional Random Variables
b).
X Y X2 Y2 XY
6 40 36 1600 240
8 36 64 1296 288
10 20 100 400 200
18 14 324 196 252
20 10 400 100 200
23 2 529 4 46
∑x 85 ∑y 122
X = = = 14.17 , Y = = = 20.33
n 6 n 6
2 2
∑ x2 ⎛ ∑ x ⎞ 1453 ⎛ 85 ⎞
σx = –⎜ ⎟ = – ⎜ ⎟ = 6.44
n ⎝ n ⎠ 6 ⎝6 ⎠
2
∑ y2 ⎛ ∑ y ⎞
2
3596 ⎛ 122 ⎞
σy = –⎜ ⎟ = – = 13.63
n ⎝ n ⎠ 6 ⎜⎝ 6 ⎠⎟
1226
∑ xy
−xy − (14.17)(20.33)
r= n = 6 = −0.95
σ xσ y (6.44)(13.63)
σx 6.44
bxy = r = −0.95× = −0.45
σy 13.63
σy 13.63
byx = r = −0.95× = −2.01
σx 6.44
The regression line X on Y is
( )
x − x = bxy y − y ⇒ x −14.17 = −0.45 y − y ( )
⇒ x = −0.45 y + 23.32
The regression line Y on X is
( )
y − y = byx x − x ⇒ y − 20.33 = −2.01( x −14.17 )
⇒ y = −2.01x + 48.81
Problem 26. a) Using the given information given below compute x , y and r . Also compute
σ y when σ x = 2, 2x + 3y = 8 and 4x + y = 10 .
b) The joint pdf of X and Y is
27
Unit.2. Two Dimensional Random Variables
X
Y -1 1
1 3
0 8 8
2 2
1 8 8
Find the correlation coefficient of X and Y .
Solution:
a). When the regression equation are Known the arithmetic means are computed by solving the
equation.
2x + 3y = 8------------- (1)
4x + y = 10 ------------ (2)
(1) × 2 ⇒ 4x + 6 y = 16 --------- (3)
( 2) − ( 3) ⇒ −5 y = −6
6
⇒y=
5
⎛6⎞
Equation (1) ⇒ 2x + 3 =8
⎜5 ⎟
⎝ ⎠
18
⇒ 2x = 8 −
5
11
⇒x=
5
11 6
i.e. x = & y =
5 5
To find r , Let 2x + 3y = 8 be the regression equation of X on Y .
3
2x = 8 − 3y ⇒ x = 4 − y
2
3
⇒ bxy = Coefficient of Y in the equation of X on Y = −
2
Let 4x + y = 10 be the regression equation of Y on X
⇒ y = 10 − 4x
⇒ byx = coefficient of X in the equation of Y on X = −4 .
r = ± bxybyx
= − ⎛ − 3 ⎞ −4
⎜ 2 ⎟( )
( b &b
xy yx
are negative )
⎝ ⎠
= −2.45
Since r is not in the range of (−1 ≤ r ≤ 1) the assumption is wrong.
Now let equation (1) be the equation of Y on X
28
Unit.2. Two Dimensional Random Variables
8 2x
⇒ y= −
3 3
⇒ byx = Coefficient of X in the equation of Y on X
2
b =−
yx
3
from equation (2) be the equation of X on Y
1
b =−
xy
4
2 1
r = ± bxybyx = – × − = 0.4081
3 4
2
To compute σ from equation ( 4 ) b = −
y yx
3
σy
But we know that byx = r σ
x
2 σy
⇒− = 0.4081×
3 2
⇒ σ y = −3.26
b). Marginal probability mass function of X is
1 3 4
When X = 0, P ( X ) = + =
8 8 8
2 2 4
X = 1, P ( X ) = + =
8 8 8
Marginal probability mass function of Y is
1 2 3
When Y = −1, P (Y ) = + =
8 8 8
3 2 5
Y = 1, P (Y ) = + =
8 8 8
4 4 4
E ( X ) = ∑ x p ( x) = 0 × +1× =
x 8 8 8
3 5 3 5 2
E (Y ) = ∑ y p ( y ) = −1× +1× = − + =
y 8 8 8 8 8
( ) 4
E X 2 = ∑ x2 p ( x) = 02 × +12 × =
4 4
x 8 8 8
E (Y ) = ∑ y p ( y ) = ( −1) × +1 × = + = 1
2 2 2 3 2 5 3 5
y 8 8 8 8
( )
V ( X ) = E X 2 − ( E ( X ))
2
2
4 ⎛4⎞ 1
= −⎜ ⎟=
8 ⎝8 ⎠ 4
29
Unit.2. Two Dimensional Random Variables
V ( Y ) = E (Y 2 ) − ( E (Y ) )
2
2
⎛ 1 ⎞ 15
= 1− ⎜ ⎟ =
⎝ 4 ⎠ 16
E ( XY ) = ∑∑ xy p ( x, y )
x
1y 3 2 ⎛2⎞
= 0× + 0 × + ( −1) +1× =0
⎜
⎝8⎟ ⎠
8 8 8
1 1 1
Cov ( X ,Y ) = E ( XY ) − E ( X ) E (Y ) = 0 − × = −
1 2 4 8
Cov ( X ,Y ) −
r= = 8 = −0.26 .
V ( X ) V (Y ) 1 15
4 16
Problem 27. a) Calculate the correlation coefficient for the following heights (in inches) of
fathers X and their sons Y .
X 65 66 67 67 68 69 70 72
Y 67 68 65 68 72 72 69 71
b) If X and Y are independent exponential variates with parameters 1, find the pdf of
U = X −Y .
Solution:
X Y XY X2 Y2
65 67 4355 4225 4489
66 68 4488 4359 4624
67 65 4355 4489 4285
68 72 4896 4624 5184
69 72 4968 4761 5184
70 69 4830 4900 4761
72 71 5112 5184 5041
∑x 544
X= = = 68
n 8
∑y 552
Y= = = 69
n 8
XY = 68× 69 = 4692
30
Unit.2. Two Dimensional Random Variables
1 2 1
σX = ∑ x2 − X = (37028) − 682 = 4628.5 − 4624 = 2.121
n 8
1 1
σY = ∑ y2 − y2 = (38132) − 692
= 4766.5 − 4761 = 2.345
n 8
1 1
Cov ( X ,Y ) = ∑ XY − X Y = (37650) − 68× 69
n 8
= 4695 − 4692 = 3
The correlation coefficient of X and Y is given by
Cov ( X ,Y ) 3
r ( X ,Y ) = =
σ XσY (2.121)(2.345)
3
=
= 0.6032 .
4.973
b). Given that X and Y are exponential variates with parameters 1
f X ( x) = e−x , x ≥ 0, f Y( y ) = e− y , y ≥ 0
Also f XY ( x, y ) = f X ( x) f y ( y ) since X and Y are independent
= e− xe− y
= e−( x+ y ) ; x ≥ 0, y ≥ 0
Consider the transformations u = x − y and v = y
⇒ x = u +v, y = v v
∂x ∂x
∂ ( x, y ) ∂v 1 −1
J = = ∂u = =1
∂ (u, v) ∂y ∂y 0 1
∂u ∂v
fUV (u, v) = f XY ( x, y ) J = e− xe− y = e ( )e−v
− u +v
= e−(u+2v) , u + v ≥ 0, v ≥ 0 RI R II
In Region I when u < 0 v= u
∞ ∞
= 0−e ⎦ = ⎤
2u
⎡
−2 ⎣ 2
In Region II when u > 0
∞
f (u ) = ∫ f (u, v)dv
0
∞ −( u +2v ) −u
= e dv = e
∫
0 2
31
Unit.2. Two Dimensional Random Variables
⎧ eu
, u<0
∴ f (u ) = ⎨ ⎪ 2
−u
⎪e
u>0
,
⎩⎪ 2
Problem 28. a) The joint pdf of X and Y is given by f ( x, y ) = e−(x+ y) , x > 0, y > 0 . Find the
X +Y
pdf of U = .
2
b) If X and Y are independent random variables each following N (0, 2) , find the pdf of
Z = 2 X + 3Y . If X and Y are independent rectangular variates on (0,1) find the distribution of
X
.
Y
Solution:
x+ y
a). Consider the transformation u = &v = y
2
⇒ x = 2u − v and y = v
∂x ∂x
∂ ( x, y ) ∂v 2 −1
J= = ∂u = =2
∂ (u, v) ∂y ∂y 0 1
∂u ∂v
fUV (u, v) = f XY ( x, y ) J
= e−( x+ y ) 2 = 2e−( x+ y) = 2e−(2u −v+v)
= 2e−2u , 2u − v ≥ 0, v ≥ 0
u
fUV (u, v) = 2e−2u , u ≥ 0, 0 ≤ v ≤
2
u u
2 2
32
Unit.2. Two Dimensional Random Variables
∂x ∂x 1
∂ –3 1
J =
( x, y ) =
∂z ∂w =
2
2 = .
∂ ( z, w)
∂y ∂y
0 1
2
∂z ∂w
Given that X and Y are independent random variables following N (0, 2)
(
− x2 + y2 )
∴ f XY ( x, y ) = 1 e 8 , −∞ < x, y < ∞
8π
The joint pdf of ( z, w) is given by
fZW ( z, w) = J f XY ( x, y )
⎡1 ⎤
( z −3w)
2
− + w2
1 1 ⎢ ⎥
= . e ⎣4 8 ⎦
2 8π 1
1 − ⎡ ( z −3w) +4 w ⎤ 2 2
= e 32 ⎥ ⎦ , −∞ < z, w < ∞ .
16π ⎢ ⎣
The pdf of z is the marginal pdf obtained by interchanging fZW ( z, w) w.r.to w over the range of
w.
1 ∞ ⎛ − 1 ( z −6 wz +13w ) ⎞ 2 2
∴ fZ ( z ) =
∫ ⎜e 32
16π −∞ ⎝
⎟dw
⎠
z ∞ ⎛ − 13 ⎜ w − 6 wz +⎛⎜3z ⎞⎟ −⎛⎜3 z ⎞⎟ ⎟ ⎞
2
⎛ ⎞
2
2 2
–
1
e 32 ∫ ⎜ e 32 ⎝ 13 ⎝ 13 ⎠ ⎝ 13 ⎠ ⎠ ⎟dw
⎜ ⎟
=
16π −∞ ⎜ ⎟
⎝ ⎠
∞ ⎛ 3z ⎞ ⎞
2
z 9z2 13 ⎛
2
z2 ∞ 13 2
– ⎟
1
∫ e dt
− + ⎜ w− ⎟ dw −32 t
= e 32 13×32 ∫ ⎜ e 32 ⎝ 13 ⎠ 1 − 8×13
16π −∞
⎜ ⎟ = 16π e −∞
⎝ ⎠
13 2 13 16 r32
r = t ⇒ dr = tdt ⇒ dr = dt ⇒ dr = dt
32 16 13t 13
16 13 4 –
1
dr = dt ⇒ r 2 dr = dt
13 r32 13 × 2
2 4 –
z ∞
−
1 2
2
dr
= 16π 13 × 2 e 8×13 ∫0 e−r r
z2
z2 ∞ 1 z2 −
1 − −
e 2( 2 )
1 – 1 2
= ∫e r π =
−r 13
e 8×13 2
dr = e 8×13
2π 13 × 2 0 2π 13 × 2 2 13 2π
i.e. Z (
N 0, 2 13 )
b).(ii) Given that X and Y are uniform Variants over (0,1)
⎧1, 0 < x < 1 ⎧1, 0 < y < 1
∴ fX ( x) = ⎨ and f Y ( y ) = ⎨
⎩0, otherwise ⎩ 0, otherwise
33
Unit.2. Two Dimensional Random Variables
Problem 29. a) If X1, X 2 , .....Xn are Poisson variates with parameter λ = 2 . Use the central limit
theorem to estimate P (120 < Sn < 160) where sn = X1 + X 2 + ...... + Xn and n = 75 .
b) A random sample of size 100 is taken from a population whose mean is 60 and variance is
400. Using central limit theorem, with what probability can we assent that the mean of the
sample will not differ from μ = 60 by more than 4.
Solution:
a). Given that E ( X i ) = λ = 2 and Var ( X i ) = λ = 2
34
Unit.2. Two Dimensional Random Variables
= P ⎡⎣ −4 ≤ X − 60 ≤ 4 ⎤⎦
⎡ 56 − 60 64 − 60 ⎤
= P ⎡ 56 ≤ X ≤ 64 ⎤ = P ≤z≤
⎣ ⎦ ⎣⎢ 2 2 ⎥⎦
= P[−2 ≤ Z ≤ 2]
= 2P [0 ≤ Z ≤ 2] = 2 × 0.4773 = 0.9446
Problem 30 a) If the variable X1, X 2 , X 3 , X 4 are independent uniform variates in the interval
(450, 550) , find P (1900 ≤ X1 + X 2 + X 3 + X 4 ≤ 2100) using central limit theorem.
b) A distribution with unknown mean μ has a variance equal to 1.5. Use central limit
theorem to find how large a sample should be taken from the distribution in order that the
probability will be at least 0.95 that the sample mean will be within 0.5 of the population mean.
Solution
a). Given that X follows a uniform distribution in the interval (450, 550)
35
Unit.2. Two Dimensional Random Variables
b + a 450 + 550
Mean = = = 500
2 2
( b − a ) ( 550 − 450 )
2 2
Variance = = = 833.33
12 12
By CLT S n= X 1+ X 2+ X 3+ X follows
4 a normal distribution with N ( nμ , nσ 2 )
S − nμ
The standard normal variable is given by Z = n
nσ 2
1900 − 4× 500 100
when Sn = 1900 , Z = =− = −1.732
4× 833.33 57.73
2100 − 2000
when S = 2100 , Z = = 100 = 1.732
n 57.73
4× 833.33
∴ P (1900 ≤ Sn ≤ 2100) = P (−1.732 < z < 1.732)
= 2 × P (0 < z < 1.732) = 2× 0.4582 = 0.9164 .
( )
i.e. P −0.5 < X − μ < 0.5 ≥ .95
( )
P X − μ < 0.5 ≥ .95
⎡ ⎤
σ
P⎢ < 0.5⎥ ≥ 0.95
z n ⎦
⎣
⎡
n⎤
P ⎢ z < 0.5 ⎥ ≥ 0.95
⎣ σ ⎦
⎡ n ⎤
P ⎢ z < 0.5 ⎥ ≥ 0.95
⎣ 1.5 ⎦
( )
ie P Z < 0.4082 n ≥ 0.95
Where ' Z ' is the standard normal variable.
The Last value of ' n ' is obtained from P (Z )
< 0.4082 n = 0.95
( )
2P 0 < z < 0.4082 n = 0.95
⇒ 0.4082 n = 1.96 ⇒ n = 23.05
∴The size of the sample must be atleast 24.
36
Unit.4. Correlation and spectral densities
PART-A
Problem1. Define autocorrelation function and prove that for a WSS process { X (t )},
RXX (−τ ) = RXX (τ ) 2.State any two properties of an autocorrelation function.
Solution:
Let {X (t )}be a random process. Then the auto correlation function of the process
{ X (t )} is the expected value of the product of any two members X (t1 ) and X (t2 ) of
the process and is given by RXX (t1, t2 ) = E ⎣ ⎡ X (t1 ) X (t2 )⎤ ⎦ or
RXX ( t, t + τ ) = E ⎡⎣ X ( t ) X ( t + τ ) ⎤⎦
For a WSS process{X (t )}, RXX (τ ) = E ⎡ ⎣ X (t ) X (t −τ )⎤ ⎦
∴ RXX ( −τ ) = E ⎡⎣ X ( t ) X ( t + τ ) ⎤⎦ = E ⎡⎣ X ( t + τ ) X ( t ) ⎤⎦ = R ( t + τ − t ) = R (τ )
Problem 2. State any two properties of an autocorrelation function.
Solution:
The process { X ( t )} is stationary with autocorrelation function R (τ ) then
( i ) R (τ ) is an even function of τ
( ii ) R (τ ) is maximum at τ = 0 i.e., R (τ ) ≤ R ( 0 )
Problem 3. Given that the autocorrelation function for a stationary ergodic process with
4
no periodic components is R (τ ) = 25 + . Find the mean and variance of the
1+ 6τ 2
process{X (t )}.
Solution:
Lt Lt 4
μ 2x = R (τ ) = 25 + = 25
τ →∞ τ →∞ 1+ 6τ 2
∴μx = 5
( )
E X 2 (t ) = R XX ( 0) = 25 + 4 = 29
Var ( X ( t ) ) = E ⎡⎣ X 2 ( t ) ⎤⎦ − E ⎡⎣ X ( t ) ⎤⎦ = 29 − 25 = 4
2
Problem 4. Find the mean and variance of the stationary process { X (t )} whose
25τ 2 + 36
autocorrelation function R (τ ) = .
6.25τ 2 + 4
Solution:
1
Unit.4. Correlation and spectral densities
36
25 +
25τ 2 + 36
R (τ ) = = τ2
6.25τ 2 + 4 6.25 + 2
4
τ
Lt
μ2 = R (τ ) = 25 2500
= =4
τ →∞
x
6.25 625
μx = 2
36
E ⎡ ⎣ X 2 (t )⎦ ⎤ = RXX ( 0 ) = = 9
4
{ }
Var X ( t )⎤⎦ = E ⎡⎣ X ( t ) ⎤⎦ − E ⎡⎣ X ( t ) ⎤⎦ = 9 − 4 = 5
2 2
Problem 6. Find the mean – square value of the process { X (t )} if its autocorrelation
−τ 2
function is given by R (τ ) = e 2
.
Solution:
⎛ τ2 ⎞
−
= RXX ( 0 ) = e
Mean-Square value = E ⎡ ⎣ X
⎜ ⎟
2
=1 (t )⎤ ⎦ 2
⎝ ⎠τ =0
Problem 7. Define the power spectral density function (or spectral density or power
spectrum) of a stationary process?
Solution:
If {X (t )} is a stationary process (either in the strict sense or wide sense with auto
correlation function R (τ ) , then the Fourier transform of R (τ ) is called the power
spectral density function of { X (t )} and denoted by SXX (ω ) or S (ω ) or S X (ω )
∞
Thus S ( ω ) = ∫ R (τ ) e −iωτ dτ
−∞
Problem 8. State any two properties of the power spectral density function.
Solution:
(i). The spectral density function of a real random process is an even function.
2
Unit.4. Correlation and spectral densities
(ii). The spectral density of a process{X (t )}, real or complex, is a real function of ω and
non-negative.
Problem 9. State Wiener-Khinchine Theorem.
Solution:
If X T (ω ) is the Fourier transform of the truncated random process defined as
⎧⎪ X (t ) for t ≤ T
XT (t ) = ⎨
⎪⎩0 for t > T
Where { X (t )} is a real WSS Process with power spectral density function S (ω ) , then
S (ω ) = Lt ⎡ 1
E X
(ω ) ⎤
{
2
}
T → ∞ ⎢ ⎣ 2T T ⎥⎦
Problem 10. If R (τ ) = e −2 λ (τ ) is the auto Correlation function of a random
process{X (t )}, obtain the spectral density of { X (t )}.
Solution:
∞
S ( ω ) = ∫ R (τ ) e −iωτ dτ
−∞
∞
= ∫ e −2 λ τ ( cosωτ − isinωτ ) dτ
−∞
∞
= 2∫ e −2λτ cosωτ dτ
0
∞
⎡ 2e −2 λτ ⎤
2 (
=⎢ 2 −2λ cosωτ + ω sinωτ ) ⎥
⎣ 4λ + ω ⎦0
4λ
S (ω ) =
4λ 2 + ω 2
Problem 11. The Power spectral density of a random process { X (t )} is given by
⎧⎪π , ω < 1
SXX (ω ) = ⎨ Find its autocorrelation function.
⎪0,
⎩ elsewhere
Solution:
1∞
RXX (τ ) = ∫ SXX ( ω ) eiωτ d ω
2π −∞
1 1
= π eiωτ d ω
∫
2π −1 1
= ⎡⎢ e ωτ ⎤⎥
i
1
2 iτ
⎣ ⎦ −1
⎢1 ⎡ eiτ − e −iτ ⎤ ⎥
=2 iτ
⎣ ⎦
3
Unit.4. Correlation and spectral densities
1 ⎢⎡ eiτ − 2i
e − iτ ⎤ ⎥ sinτ τ
=τ =
⎣ ⎦
Problem 12. Define cross-Spectral density.
Solution:
The process { X (t )} and {Y (t )} are jointly wide-sense stationary with the cross-
correlation function RXY (τ ) , then the Fourier transform of RXY (T ) is called the cross
spectral density function of { X (t )} and {Y (t )} denoted as SXY (ω )
∞
Thus SXY (ω ) = ∫ R (τ )e
XY
−iωτ
dτ
−∞
Problem 13. Find the auto correlation function of a stationary process whose power
ω ≤1
spectral density function is given by s (ω ) = ⎨⎪ω⎧ for
2
.
⎪⎩0 for ω > 1
Solution:
1∞
R (τ ) = S ( ω )eiωτ d ω
∫
2π −∞
1 1 2
= ∫ ω ( cos ωτ + i sin ωτ )dω
2π −1
1 ⎡ 2 ⎛ sin ωτ ⎞ ⎛ −cos ωτ ⎞ ⎛ − sin ωτ ⎞⎤ 1
+ 2⎜
1
= ∫ ω 2 .cos ωτ dω = ω
⎢ ⎜ ⎟ − 2ω ⎜ 2 ⎟ 3 ⎟⎥
−1 π⎣ ⎝ τ ⎠ ⎝ τ ⎠ ⎝ τ ⎠⎦ 0
1 ⎡ − sin τ 2 cos τ 2 sin τ ⎤
R (τ ) = + −
π⎢ ⎣ τ τ2 τ3 ⎥⎦
1
Problem 14. Given the power spectral density : Sxx (ω ) = , find the average power
4 + ω2
of the process.
Solution:
1∞
R (τ ) = S ( ω )eiωτ d ω
2π −∞ ∫
∞
1 ⎡ 1 ⎤ eiωτ d ω
= ∫ ⎢
2π −∞ ⎣ 4 + ω ⎦
2⎥
2π ∫−∞ 4 + ω 2
=
1 ∞ dω
= 2∫ 2
2π 02 + ω 2
4
Unit.4. Correlation and spectral densities
1 ⎡ 1 −1 ⎛ ω⎜ ⎞⎤ ⎟ ∞
= tan
π ⎢⎣ 2 ⎝ ⎠⎥⎦ 0
1 ⎡ π ⎤ 12
= −0 = .
π ⎢⎣ 4 ⎦⎥ 4
Problem 15. Find the power spectral density of a random signal with autocorrelation
function e −λ τ .
Solution:
∞
S ( ω ) = ∫ R (τ )e −iωτ dτ
−∞
∞
= 2 ∫ e −λ τ cos ωτ dτ
0
∞
⎡ e− λτ ⎤
2(
= 2⎢ −λ cos ωτ + ω sin ωτ )⎥
⎣ λ +ω
2
⎦0
⎡
= 2 0− 1
(−λ )⎤ = 2 2λ 2
⎣ λ +ω
2 2
⎦ λ +ω
PART-B
Problem 16. a). If { X ( t )} is a W.S.S. process with autocorrelation function RXX (τ ) and
if Y (t ) = X (t + a ) − X (t − a ) .Show that RYY (τ ) = 2RXX (τ ) − RXX (τ + 2a ) − RXX (τ − 2a ) .
Solution:
Ryy (τ ) = E ⎡⎣ y ( t ) y ( t + τ ) ⎤⎦
{ }
= E ⎡⎣ X ( t + a ) − X ( t − a ) ⎤⎦ ⎡⎣ X ( t + τ + a ) − X ( t + τ − a ) ⎤⎦
= E ⎡⎣ X ( t + a ) X ( t + τ + a ) ⎤⎦ − E ⎡⎣ X ( t + a ) X ( t + τ − a )⎤⎦
– E ⎣⎡ X ( t − a ) X ( t + τ + a ) ⎤⎦ + E ⎣⎡ X ( t − a ) X ( t + τ − a ) ⎤⎦
= Rxx (τ ) − E ⎡⎣ X ( t + a ) X ( t + a + τ − 2a ) ⎤⎦
– E ⎡⎣ X ( t − a ) X ( t − a + τ + 2a ) ⎤⎦ + Rxx (τ )
= 2Rxx (τ ) − Rxx (τ − 2a) − Rxx (τ + 2a)
b). Assume a random signal Y (t ) = X (t ) + X (t − a ) where X (t ) is a random signal and
' a ' is a constant. Find RYY (τ ) .
Solution:
RYY (τ ) = E ⎡⎣ Y ( t ) Y ( t + τ ) ⎤⎦
{
= E ⎡⎣ X ( t ) + X ( t − a ) ⎤⎦ ⎡⎣ X ( t + τ ) + X ( t + τ − a ) ⎤⎦ }
5
Unit.4. Correlation and spectral densities
= E ⎡⎣ X ( t ) X ( t + τ )⎤⎦ + E ⎡⎣ X ( t ) X ( t + τ − a ) ⎤⎦
+ E ⎡⎣ X ( t − a ) X ( t + τ ) ⎤⎦ + E ⎡⎣ X ( t − a ) X ( t + τ − a )⎤⎦
= Rxx (τ ) + Rxx (τ + a ) + Rxx (τ − a ) + Rxx (τ )
Ryy (τ ) = 2Rxx (τ ) + Rxx (τ + a ) + Rxx (τ − a )
Problem 17. a). If { X (t ) } and {Y (t )} are independent WSS Processes with zero means,
find the autocorrelation function of{Z (t )} , when (i) Z (t ) = a + bX (t ) + C Y (t ) ,
(ii) Z (t ) = aX (t )Y (t ) .
Solution:
Given E ⎡ ⎣ X (t )⎤ ⎦ = 0 E ⎡⎣ Y ( t )⎤⎦ = 0 -------- (1)
,
{X (t)} and {Y (t)} are independent
E { X (t )Y (t )} = E ⎣ ⎡ X (t )Y (t )⎦ ⎤ = 0 -------- (2)
(i). RZZ (τ ) = E ⎡⎣ Z ( t ) Z ( t + τ ) ⎤⎦
{
= E ⎡⎣ a + bX ( t ) + cY ( t ) ⎤⎦ ⎡⎣ a + bX ( t + τ ) + cY ( t + τ ) ⎤⎦
= E {a 2
+ ab X ( t + τ ) + acY ( t + τ ) + baX ( t ) + b 2 X ( t ) X ( t + τ )
+ bc X ( t ) Y ( t + τ ) + caY ( t ) + cbY ( t ) X ( t + τ ) + c 2 y ( t ) y ( t + τ ) }
( )
= E a 2 + abE ⎣⎡ X ( t + τ ) ⎤⎦ + acE ⎡⎣ Y ( t + τ )⎤⎦ + baE ⎡⎣ 2X ( t ) ⎤⎦ + b E ⎡⎣ X ( t ) X ( t + τ )⎤⎦
+ bc E ⎡⎣ X ( t ) y ( t + τ ) ⎤⎦ + caE ⎡⎣ Y ( t ) ⎤⎦ + cbE ⎡⎣ Y ( t ) X ( t + τ ) ⎤⎦ + c 2 E ⎡⎣ Y ( t ) Y ( t + τ ) ⎤⎦
= a2 + b2R XX (τ ) + c2 R (τ )
YY
RZZ (τ ) = E ⎡⎣ Z (t ) Z ( t + τ )⎤⎦
= E ⎡⎣ aX ( t ) Y ( t ) aX ( t + τ ) Y ( t + τ ) ⎤⎦
= E ⎡⎣ a 2 X ( t ) X ( t + τ ) Y ( t ) Y ( t + τ ) ⎤⎦
= a 2 E ⎡⎣ X ( t ) X ( t + τ ) ⎤⎦ E ⎡⎣ Y ( t ) Y ( t + τ ) ⎤⎦
R (τ ) = a 2 R (τ ) R (τ )
ZZ XX YY
6
Unit.4. Correlation and spectral densities
Var ( Y ) = E ( Y 2 ) − ⎡⎣ E ( Y ) ⎤⎦
2
( ) (
E Y 2 = E X 2 ( 5) )
But E ⎡ ⎣ X (t )⎦ ⎤ = RXX (0)
2
= 9 + 4e−0.2101
= 9 + 4 = 13
Thus Var ( Y ) = 13 − ( 3) = 13 − 9 = 4
2
Var ( Z ) = E ( Z 2 ) − ⎡⎣ E ( Z ) ⎤⎦
2
E ⎡⎣ Z 2 ⎤⎦ = E ⎡ ⎣X 2 ( 8 ) ⎤ ⎦ ⎡⎣ Z = X ( 8) ⎤⎦
= RXX ( 0)
= 9 + 4 = 13
Hence Var ( Z ) = 13 − 32 = 13 − 9 = 4( )
E [YZ ] = R (5,8) = 9 + 4e−0.25−8 ( R (t , t
1 2 ) = 9 + 4e−0.2 t −t
1 2
)
= 9 + 4e −0.6
variance of X (t ) .
Solution:
Given Rxx (τ ) = 9 + 2e τ
Mean of X (t ) is given by
Lt
X = E ⎡⎣ X ( t ) ⎤⎦ = Rxx (τ )
2 2
τ →∞
Lt
τ →∞ (
= 9 + 2e
) −τ
2
X =9
X =3
Also E ⎡ ⎣ X 2 (t )⎦ ⎤ = RXX (0) = 9 + 2e 0 = 9 + 2 = 11
Var { X ( t )} = E ⎡⎣ X 2 ( t ) ⎤ ⎦ − ⎡ ⎣E ( X ( t ) ) ⎤ ⎦
2
= 11− 32 = 11− 9 = 2
Mean of Y ( t ) = E ⎡⎣ Y ( t ) ⎤⎦
7
Unit.4. Correlation and spectral densities
⎡2 ⎤
= E ⎢ ∫ X (t ) dt ⎥
⎣0 ⎦
2
= ∫ E ⎡ ⎣ X (t )⎤ ⎦ dt
0
2
= ∫ 3dt = 3 ( t ) 0 = 6
2
∴ E ⎡⎣ Y ( t ) ⎤⎦ = 6
b). Find the mean and autocorrelation function of a semi random telegraph signal process.
Solution:
Semi random telegraph signal process.
If N (t ) represents the number of occurrences of a specified event in (0, t )
and X ( t ) = ( −1) , then { X ( t )} is called the semi random signal process and N (t ) is a
N (t )
= ∑
K =even K!
( λt )
2
⎡ ⎤
= e−λt ⎢1+ + ...⎥
⎣⎢ 2 ⎥⎦
P ⎡⎣ X ( t ) = 1⎤⎦ = e −λt coshλt
P ⎡⎣ X ( t ) = −1⎤⎦ = P ⎡⎣ N ( t ) is odd ⎤⎦
e−λt ( λt )
k
= ∑
K =odd K!
( λt )
3
⎡ ⎤
= e −λt ⎢ λ t + + ...⎥
⎢⎣ 3! ⎥⎦
= e sinhλ t
−λ t
Mean{X (t )} = ∑ K P ( X (t ) = K )
K =−1,1
8
Unit.4. Correlation and spectral densities
( λτ ) ( λτ )
n n
= ∑
n=even
eλt
∑ e− λ t
n! – n=odd n!
= e −λτ coshλτ − e λτ sinhλτ
= e −λτ [ coshλτ − sinhλτ ]
= e− λτ e− λτ
R (τ ) = e −2λτ
Problem 19. a). Find Given the power spectral density of a continuous process as
ω2 + 9
SXX (ω) = 4
ω + 5ω2 + 4
find the mean square value of the process.
Solution:
We know that mean square value of { X (t )}
1 ∞
= E X 2 (t ) =
{ } ∫ SXX (ω ) dω
2π −∞
1 ∞ ω2 + 9
2π ∫−∞ ω 4 + 5ω 2 + 4
= dω
( )
1 ∞ ω2 + 9
= .2∫ 4 dω
2π 0ω + 5ω 2 + 4
1∞ ω2 + 9
= ∫ 4 dω
π 0ω + ω 2 + 4ω2 + 4
1∞ ω2 + 9
= ∫ 2 2 dω
(
π 0ω ω +1 + 4 ω 2 +1 ) ( )
∞
ω2 + 9
{
i.e., E X (t ) = ∫
2 1
} dω
π 0 ω 2 + 4 ω 2 +1 ( )( )
let ω2 = u
∴ We have ω2 + 9 u+9
=
(
ω 2 + 4 ω 2 +1 )(
( u + 4 )( u +1) )
−4 + 9 −1+ 9
5 8
= −4 +1 + −1+ 4 = − + ….Partial fractions
u+4 u +1 3(u + 4) 3(u +1)
i.e., ω +9
2
5 8
=− +
(ω 2 + 4 ω 2 +1 )( 3 ω2 + 4 )
3 ω 2 +1 ( ) ( )
∴From (1),
1 ∞1⎡ −5 8 ⎤
{
E X 2 (t ) = ∫ ⎢ }+ ⎥ dω
π 0 (
3 ⎢⎣ ω 2 + 4 ) (ω 2
)
+1 ⎥⎦
9
Unit.4. Correlation and spectral densities
∞
1 ⎡ 1 ω ⎤
= ⎢ −5. tan
−1 + 8 tan −1 ω ⎥
3π ⎣ 2 2 ⎦0
1 ⎡ 5 ⎛π ⎞ ⎛π ⎞ ⎤
= − ⎜ +⎟ 8 ⎜ −⎟0
3π ⎢ ⎣ 2 ⎝ 2 ⎠ ⎝ 2 ⎠ ⎥⎦
1 π⎛ 5 ⎞ 1 ⎛ 11 ⎞
= . − +8 =
⎜ ⎟ 6⎜ 2 ⎟
3π 2 2
⎝ 11 ⎠ ⎝ ⎠
E {X 2 (t ) } = .
12
b). If the 2n random variables Ar and Br are uncorrelated with zero mean and
( ) ( )
E Ar2 = E B 2r = σ 2r. Find the mean and autocorrelation of the process
n
= ∑ ⎣⎡ E ( Ar ) cosωr t + E ( Br ) sinωr t ⎤⎦
r =1
E ⎡ ⎣ X (t )⎤ ⎦ = E ( Ar ) = E ( Br ) = 0
0
Autocorrelation function:
R (τ ) = E ⎡⎣ X ( t ) X ( t + τ ) ⎤⎦
⎧n n
= E ⎨∑∑ ( Ar cosωr t + Br sinωr t ) ( As cosωs ( t + τ ) + Bs sinωs ( t + τ ) )
⎩ r =1 s=1
Given 2n random variables Ar and Br are uncorrelated
E [ Ar As ], E [ Ar Bs ], E [Br As ], E [Br , Bs ] are all zero for r ≠ s
n
= ∑σ cos
r ω
2
r (t − t −τ )
r =1
n
= ∑σ cos
r ω
2
r ( −τ )
r =1
n
R (τ ) = ∑σ 2cos
r ω τ r
r =1
10
Unit.4. Correlation and spectral densities
Given R (τ ) = Ae−α τ
R (t1, t2 ) = Ae−α t1 −t2
E ⎡ ⎣ X 2 (t )⎦ ⎤ = RXX (0) = Ae−α 0 = A
∴ E ⎡⎣ X ( 8 ) − X ( 5) ⎤⎦ = E ⎡⎣ X 2 ( 8 ) ⎤⎦ + E ⎡⎣ X 2 ( 5 ) − 2E ⎡⎣ X ( 8 ) X ( 5 ) ⎤⎦ ⎤⎦
2
E ⎡⎣ X 2 ( 8) ⎤⎦ = E ⎡⎣ X 2 ( 5) ⎤⎦ = A
Also E ⎡⎣ X ( 8) X ( 5 ) ⎤⎦ = R ( 8, 5) = Ae −α 8 −5 = Ae −3α
Substituting (2), (3) in (1) we obtain
E ⎡⎣ X ( 8 ) − X ( 5 ) ⎤⎦ = A + A − 2 Ae −3α
2
= 2 A − 2 Ae−3α
(
= 2 A 1− e −3α )
.
b). Two random process { X (t )} and {Y (t )} are given by
X (t ) = A cos(ω t+ θ ) ,Y (t ) = Asin (ωt + θ ) where A & ω are constants and θ is a
uniform random variable over 0 to 2 π . Find the cross-correlation function.
Solution:
RXY (τ ) = E ⎡⎣ X ( t ) Y ( t + τ ) ⎤⎦
= E ⎡⎣ Acos ( ωt + θ ) Asin ( ω ( t + τ ) + θ ) ⎤⎦
= A2 E ⎡⎣ sin ( ωt + ωτ + θ ) cos ( ωt + θ )⎤⎦
1
θ is a uniform random variables fθ (θ ) = , 0 ≤ θ ≤ 2π
2π
2π
A2
∴ RXY (τ ) =
2π ∫ sin (ωt + ωτ + θ ) cos (ωt + θ ) dθ
= A∫ ⎜ ⎛ sin ( 2ωt + ωτ + 2θ ) + sin ( ωt ) ⎞
0
2 2π
2π ⎟ dθ
0 ⎝
2 ⎠
2π
= 4π − cos ( 2ωt 2+ ωτ + 2θ ) + θ sin (ωt )⎥ ⎤
A ⎡
2
⎢
⎣ ⎦0
2
A
=
4π
[0 + 2π sinωτ ]
A2
= sinωτ
2
Problem 21. a). Find the cross-correlation function of
W (t ) = A (t ) + B (t ) & Z (t ) = A (t ) − B (t ) where A (t ) and B (t ) are statistically
independent random variables with zero means and autocorrelation function
RAA τ = e− τ , − ∞ < τ < ∞ , R BB τ = 3e − τ – ∞ < τ < ∞ respectively.
Solution:
11
Unit.4. Correlation and spectral densities
Given E ⎡⎣ A ( t ) ⎤⎦ = 0, E ⎡⎣ B ( t ) ⎤⎦ = 0
A (t ) & B (t ) are independent
RAB (τ ) = E ⎡⎣ A ( t ) B ( t + τ ) ⎤⎦ = E ⎡⎣ A ( t ) ⎤⎦ E ⎡⎣ B ( t + τ ) ⎤⎦ = 0
Similarly RBA (τ ) = E ⎡⎣ B ( t ) A ( t + τ ) ⎤⎦ = E ⎡⎣ B ( t ) ⎤⎦ E ⎡⎣ A ( t + τ ) ⎤⎦ = 0
∴ RWZ (τ ) = E ⎡W
⎣ ( t ) Z ( t + τ ) ⎤⎦
{
= E ⎡⎣ A ( t ) + B ( t ) ⎤⎦ ⎡⎣ A ( t + τ ) − B ( t + τ ) ⎤⎦ }
= E ⎡⎣ A ( t ) A ( t + τ ) − A ( t ) B ( t + τ ) + B ( t ) A ( t + τ ) − B ( t ) B ( t + τ ) ⎤⎦
= E ⎡⎣ A ( t ) A ( t + τ ) ⎤⎦ − E ⎡⎣ A ( t ) B ( t + τ ) ⎤⎦ + E ⎡⎣ B ( t ) A ( t + τ ) ⎤⎦ − E ⎡⎣ B ( t ) B ( t + τ ) ⎤⎦
= RAA (τ ) − RAB (τ ) + RBA (τ ) − RBB (τ )
= RAA (τ ) − RBB (τ )
−τ
= e – 3e − τ
RWZ (τ ) = −2e − τ
b). The random processes { X (t )} and {Y (t )} defined by
X (t ) = A cos ωt + B sin ωt,Y (t ) = B cos ωt − A sin ωt where A & B are uncorrelated zero
mean random variables with same variance find its autocorrelation function.
Solution:
Given E ( A) = E ( B) = 0 & E A2 = E B 2 ( ) ( ) =σ 2
{
= E [ Acosω t + B sinωt ] ⎣⎡ B cosω ( t + τ ) − Asinω ( t + τ ) ⎤⎦ }
⎡ ABcosωtcosω ( t + τ ) − A cosωtsinω ( t + τ ) ⎤
2
= E⎢ ⎥
⎢⎣ +B 2 sinω tcosω ( t + τ ) − BAsinωtsinω ( t + τ ) ⎥⎦
( )
= E ( AB ) cosωtcosω ( t + τ ) − E A2 cosω tsinω ( t + τ )
( )
+ E B 2 sinωtcosω ( t + τ ) − E ( BA ) sinωtsinω ( t + τ )
= E ( B 2 ) sinωtcosω ( t + τ ) − E ( A2 ) cosωtsinω ( t + τ ) ( E ( AB ) = 0 )
= σ ⎡⎣ sinωtcosω ( t + τ ) − cosωtsinω ( t + τ ) ⎤⎦ ⎡⎣ E2 ( A
2
) = E2 (B ) = σ 2
⎤
⎦
= σ 2 sinω ( t − t −τ )
= σ 2 sinω ( −τ )
RXY (τ ) = −σ 2 sinωt ⎣⎡ sin ( −θ ) = −sinθ ⎤⎦
12
Unit.4. Correlation and spectral densities
Problem 22. a). Consider two random processes X (t ) = 3cos (ωt + θ ) and
⎛ π⎞
Y ( t ) = 2 cos ω t + θ − where θ is a random variable uniformly distributed in
⎜ 2⎟
⎝ ⎠
(θ , 2π ) . Prove that RXX ( 0 ) RYY ( 0 ) ≥ RXY (τ ) .
Solution:
Given X (t ) = 3cos (ωt + θ )
⎛ π⎞
Y ( t ) = 2cos ωt + θ −
⎜ 2⎟
⎝ ⎠
RXX (τ ) = E ⎡⎣ X ( t ) X ( t + τ ) ⎤⎦
= E ⎣⎡ 3 cos ( ωt + θ ) 3cos ( ωt + ωτ + θ ) ⎤⎦
9
= E ⎡⎣ cos ( 2ωt + ωτ + 2θ ) + cos ( −ωτ ) ⎤⎦
2
1
θ is uniformly distributed in (0, 2π ) , f (θ ) =
2π
9 2π 1
= ∫ ⎡⎣ cos ( 2ωt + ωτ + 2θ ) + cos ωτ ⎤⎦ dθ
2 0 2π
= 9 1 ⎢⎡ sin ( 2ωt + ωτ + 2θ )
2π
⎤
2 2π 2 + θ cosωτ ⎥
⎣ ⎦0
9 ⎡ ⎡ sin ( 2ω t + ωτ + 2θ ) ⎤ 2π ⎤
= [0 + 2πcosωt ] ⎢ ⎢ ⎥ = 0⎥
4π 2
⎣⎢ ⎣ ⎦0 ⎥⎦
9
RXX (τ ) = cosωτ
2
9
RXX ( 0 ) =
2
RYY (τ ) = E ⎣⎡ Y ( t ) Y ( t + τ ) ⎦⎤
⎡ ⎛ π⎞ ⎛ π ⎞⎤
=
E ⎢ 2cos ⎜⎝ω t + θ − ⎟ 2cos ω
⎜ ⎝t + ωτ + θ − ⎟
⎣ 2 ⎠ 2 ⎥ ⎠⎦
4
= E ⎡⎣ cos ( 2ωt + ωτ + 2θ − π ) + cosωτ ⎤⎦
2
2π
1
= 2 ∫ ⎡⎣ cos ( 2ωτ + ωτ + 2θ − π ) + cosωτ ⎤⎦ dθ
0
2π
1 ⎡ sin ( 2ω t + ωτ + 2θ − π ) ⎤
2π
= ⎢ + θ cosωτ ⎥
π 2
⎣ ⎦0
= 2cosωτ
13
Unit.4. Correlation and spectral densities
π⎞
Similarly, R (τ ) = 3cos ⎛ωτ −
XY ⎜ 2⎟
⎝ ⎠
RXY (τ ) max
=3
By property (2) i.e.,RXX ( 0 ) RYY ( 0 ) > RXY (τ )
⎛ π⎞
∴ 9 ≥ 3cos ωτ − , ∀τ
.2 ⎜ ⎟
2 2
⎝ ⎠
In this care RXY (τ ) takes on its maximum possible value of 3 at
π n2π
τ= + , ∀n
2ω ω
Since it is periodic
R (0) R (0) = R (τ )
2
XX YY XY max
b). State and prove Wiener – Khinchine Theorem
Solution:
Statement:
If X T (ω ) is the Fourier transform of the truncated random process defined as
⎧⎪ X (t ), − T ≤ t ≤ T
XT (t ) = ⎨
⎪⎩0 , otherwise
Where { X (t )} is a real WSS process with power spectral function SXX (ω ) , then
SXX (ω ) = lim ⎡ 1 E X (ω )
2 ⎤
{ }
T → ∞ ⎢ ⎣ 2T T ⎥⎦
Proof:
∞
Given XT (ω ) =
∫ X (t ) e dt
−iωt
T
−∞
T
= ∫ X (t ) e−iωt dt
−T
T T
= ∫ X ( t 1) e −iωt1 dt 1 ∫ X (t )e 2
−iωt 2
dt 2
−T −T
= X (t )X (t e −iω (t1 −t 2 ) dt dt
T T
)
∫∫ 1 2 1 2
{X }=
T T
(ω ) E ⎡ X (t ) X (t
−T −T2
∴E
) ⎤e−iω (t −t ) dt dt
1 2
T ∫∫
−T −T
⎣ 1 2 ⎦ 1 2
= R (t − t
T T
)e −iω ( t1 −t 2 )
dt dt
∫∫ 1 2 1 2
−T −T
14
Unit.4. Correlation and spectral densities
T T
T
A( T ,T ) B
T T
Q
t1 t 2 = d
S
P
D R C(T , T )
T
The double integral (1) is evaluated over the area of the square ABCD bounded by the
t1 = −T ,T & t2 = −T ,T as shown in the figure
We divide the area of the square into a number of strips like PQRS, where PQ is given by
t1 − t2 = τ and RS is given by t1 − t2 = τ + dτ .
When PQRS is at A, t1 − t2 = −T −T = −2T and t1 − t2 = 2T and at C
∴τ varies from 2T to 2T such that the area ABCD is covered.
Now dt1dt2 = elemental area of the t1, t2 plane
=Area of the strip PQRS (2)
At P , t2 = T , t1 = τ + t2 = τ − T At
Q , t1 = T , t2 = t1 =T PC =
CQ = 2T −τ ,τ > 0
PC = CQ = 2T + τ ,τ < 0
RC = SC = 2T −τ − dτ , τ > 0
When τ > 0 ,
Area of PQRS = ΔPCQ − ΔRCS
1 1
= ( 2T − τ ) − ( 2T − τ − 2τ )
2 2
2 2
= ( 2T − τ ) dτ Omitting ( dτ )
2 -------------------------
(3)
From (2) & (3)
15
Unit.4. Correlation and spectral densities
{X }
2T
(ω ) ∫ 0 (τ ) ( 2T − τ ) dτ
2
E T =
−2T
{ } τ ⎞
2T
1 E X (ω ) ⎛
T
2
= ∫ 0 (τ ) ⎜1− ⎟ dτ
2T ⎝ 2T ⎠
}
−2
( w) = lim 2T 0 (τ ) dτ −
T 2
∴
lim 1
E {X ∫
lim τ dτ
T → ∞ 2T T →∞ T → ∞ ∫ 2T
T
−2T
∞
= ∫ 0 (τ ) dτ
−∞
∞
= ∫ R (τ ) e −iωτ dτ
−∞
= S (ω )
This theorem provides an alternative method for finding S (ω ) for a WSS Process.
Problem 23. a). Define the spectral density S (ω ) of a real valued stochastic
processes{X (t ) : t ≥ 0}, obtain the spectral density of {Y (t ) : t ≥ 0},Y (t ) = α X (t ) when
1
α is increment of X (t ) such that P (α = 1) = P (α = −1) = .
2
Solution:
If X (t )} is a stationary process {either in the strict serve or wide sense with
autocorrelation function R τ , then the Fourier transform of R τ is called the Power
spectral density function of X (t )} denoted by S (w )
∞
Thus S ( ω ) = ∫ R (τ ) e −iωτ dτ
−∞
1
Given P{α = 1} = P {α = −1} =
2
1 1
∴ E (α ) = 1. + (−1). = 0
2 2
2 1 2 1
∴ E α( =) (1) . + ( −1) . = 1
2
2 2
E ⎡⎣ Y ( t ) ⎤⎦ = E ⎡⎣ α X ( t ) ⎤⎦
= E ( α ) E ⎡⎣ X ( t ) ⎤⎦ ⎡⎣ α X ( t ) areindependent ⎤⎦
E ⎡⎣ Y ( t ) ⎤⎦ = 0 ⎡⎣ E ( α ) = 0⎤⎦
RYY (τ ) = E ⎡⎣ Y ( t ) Y ( t + τ ) ⎤⎦
= E ⎡⎣ α X ( t ) α X ( t + τ ) ⎤⎦
16
Unit.4. Correlation and spectral densities
= E ⎡⎣ α 2 X ( t ) X ( t + τ ) ⎤⎦
= E ⎡⎣α 2 ⎤ ⎦E ⎡⎣ X ( t ) X ( t + τ ) ⎤⎦
= 1 E ⎡⎣ X ( t ) X ( t + τ ) ⎤⎦
= (1) P ⎡⎣ X ( t )( t + τ ) = 1⎤⎦ + (−1) P ⎡⎣ X ( t )( t + τ ) = −1⎤⎦
e −λT ( λT ) e −λT ( λT )
n
= (1) ∑ n! + (−1) ∑ n!
n=even n=odd
S ( ω ) = ∫ R (τ )e −iωτ dτ
−∞
∞
= ∫ e−2 λ τ e−i ωτ dτ
−∞
0 ∞
∫ e e dτ + ∫ e 0 e dτ
2 λτ −iωτ −2 λτ −iωτ
= −∞
0 ∞
( 2 λ −iω )τ −( 2 λ +i ω )τ
= ∫e dτ + ∫ e dτ
−∞ 0
0 ∞
⎡ ( 2λ −iω )τ ⎤ ⎡ e−( 2 λ +iω )τ ⎤
=⎢e ⎥ +⎢ ⎥
2λ − iω
⎣ ⎦ −∞ ⎣ − ( 2λ + iω ) ⎦ 0
1 1
= +
2λ − iω 2λ + iω
2λ − iω + 2λ − iω
=
4λ 2 − ω 2
4λ
= 2
4λ + ω 2
b). Show that (i) S (ω ) ≥ 0 & (ii) S (ω ) = S (−ω ) where S (ω ) is the spectral density of a
real valued stochastic process.
Solution:
(i) S (ω ) ≥ 0
Proof:
If possible let S (ω ) < 0 at ω = ω0
ε ε
i.e., let S (ω ) < 0 in ω0 – < ω < ω 0 + , where ε is very small
2 2
Let us assume that the system function of the convolution type linear system is
17
Unit.4. Correlation and spectral densities
⎧ ε ε
⎪1 , ω0 − < ω < ω0 +
H ( ω ) =⎨ 2 2
⎪⎩0 , elsewhere
Now SYY (ω ) = H (ω ) 2S (ω )
⎧ XX
⎪S ( ) 0 – ε < ω < ω + ε
ω , ω
= ⎨ XX 0
2 2
⎪⎩0 , elsewhere
E ⎡⎣ Y 2 ( t ) ⎤⎦ = R YY (0)
∞
1
=
2π ∫ SYY (ω )dω
−∞
ε
ω 0+
2
= 1 SXX (ω )dω
2π
ω−
∫ ε
0
2
ε
= S (ω )
2π XX 0
[ SXX (ω ) Considered a constant S XX ( ω0 ) band is
narrow] E ⎡⎣ Y 2 ( t ) ≥ 0, S ( ω ) ≥ 0 which is contrary to our initial assumption.
xx 0
Consider S ( −ω ) = ∫ R (τ )eiωτ dτ
−∞
Let τ = −u then dτ = −du and u varies from and
−∞
= ∫ R (−u )e−iωudu
−∞
∞
= S (ω )
Hence S (ω ) is even function of ω
Problem 24. a). An autocorrelation function R ( τ ) of {X (t ); −∞ < t < ∞} in given by
ce −α τ , c > 0, α > 0 . Obtain the spectral density of R ( τ ).
Solution:
Given R (τ ) = ce−α τ
18
Unit.4. Correlation and spectral densities
By definition
∞
S ( ω ) = ∫ R (τ ) e −iωτ dτ
−∞
∞
= ∫ ce−α τ e −iωτ dτ
−∞
∞ ∞
−α ( −τ )
= ∫ ce e −iωτ
dτ + ∫ ceα (τ ) e −iωτ dτ
−∞ 0
0 ∞ ⎤
⎡ (α −iω )τ −(α −iω )τ
= c⎢ ∫ e dτ + ∫ e dτ ⎥
⎣ −∞ 0 ⎦
= c ⎨⎢ ⎡ e( α −iω )τ ⎥⎤ + ⎢⎡ e−(α +iω )τ ⎥ ⎤ ⎬∞ ⎫⎪
0
α − iω −(α + iω )
⎩⎪ ⎣ ⎦ −∞ ⎣ ⎦ 0 ⎪⎭
⎡ 1 1 ⎤
=c +
⎢ ⎣ α − iω α + iω ⎥ ⎦
⎡ α + iω + α − iω ⎤
=c
⎣ ⎢ α −ω ⎥⎦
2 2
2αc
S (ω ) =
α 2 + ω2
b). Given that a process X (t )} has the autocorrelation function
RXX (τ ) = Ae−α τ cos(ω τ )0 where A > 0 , > 0 and ω0 are real constants, find the power
spectrum of X (t).
Solution:
∞
By definition S ( ω ) = ∫ R (τ ) e − iωτ dτ
−∞
∞
∫ −∞
0
= A e −α τ cosω τ cosωτ + A
∫ e cosω τ ( −sinωτ ) dτ
−α τ
∫ ∞
−∞
0
−∞
∞
0
∫ −ατ∞ 0
= 2 A e cosω τ cosωτ dτ
⎡ cos (ω + ω )τ + cos (ω − ω )τ ⎤
0
∞
= 2A ∫ e−ατ ⎢
⎥ dτ
0 0
−∞ ⎣ 2 ⎦
∞ – a
Using ∫e
0
ax
cosbxdx = a2 + b2
19
Unit.4. Correlation and spectral densities
⎡ α α ⎤
S (ω ) = A ⎢ 2⎥
⎢ α + (ω + ω ) + α + (ω − ω ) ⎥
2 2 2
⎣ 0 0 ⎦
= Aα ⎡⎢ α 2 + (ω 1 + ω ) + α 2 + (ω 1− ω 2 ⎤⎥
2
⎣⎢ 0 0) ⎥⎦
Problem 25. a). Find the power spectral density of the random binary transmission
⎪⎧1− τ
process whose autocorrelation function is R (τ ) = for τ ≤ 1 .
⎨
⎪0
⎩ elsewhere
Solution:
∞
By definition S ( ω ) = ∫ R (τ )e −iωτ dτ
−∞
1
= ∫ ⎡⎣ 1− τ ⎤⎦ e − iωτ dτ
−1
0 1
20
Unit.4. Correlation and spectral densities
−α τ
b). Show that the power spectrum of the autocorrelation function e ⎡⎣ 1+ α τ ⎤⎦ is
4α 3
.
(α + w )
2
2 2
Solution:
∞
By definition, S ( ω ) = ∫ R (τ )e −iωτ dτ
−∞
∞
= ∫ e −α τ ⎡⎣ 1+ α τ ⎤⎦ e −iωτ dτ
−∞
0 ∞
= ⎢ (1− ατ ) − ( −α ) 2 ⎥
+ ⎢ (1+ ατ ) − (α ) 2 ⎥
α − iω ( α − iω ) − (α + iω ) ( α + iω )
⎢⎣ ⎥⎦ −∞ ⎢⎣ ⎥⎦ 0
⎡ 1 α ⎥ ⎤ ⎡ ⎢ 1 α ⎥ ⎤
= ⎢ α − iω + (α − iω ) 2 + 0 + α + iω + (α + iω ) 2
⎣⎢ ⎥⎦ ⎢⎣ ⎥⎦
⎡ 1 ⎡ ⎤
= + 1 ⎤ +α 1
+
1
α − iω α + iω ⎥ ⎦ ⎢⎣ (α − iω ) (α + iω )2 ⎥ ⎦
2
⎡ α + iω 2 + α − iω 2 ⎤
2α ( ) ( )
= 2 + α ⎢ ⎥⎥
(α + ω )
2
2 2
α + ω2
⎢
⎣ ⎦
2α
= 2 2 +
2α α 2 − ω2 ( )
α +ω ( )
2
α 2 + ω2
2α (α + ω ) + 2α (α − ω )
2 2 2 2
=
(α + ω ) 2 2 2
2α (α + ω + α − ω )
2 2 2 2
=
(α + ω ) 2 2 2
4α 3
= (α 2 − ω 2 )2
Problem 26. a). Find the spectral density function whose autocorrelation function is
⎧ τ
⎪1− ;τ ≤T
given by R (τ ) = ⎨ τ
⎪0 ; elsewhere
⎩
Solution:
21
Unit.4. Correlation and spectral densities
S ( ω ) = ∫ R (τ )e −iωτ dτ
−∞
T
⎡ 1− τ ⎤ −iωτ
= ∫⎢ ⎥ e dτ
−T ⎣
τ ⎦
T
⎛ τ ⎞
= ∫ ⎜1− ⎟ [cosωτ − isinωτ dτ]
−T ⎝
T ⎠
T
⎛ τ ⎞
= ⎜1− ⎟ cosωτ dτ
∫ T⎠
−T ⎝
T
⎛ τ ⎞
= 2 ⎜1− ⎟ cosωτ dτ
∫0 ⎝ T ⎠
T
⎛ τ⎞
= 2 ⎜1− ⎟ cosωτ dτ
∫0 ⎝ T ⎠ T
⎡⎛ τ ⎞ sinωτ ⎛ −1 ⎞⎛ −cosωτ ⎞⎤
= 2 ⎢ ⎜1− ⎟ – ⎜ τ ⎟⎜
T ω ω 2 ⎟⎥
⎣⎝ ⎠ ⎝ ⎠⎝ ⎠⎦ 0
⎡ −cosωT 1 ⎤
=2 +
⎣⎢ ωT ω2T ⎥ ⎦
2
= [
2 1− cosωT
]
ω222T 2 ⎛ ωT ⎞
= ω T 2 sin ⎜ ⎟
2
⎝ ⎠
⎛ ωT ⎞
4sin2
⎜2 ⎟
S (ω ) = ⎝ ⎠
ω2T
b). Find the power spectral density function whose autocorrelation function is given by
A2
R (τ ) = cos (ω0τ )
2
Solution:
∞
S ( ω ) = ∫ R (τ ) e −iωτ dτ
−∞
∞ 2
A
=
∫ cos ( ω 0τ ) e −iωτ dτ
−∞
2
∞
A2
=
2 −∞
∫ cos (ω τ ) [cosωτ − isinωτ ] dτ
0
∞ ∞
A ⎡ 2
⎤
⎢ ⎡ cos ( ω − ω0 )τ + cos ( ω + ω0 ) τ ⎤⎦ dτ − i ∫ ⎡⎣ sin ( ω + ω0 ) τ + sin ( ω − ω0 ) ⎤⎦ dτ ⎥
4 ∫⎣
=
⎣ −∞ −∞ ⎦
22
Unit.4. Correlation and spectral densities
A2 ⎡ ∞ ∞
4 ⎣ −∞ −∞ ⎦
By definition of Dirac delta function
1 ∞ iωτ
S (ω ) = e dτ
2π −∞∫
∞
2πδ ( ω ) = ∫ e −iωτ dτ
−∞
(1) reduces to
A2
S (ω ) = ⎡⎣ 2πδ ( ω + ω0 ) + 2πδ ( ω − ω0 ) ⎤⎦
4
π A2
S (ω ) = ⎡ δ (ω + ω0 ) + δ (ω − ω0 )⎤ ⎦
4 ⎣
Problem⎧ b27. a). If the power spectral density of a WSS process is given by
(a − ω ); ω ≤ a
⎪
S (ω ) = ⎨ a find its autocorrelation function of the process.
⎪0 ;ω >a
⎩
Solution:
∞
1
R (τ ) = S ( ω ) eiωτ dτ
2π ∫ −∞
b ( a − ω ) eiωτ dτ
a
1
= ∫
2π a −a
a
1 b
= ∫ ( a − ω )(cosωτ + isinωτ d)ω
2π a−a a
2 b
( a − ω ) cosωτ d ω
2π ∫ 0a
=
sinωτ cosωτ ⎤
a
b ⎡
= ⎢ (a −ω) – ⎥
πa ⎣ τ τ
2
⎦0
b ⎡ cosaτ 1 ⎤
= − +
⎢
πa ⎣ τ 2
τ2 ⎥⎦
= [
b 1− cosaτ
]
π aτ 2 2 ⎛ aτ ⎞
b 2sin
= π aτ 2 ⎜2⎟
⎝ ⎠
23
Unit.4. Correlation and spectral densities
b).The power spectral density function of a zero mean WSS process { X (t )} is given
⎧⎪1 ; ω < ω0 ⎞
by S ( ω ) = . Find R (τ ) and show also that X (t ) & X ⎛t + π are
⎨ ⎜ ⎟
⎪0
⎩ ; elsewhere ⎝ ω0⎠
uncorrelated
Solution:
1∞
R (τ ) = S ( ω ) e iωτ d ω
2π −∞ ∫
ω0
1
eiωτ d ω
2π −∫ω
=
0 ω
= 1 ⎢ ⎡ e ωτ ⎥ ⎤
0
i
2π iτ
⎣ 0
⎦ −ω
πτ1 e −2 e−iω0τ ⎤ ⎥
⎢ ⎡ iω0τ
=
⎣ ⎦
sinω0τ
=
πτ ⎞
To show that X (t ) X ⎛t + π are uncorrelated we have to show that the auto
& ⎜ ⎟
⎝ ω0 ⎠
covariance is zero.
⎞⎤
⎡ ⎛ π =0
i.e., C X (t ) X t +
⎢⎣ ⎜⎝
ω 0⎟⎥⎠⎦
Consider,
⎡ ⎛ π ⎞⎤ ⎛π ⎞ ⎡ ⎛ π ⎞⎤
C ⎢ X (t ) X ⎜ t + ⎟⎥ = R XX ⎜ − E ⎡ ⎣ X (t )⎦ ⎤ E ⎢ X ⎜ t + ⎟⎥
ω ω ⎟ ω
⎣ ⎝ 0 ⎠⎦ ⎝ 0 ⎠ ⎣ ⎝ 0 ⎠⎦
= RXX ⎜ π − 0
⎛ ⎞
ω ⎟
⎝0⎠
⎡ ⎛ π ⎞⎤ sinπ
C ⎢ X (t ) X ⎜ t + ⎟⎥ = =0
ω πτ
⎣ ⎝ 0 ⎠ ⎦
Hence X ( t ) X ⎛t + π ⎞ are uncorrelated.
& ⎜ ⎟
⎝ ω0⎠
Problem 28. a). The power spectrum of a WSS process { X (t )} is given
1
by S (ω ) = . Find the autocorrelation function and average power of the process.
(1+ ω ) 2 2
Solution:
1 ∞
R (τ ) = S ( ω ) eiωτ d ω
∫
2π −∞
24
Unit.4. Correlation and spectral densities
∞ 1
1 eiωτ d ω ------- (1)
= ∫
2π −∞ (1+ ω 2 )
2
= ⎢
z →i⎢ ⎣ ( z + i )4 ⎦⎥
Lt ⎡ ( z + i ) i a eiaz − 2eiaz ⎤
= ⎢
z →i⎢ ⎣ ( z + i )3 ⎦⎥
e−a (−2a − 2) 2e−a (1+ a ) e−a (1+ a )
= = =
8 ( −i ) 8i 4i
n
Res
By Cauchy residue theorem, f (z)dz = 2 i f (z) and taking limits R
c
z=a
i=1 i
25
Unit.4. Correlation and spectral densities
2π jτ 2π W jτ j 2τ 2 ⎥
⎣ ⎦ −W ⎣ ⎦ −W
jb ⎡ We τ – − jWτ
e jWτ We − jWτ e 2 ⎥⎤
jWτ − jWτ jW
a ⎡e −e ⎤⎥
= 2π ⎢ jτ + 2π W ⎢ jτ + τ 2 + jτ τ
⎣ ⎦ ⎣ ⎦
asinπτ(Wτ ) bW ⎢ ⎡ e − e2
jW τ − jW τ ⎤ ⎥ ⎢
jb 2 ⎡ e − e2 ⎤
jW τ − jW τ ⎥
= + π Wτ + π Wτ
⎣ ⎦ ⎣ ⎦
asin (Wτ ) b cos (Wτ ) bsin (W τ )
= + −
1 πτ⎡ ⎛ b ⎞ πτ π Wτ 2 ⎤
= 2 ⎢ ⎜ aτ − ⎟sin (Wτ ) + bτ cos (Wτ ) ⎥
πτ W
⎣⎝ ⎠ ⎦
Problem 29. a). The cross-power spectrum of real Random process { X (t )} and {Y (t )}
is given by S (ω ) = a + jbω; ω < 1 . Find the cross correlation function.
XY ⎨
⎪0
⎩ ; elsewhere
Solution:
1∞
RXY (τ ) = ∫ SXY ( ω ) e jωτ d ω
2π −∞
1
1
=
∫ ( a + jbω ) e jωτ d ω
2π −1
1
= a ⎢ ⎡ e ωτ ⎥ ⎤ + b ⎢ ⎡ ωe ωτ – e jωτ ⎤ 1
j j
2π jτ 2π jτ j 2τ 2 ⎥
⎣ ⎦ −1 ⎣ ⎦ −1
a ⎢ ⎡ e jτ − e− jτ ⎤ ⎥ jb ⎢ ⎡ e jτ jτ − jτ – − jτ
= 2π + 2π jτ + eτ 2 + ejτ e 2 ⎥⎤
jτ τ
⎣ ⎦ j ⎣ j ⎦
a jb ⎡ e τ − e− τ ⎤ + jb ⎡ e jτ − e − jτ ⎤
=
πτ
[ sinτ ] + 2π ⎢ jτ
⎥
2π
⎢
τ2 ⎥
⎣ ⎦ ⎣ ⎦
asinτ bcosτ bsinτ
= + −
πτ πτ πτ 2
26
Unit.4. Correlation and spectral densities
1
= ⎡ ( aτ − b ) sinτ + bτ cosτ ⎤
πτ2⎣ ⎦
b). Find the power spectral density function of a WSS process with autocorrelation
function. R (τ ) = e −ατ .
2
Solution:
∞
∫ e e dτ
− aτ −iωτ
= −∞
2
∞
(
− aτ 2 +iωτ )
= ∫e dτ
−∞
∞ ⎛ iωτ ⎞
−a τ 2 + ⎟
⎝⎜ ⎠
= ∫e a
dτ
−∞
∞ − a ⎛τ 2 + i ω ⎞
τ
⎜ 2. . ⎠⎟
∫e
⎝
= a
dτ
−∞
⎡⎛ 2 2iω i 2ω 2 ⎞ i2ω 2 ⎤
∞ −
a ⎢ ⎜τ + t+ − ⎥
⎢ ⎣ ⎝ 2a 2 2 a 2 ⎟ ⎠ 2 2 a 2 ⎥⎦
dτ
= ∫e
−∞
⎡ ⎛ iω ⎞2 ω2 ⎤
∞
−a ⎢ ⎜τ + ⎟ + 4a2 ⎥
⎢ ⎣ ⎝ 2a ⎠
dτ
∫e
⎦
=
−∞
⎡ ⎛ iω ⎞2 ⎤ ω2
∞
−a ⎢ ⎜τ + ⎟ ⎥ −
= ∫e
−∞
⎢ ⎣ ⎝ 2a ⎠ ⎥ ⎦ 4a 2 .e dτ
2
ω2 ∞ ⎛ iω ⎞
− −a ⎜τ + ⎟
∴ SXX (ω ) = e 2a
dτ
∫e
⎝ ⎠
4a
−∞
iω ⎞
2
⎛ ⎛ iω ⎞
Now let, a τ + ⎟ = u2 i.e. u = a τ + ⎟
⎜ ⎜
⎝ 2a ⎠ ⎝ 2a ⎠
∴ du = a dτ
and as, τ → −∞, u → −∞ and as τ → ∞, u → ∞
ω2 ∞
(ω ) = e
−
e−u du
2
∴ SXX
ω 2
4a
∫−∞ a
−
4a ∞
e 2
a .2∫ e du
= −u
2
.... e −u is even
0
−ω 2
∞
2e 4a π 2
dx = x
∫e
−X
= ... Standard Result :
a 2 0 2
27
Unit.4. Correlation and spectral densities
π −ω
2
∴ SXX (ω ) =
ae
4a
Problem 30. a). The autocorrelation function of the random telegraph signal process is
given by R (τ ) = a 2e −2γ τ . Determine the power density spectrum of the random telegraph
signal.
Solution:
∞ ∞
= a 2 ∫ e −2 γ τ e−iωτ dτ + a ∫ e2 −2γ τ
e −iωτ dτ
−∞ 0
0 ∞
= a 2 ∫ e( 2 γ −iω )τ dτ + a 2 ∫ e − ( 2 γ +iω )τ
dτ
−∞ 0
0 ∞
⎡ e( 2γ −iω )τ ⎤ ⎡ e−( 2 γ −iω )τ ⎤
=a ⎢2
+a ⎢
2
(⎣ 2γ − iω ) ⎦ −∞ ⎣ − ( 2γ − iω ) ⎦ 0
⎥ 2
⎥ a2
=
a
( 2γ − iω )
(
e0 – e−∞ − ) e −∞ – e0 ( )
( 2γ + iω )
a2 a2
=
2γ − iω
( )
1− 0 − (0 −1)
( 2γ + iω )
⎡ 1 1 ⎤ ⎡ ⎢2γ + iω + 2γ − iω ⎤ ⎥
= a 2 ⎢ 2γ − iω + 2γ + iω ⎥ = a 2 ( 2γ ) + ω 2
2
⎣ ⎦ ⎢⎣ ⎥⎦
2⎡ 4γ ⎤ 4a 2γ
i.e., SXX (ω ) = a ⎢ 2 2 ⎥
= 2
⎣ 4γ + ω ⎦ 4γ + ω
2
S (ω ) = ∫ R (τ ) e
XX
−iωτ
dτ
−∞
∞ ∞ 0 ∞
= ∫ ⎡ 1+
⎣ e ⎤ e⎦ dτ
−α τ −iωτ
∫ e dτ + ∫ e−∞e dτ + ∫ e e0 dτ
−iωτ ατ −iωτ − ατ −iωτ
−∞
= −∞
1 1
= δ (ω ) + +
α − iω α + iω
2α
= δ (ω ) +
α 2 + ω2
*******
28
Unit.5. Linear System with Random Inputs
PART - A
sin2 aω
⇒ SYY (ω ) = SXX (ω ) .
a2 ω 2
Problem 2. Give an example of cross-spectral density.
Solution:
The cross-spectral density of two processes X (t ) and Y (t ) is given by
⎧⎪ p + iqω, if ω < 1
S
XY
(ω ) = ⎨
⎪⎩0, otherwise
⎧ A, 0 ≤ t ≤ 1
Problem 3. If a random process X (t ) is defined as X (t ) = ⎨ , where A is a
⎩0, otherwise
random variable uniformly distributed from −θ to θ . Prove that autocorrelation function
θ2
of X (t ) is .
3
Solution:
RXX ( t, t + τ ) = E ⎣⎡ X ( t ) .X ( t + τ ) ⎦⎤
⎡⎣ X ( t ) is cons tan t ⎤ ⎦
= E ⎡⎣ A2 ⎤⎦
But A is uniform in ( −θ ,θ )
1
∴ f (θ ) = , −θ < a < θ
2θ
θ
∴RXX ( t, t + τ ) = ∫ a2 f ( a ) da
−θ θ
θ 1 1 ⎡ a3 ⎤
=
∫
−θ
a2 .
2θ
dθ =
2θ ⎢⎣ 3 ⎥⎦ −θ
1
Unit.5. Linear System with Random Inputs
1 ⎡ 3 1 θ2
θ − ( −θ ) ⎤ = θ3 =
3
=
.2
6θ ⎣ ⎦ 6θ 3
1
Problem 4. Check whether is a valid autocorrelation function of a random
1+ 9τ 2
process.
1
Solution: Given R (τ ) =
1+ 9τ 2
1 1
∴ R (− τ ) = = = R (τ )
( )
1+ 9 −τ 2 1+ 9τ 2
∴ R (τ ) is an even function. So it can be the autocorrelation function of a random
process.
Problem 5. Find the mean square value of the process X(t) whose power density spectrum
4
is .
4 + ω2
Solution:
4
Given SXX (ω ) =
4 +ω2
1 ∞
Then RXX (τ ) = ∫ SXX (ω ) eiτω dω
2π −∞
Mean square value of the process is E ⎡ ⎣ X 2 (t )⎦ ⎤ = RXX (0)
∞
1
2π −∞∫ XX ( )
= S ω dω
1 ∞ 4
2π ∫−∞ 4 + ω 2
= dω
= 4∫∞ 12 dω ⎡ 1
⎢ 4 + ω 2 is even
⎤
π 4+ω ⎥
∞ ⎣ ⎦
ω⎤
0
4⎡ 1 2
= . ⎢ tan −1 = (
tan −1 ∞ − tan −1 0 )
π 2⎣ ⎥
2⎦0 π
2 π
= . =1
π2 ⎧1
; 0≤t ≤T
Problem 6. A Circuit has an impulse response given by h ( t ) = ⎪⎨ T find the
⎪⎩0 ; elsewhere
relation between the power spectral density functions of the input and output processes.
Solution:
T
H (ω ) = ∫ h (t ) e−iωtdt
0
2
Unit.5. Linear System with Random Inputs
T
1
= e−iω tdt
∫T
0
T
1 ⎡ e−iωt ⎤
= ⎢ ⎥
T ⎣ −iω ⎦ 0
1 ⎢⎡ −e −iωTiω+1⎤ ⎥
=T
⎣ ⎦
= (1− e )
−i ωT
Tiω
SYY (ω ) = H (ω ) 2 S (ω )
XX
(1− e ) −iωT 2
= SXX (ω )
ω 2T 2
Problem 7. Describe a linear system.
Solution:
Given two stochastic process { X 1 ( t )} and{X 2 (t )}, we say that L is a linear
transformation if
L ⎡⎣ a1 X 1 ( t ) + a2 X 2 ( t ) ⎤⎦ = a1 L ⎡⎣ X1 ( t ) ⎤⎦ + a2 L ⎡⎣ X 2 ( t
)⎤⎦
Problem 8. Given an example of a linear system.
Solution: x (t ) .
Consider the system f with output tx (t ) for an input signal
i.e . y (t ) = f ⎣ ⎡ X (t )⎤ ⎦ = t x (t )
Then the system is linear.
For any two inputs x1 (t ) , x2 (t ) the outputs are tx1 (t ) and tx2 (t ) Now
f ⎡⎣ a1 x1 ( t ) + a2 x2 ( t ) ⎤⎦ = t ⎡⎣ a1 x1 ( t ) + a2 x2 ( t ) ⎤⎦
= a1tx1 (t ) + a2tx2 (t )
= a1 f ( x1 ( t ) ) + a2 f ( x2 ( t ) )
∴the system is linear.
Problem 9. Define a system, when it is called a linear system?
Solution:
Mathematically, a system is a functional relation between input x (t ) and output y (t ) .
Symbolically, y (t ) = f ⎣ ⎡ x (t )⎤ ⎦ , −∞ < t < ∞..
The system is said to be linear if for any two inputs x1 (t ) and x2 (t ) and constants
a1, a2 , f ⎡⎣ a1 x1 ( t ) + a2 x2 ( t ) ⎤⎦ = a1 f ⎡⎣ x1 ( t ) ⎤⎦ + a2 f ⎡⎣ x2 ( t ) ⎤⎦ .
Problem 10. State the properties of a linear system.
Solution:
Let X1 (t ) and X 2 (t ) be any two processes and a and b be two constants.
3
Unit.5. Linear System with Random Inputs
then Y (t ) = h (t ) * X (t ) = ∫ h (u ) X (t − u ) du
−∞
Problem 13. Write a note on noise in communication system.
Solution:
The term noise is used to designate unwanted signals that tend to disturb the transmission
and processing of signal in communication systems and over which we have incomplete
control.
Noise
⎪⎩0 , otherwise
Problem 15. Define (a) Thermal Noise (b) White Noise.
Solution:
4
Unit.5. Linear System with Random Inputs
(a) Thermal Noise: This noise is due to the random motion of free electrons in a
conducting medium such as a resistor.
(or)
Thermal noise is the name given to the electrical noise arising from the random
motion of electrons in a conductor.
(b) White Noise(or) Gaussian Noise: The noise analysis of communication
systems is based on an idealized form of noise called White Noise.
PART-B
Problem 16. A random process X (t ) is the input to a linear system whose impulse
response is h (t ) = 2e−t , t ≥ 0 . If the autocorrelation function of the process is
RXX (τ ) = e −2 τ , find the power spectral density of the output process Y (t ) .
Solution:
Given X (t ) is the input process to the linear system with impulse response
h (t ) = 2e−t , t ≥ 0
So the transfer function of the linear system is its Fourier transform
∞
H (ω ) = ∫ h (t ) e−iωtdt
−∞
∞
⎡⎣ 2e − t , t ≥ 0⎤⎦
= ∫−∞2e−te−iωt dt
∞
= 2∫ e−(1+iω )t dt
0
∞
⎡ e−(1+iω )t ⎤
= 2⎢ ⎥
⎣ − (1+ iω ) ⎦ 0
−2 2
=
1+ iω
[0 −1] =
1+ iω
Given RXX (τ ) = e −2τ
= ∫ e−2 τ e −iωτ dτ
−∞
0 ∞
∫ e e dτ + ∫ e 0 e dτ
2τ −iωτ −2τ −iωτ
= −∞
0 ∞
( 2−iω )τ − ( 2+iω )τ
= ∫e dτ + ∫ e dτ
−∞ 0
5
Unit.5. Linear System with Random Inputs
0 ∞
⎡ e( 2−iω )τ ⎤ ⎡ e−( 2−i ω )τ ⎤
=⎢ ⎥ +⎢ ⎥
⎣ 2 − iω ⎦ −∞ ⎣ − ( 2 + iω ) ⎦ 0
1 1
= [1− 0]− [0 −1]
2 − iω 2 + iω
1 1
= +
2 − iω 2 + iω
2 + iω + 2 − iω 4
= =
(2 + iω )( 2 − iω ) 4 + ω 2
We know the power spectral density of the output process Y (t ) is given by
S YY (ω ) = H (ω ) S (ω )
2
XX
2
4
= 2
1+ iω 4 + ω
2
4 4
=
(1+ ω ) 4 + ω2
2
16
=
( )(
1+ ω 2 4 + ω 2 )
Problem 17. If Y ( t ) = A cos ( ω0 t + θ ) + N ( t ) , where A is a constant , θ is a random
variable with uniform distribution in (−π ,π ) and N (t ) is a band-limited Gaussian white
, for ω − ω < ω
⎧ N0
⎪
noise with a power spectral density SNN (ω ) = ⎨ 2 0 B
. Find the power
⎪⎩0, elsewhere
spectral density of Y (t ) . Assume that N (t ) and θ are independent.
Solution:
Given Y ( t ) = A cos ( ω0 t + θ ) + N ( t )
N (t ) is a band-limited Gaussian white noise process with power spectral density
N0
SNN (ω ) = , ω − ω 0 < ωB ie. ω0 − ωB < ω < ω0 + ωB
2
∞
Required SYY (ω ) = ∫ R (τ ) e
YY
−iωτ
dτ
−∞
6
Unit.5. Linear System with Random Inputs
∴E ⎡⎣ cos ( ω0 t + θ ) ⎤⎦ = ∫ cos ( ω0 t + θ ) f (θ ) dθ
−π
π
1
= ∫ [cos ω t.cos θ − sin ω t.sin θ ]
−π
0 0
2π
dθ
π π
1 ⎡ ⎤
= ⎢ cos ω 0 ∫
t cos θ d θ − sin ω0 ∫
t sin θ dθ⎥
2π
1 ⎣ −π −π ⎦
= ⎡ cos ω t [sin θ ]π – sin ω t.0 ⎤ = 0
2π ⎣
0
−π 0 ⎦
π
1
Similarly E ⎡⎣ cos ( 2ω0 t + ω0τ + 2θ ) ⎦⎤ = cos ( 2ω0t + ω0τ + 2θ )
∫ −π
2π
dθ
π
1
=
2π ∫ ⎡⎣ cos ( 2ω t + ω τ ) cos 2θ − sin ( 2ω t + ω τ ) sin 2θ ⎤⎦ dθ
−π
0 0 0 0
π π
1 ⎧ ⎫
= ⎨cos ( 2ω0 t + ω0τ ) ∫ cos 2θ dθ − sin ( 2ω0 t + ω0τ ) ∫ sin 2θ dθ ⎬
2π
⎩ −π π −π ⎭
1⎧⎪ ⎡ sin 2θ ⎤ ⎪⎫
= ⎨cos ( 2ω0 t + ω0τ ) . ⎢ − sin ( 2ω0 t + ω0τ ) .0 ⎬ = 0
2π ⎪⎩ ⎣ 2 ⎥ −π ⎪⎭
A2
∴ R (τ ) =
YY
cos ω0τ + RNN (τ )
2 ⎤ −iωτ
∞
⎡ A2
∴ SYY ( ω ) = ∫ ⎢ cos ω0τ + RNN (τ ) ⎥ e dτ
2
−∞ ⎣ ⎦
2 ∞ ∞
A
= 2 cos ω τ 0.e −iωτ d τ +
∫ ∫ RNN (τ ) e−i ωτ dτ
−∞ −∞
π A2
= { δ (ω − ω ) + δ (ω + ω ) } + S ( ω )
0 0 NN
2
7
Unit.5. Linear System with Random Inputs
SYY ( f ) = 1 N0
1+ 4π f R C 22
2 2 2
1 ∞
∴RYY (τ ) = ∫ SYY ( ω ) eiωτ d ω
2π −∞
∞
1 ei22πτ f2 2 2 N 0 d f
= ∫
2π −∞ 1+ 4π f R C 2
N ∞ ei ( 2πτ ) f
= 4π0 −∞ 2 2 2 ⎛
∫ 4π R C ⎜ 1 + f 2 ⎞⎟ df
4π 2 R 2 C 2
⎝ ⎠
N ∞
ei ( 2πτ ) f
=
16π R C
23 02
−∞ ⎛ 1 ⎞
∫ 2 df
⎜ + f2
⎝ 2 π ⎠
⎟ ∞ π −ma
RC eimx
We know from contour integration ∫ dx = e
−∞ a2 + x2 a
τ
N0 π −
∴R (τ ) = e 2π RC
16π 3 R2 C 2
YY
1
2π RC
N0 τ
–
= 2π 2 RC e 2π RC
16π 3 R2 C 2
τ
N0 –
= e 2π RC
8π RC
Problem 19. A wide-sense stationary noise process N(t) has an autocorrelation function
RNN (τ ) = Pe−3 τ , where P is a constant Find its power spectrum
Solution:
8
Unit.5. Linear System with Random Inputs
= ∫ P e−3 τ e −iωτ dτ
−∞
∞
⎧0 ⎫
= P ⎨ ∫ e e dτ + ∫ e e dτ ⎬
3τ −iωτ −3τ −iωτ
⎩ −∞ 0 ⎭
∞
⎧ 0 ( 3−iω )τ ⎫
= P⎨ ∫ e dτ + ∫ e− ( 3+iω )τ dτ ⎬
⎩ −∞ 0 ⎭
0 ∞
⎧⎪ ⎡ e( ) ⎤
3−i ω τ ⎡ e( ) ⎤
− 3+i ω τ
= P ⎨⎢ ⎥ +⎢ ⎥
⎪⎩ ⎣ 31− iω ⎦ −∞ ⎣ − ( 31 + iω ) ⎦ 0
⎧ ⎫
= P ⎨ 3 − iω (1− 0 ) − 3 + iω ( 0 −1)⎬
⎩ ⎭
⎧ 1 1 ⎞
=P +
⎨⎩ ⎟
3 − iω 3 + iω ⎠⎬
⎧⎨⎪( 33 −
+ iiωω )(
+ 33−+iωiω ) ⎪
⎫
= 9+ω
6P 2
=P
⎩⎪ ⎪⎭
Problem 20. A wide sense stationary process X(t) is the input to a linear system with
impulse response h (t ) = 2e−7t ,t ≥ 0 . If the autocorrelation function of X(t) is
RXX (τ ) = e −4 τ , find the power spectral density of the output process Y(t).
Solution:
Given X(t) is a WSS process which is the input to a linear system and so the output
process Y(t) is also a WSS process (by property autocorrelation function)
Further the spectral relationship is S YY (ω ) = H (ω ) S (ω )
2
XX
∫ R (τ )e
−iωτ
= XX
dτ
−∞
∞
= ∫ e−4 τ e −iωτ dτ
−∞
0 ∞
∫ e e dτ + ∫ e 0 e dτ
4τ −iωτ −4τ −iωτ
= −∞
0 ∞
τ ( 4−iω ) −τ ( 4+iω )
= ∫e dτ + ∫ e dτ
−∞ 0
9
Unit.5. Linear System with Random Inputs
0 ∞
= ⎢⎡ eτ ( 4−iω ) ⎤⎥ + ⎢⎡ e −τ ( 4+iω ) ⎥⎤
4 − iω 4 + iω
⎣ ⎦ −∞ ⎣ ⎦0
1 1
= ⎡⎣ e − e ⎤⎦ +
0 −∞ ⎡⎣ e −∞ − e 0 ⎤⎦
4 − iω 4 + iω
1 1
= –
4 − iω 4 + iω
4 + iω − 4 + iω 2iω
SXX (ω ) = =
(4 − iω )( 4 + iω ) 16 + ω 2
H (ω ) = Fourier transform of h (t )
∞
= ∫ h (t )e−iωtdt
−∞
∞
⎡⎣ h ( t ) = 0 if t < 0⎤⎦
= ∫0 2e−7t .e−iωt dt
∞
= 2∫ e −( 7+iω ) t dt
0
∞
⎡ e−(7+iω )t ⎤
= 2⎢ ⎥
⎣ − ( 7 + iω ) ⎦ 0
−2 2
= ⎡⎣ e −∞ − e0 ⎤⎦ =
7 + iω 7 + iω
2 2
∴ H (ω ) = =
7 + iω 49 + ω 2
4
∴ H (ω ) =
2
49 + ω 2
Substituting in (1) we get the power spectral density of Y(t),
4 2iω 8iω
SYY (ω ) = . = .
49 + ω 16 + ω
2 2
(
49 + ω 2 16 + ω 2 )( )
Problem 21. A random process X(t) with RXX (τ ) = e −2 τ is the input to a linear system
whose impulse response is h (t ) = 2e−t , t ≥ 0 . Find cross correlation RYY (τ ) between the
input process X(t) and the output process Y(t).
Solution:
The cross correlation between input X(t) and output Y(t) to a linear system is
RXY (τ ) = RXX (τ ) h (τ )
Taking Fourier transforms, we get
SXY (ω ) = RXX (ω ) H (ω )
Given RXX (τ ) = e −2 τ
10
Unit.5. Linear System with Random Inputs
= ∫ e−2 τ e −iωτ dτ
−∞
0 ∞
∫ e e dτ + ∫ e 0 e dτ
2τ −iωτ −2τ −iωτ
= −∞
0 ∞
= ∫ e( 2−iω )τ dτ + ∫ e −( 2+iω )τ dτ
−∞ 0
0 ∞
⎡ e( 2−iω )τ ⎤ ⎡ e−( 2+iω )τ ⎤
=⎢ ⎥ +⎢ ⎥
⎣ 2 − iω ⎦ −∞ ⎣ − ( 2 + iω ) ⎦ 0
1 1
= (1− 0) − (0 −1)
2 − iω 2 + iω
1 1 4
= + =
2 − iω 2 + iω 4 + ω 2
∞
∴ H (ω ) = ∫ h (t ) e−iωtdt
−∞
∞
∫ 2 e e dt
−t −iωt
= −∞
∞
∞
−( 1+iω ) t
⎡ e−(1+iω )t ⎤
= 2∫e dt = 2 ⎢ ⎥
0
⎣ − (1+ iω ) ⎦ 0
−2
= [0 −1] = 2
1+ iω 1+ iω
4 2
∴SXY (ω ) = .
4 + ω 2 1+ iω
= 8
(2 + iω )( 2 − iω )(1+ iω )
8 A B C
Let = + +
(2 + iω )( 2 − iω )(1+ iω ) 2 + iω 2 − iω 1+ iω
∴8 = A ( 2 + iω )(1+ iω ) + B (2 + iω )(1+ iω ) + C (2 + iω )( 2 − iω )
Put ω = i 2, then 8 = A ( 4 )( −1 ) ⇒ A = −2
8
ω = i then 8 = C (1)(3) ⇒ C =
3
2
ω = −i 2, then 8 = B ( 4 )( 3 ) ⇒ B =
3
2
−2 8/3
∴ SXY (ω ) = + 3 +
2 + iω 2 − iω 1+ iω
11
Unit.5. Linear System with Random Inputs
⎝ ⎠
2 8
= −2.e − τ μ (τ ) + e τ μ ( −τ ) + e −τ μ (τ )
2 2
3 3
Problem 22. If X(t) is a band limited process such that SXX (ω ) = 0 ,where ω > σ prove
that 2 ⎡⎣ R XX ( 0 ) − RXX (τ ) ⎤⎦ ≤ σ 2τ 2 R XX ( 0)
Solution:
Given SXX (ω ) = 0 , ω > σ
⇒ SXX (ω ) = 0 if ω < −σ or ω > σ
1 ∞
RXX ( τ ) =
∫ SXX (ω ) eiτ ω dω
2π −∞
1 σ S ( ω ) eiτω dω
=
2π ∫
−σ
XX
σ
1
=
2π ∫ S XX ( ω )( cosτω + i sinτω ) dw
−σ
1 ⎧σ σ
⎫
= ⎨ ∫ S XX ( ω ) cosτω dw + i ∫ S XX ( ω ) sin τω dw⎬
2π
⎩ −σ −σ ⎭
σ σ σ
1 σ
∴ RXX (τ ) = 2 ∫ S XX ( ω ) cosτω d ω
2π 0
σ
1
S ( ω )cosτω d ω
π ∫0 XX
=
1σ
∴ RXX (0 )= ∫ SXX ( ω) dω ...(1)
π 0
1σ 1 σ
∴ RXX ( 0 ) − RXX (τ ) = ( )
π −∫σ
SXX ω dω −
π ∫ S XX (ω ) cos ωτ dω
−σ
σ
1
= ∫ S XX ( ω )(1− cos ωτ ) d ω
π1 −σ ⎛ ωτ ⎞
= σ S (ω ) 2 sin2 dω
π ∫ XX ⎜ 2 ⎟
−σ ⎝ ⎠
We know that sin θ ≤ θ
2 2
12
Unit.5. Linear System with Random Inputs
⎛ ωτ ⎞ ⎛ ωτ ⎞
2
ω 2τ 2 σ 2τ 2 ⎡ 0 ≤ ω ≤ 2σ , ω ≤σ ⎤ ⎦
⎟≤⎜
2 2
∴2 sin ⎜ ⎟ = < ⎣
⎝2 ⎠ ⎝2 ⎠ σ 2 222
1 σ τ
∴ RXX ( 0 ) − RXX (τ ) ≤
π ∫0 XX ( ) 2
.S ω dω
σ
1 σ 2τ 2
≤
π 2 ∫ SXX (ω )dω
0
σ
τσ 2 2
≤
2π ∫ SXX (ω ) dω
0
σ
τσ 2 2
≤
2π ∫ SXX (ω ) dω
0
≤σ τ
2 2
RXX (0) [Using (1)]
2π
∴2 ⎡⎣ R XX (0) − RXX (τ ) ⎤⎦ ≤ σ 2τ 2 R XX (0)
Problem 23. The autocorrelation function of the Poisson increment process is given by
⎧λ 2 for τ >∈
R (τ ) = ⎨⎪ λ⎛ τ ⎞ . Prove that its spectral density is given by
⎪ λ 2
+ ⎜1− ⎟ for τ ≤∈
⎩ ∈ ⎝ ∈ ⎠
ωt
4λ sin2
S ( ω ) = 2πλ 2δ ( ω ) + 2 .
Solution:
⎪⎧λ 2 for τ > −∈ or τ >∈
Given the autocorrelation function R (τ ) = ⎛ ⎞
⎨2 λ τ
⎪λ + ∈⎜1− ⎟
∈ ⎠
for −∈≤ τ ≤∈
⎩ ⎝
∞
−∞ −∈ ⎝ ⎠
13
Unit.5. Linear System with Random Inputs
λ ∈⎛ τ ⎞
( )
= F λ 2 + ∈ ∫ ⎜1− ⎟( cosτω − i sin τω dτ)
−∈ ⎝ ∈ ⎠
Where F (λ 2 ) is the Fourier transform of λ 2
λ ⎧⎪ ∈ ⎛ ⎞ ∈⎛ ⎞ ⎫⎪
( )
S (ω ) = F λ +2
⎨∫ ⎜ 1− τ ⎟ cos τω dτ − i ∫⎜ 1− τ ⎟ sin τω d τ ⎬ .....(1)
∈ ⎩⎪−∈ ⎝ ∈⎠ −∈ ⎝
∈ ⎠ ⎪⎭
⎛ τ ⎞ ⎛ τ ⎞
But ⎜1− ⎟ cosτω is an even function of τ and ⎜1− ⎟ sin τω is an odd function of τ
⎝ ∈ ⎠ ⎝ ∈ ⎠
∈⎛ ⎞ ∈⎛ ⎞ ∈⎛ τ⎞
τ τ
∴ ∫ ⎜1− ⎟ sin τω dτ = 0 and ∫ ⎜1− ⎟ cosτω dτ = 2 ∫ ⎜1− ⎟ cos τω dτ ( τ > 0; τ =τ )
∈ ∈ ∈ ⎝ ⎠
−∈ ⎝ ⎠ λ ∈⎛ τ ⎞ −∈ ⎝ ⎠ 0
( )
S (ω ) = F λ 2 +
∈ 0⎝
2∫ ⎜1− ⎟ cos τω dτ
∈⎠
∈
2λ ⎡ ⎛ τ ⎞⎛ sin τω ⎞ ⎛ −1 ⎞⎛ − cos τω ⎞⎤
= F λ + ⎢ ⎜ 1−⎟⎜
2
∈ ( ) ∈
−
ω ⎟ ⎜ ∈ ⎟⎜ ω2 ⎟⎥ [by Bernoulli’s formula]
⎣⎝ ⎠⎝ ⎠ ⎝ ⎠⎝ ⎠⎦ 0
2λ ⎡ 1 ⎛ τ ⎞
( ) 1 ⎤ ∈
=F λ + 2
⎜1− sin ⎟ τω − cosτω
∈ ⎢ω ∈ ∈ω 2 ⎥
⎣ ⎝ ⎠ ⎦0
2λ ⎡
= F (λ ) +
1 ⎤
2
0− ( cos ∈ ω − cos 0 )
∈ ⎢ ⎣ ∈ω 2 ⎥⎦
2λ 1
= F λ2 + ( ) [1− cos ∈ ω ]
∈ ∈ω2
2λ ∈ω
= F (λ 2 ) + .2 sin2
∈ ω2 2
2
4λ 2 ∈ω
S (ω ) = F (λ ) +
2
sin .......(1)
∈ ω2
2
2
To find the value of F (λ 2 ) , we shall find the inverse Fourier transform of S (ω ) ,
R (τ ) = F − 1 ( S (ω ))
1∞
= S ( ω ) eiωτ d ω
2π −∞ ∫
Consider S ( ω ) = 2πλ 2δ ( ω ) , where δ ( ω ) is the unit impulse function.
∞
1
R (τ ) = 2πλ 2δ ( ω ) ei ωτ d ω
2π −∞ ∫
∞
=λ ∫ δ (ω ) eiωτ d ω
2
−∞
∞
= λ .1
2
[ ∫ φ (t )δ (t ) dt = φ (0)
−∞
14
Unit.5. Linear System with Random Inputs
=λ 2
⇒ ∫ ei ωτ δ ( ω ) dω = e0 = 1 as φ ( ω ) = eiτω ⎤⎦
−∞
∴ SXX (ω ) = ∫ R (τ ) e
XX
−iωτ
dτ
−∞
∞
= ∫ 3δ (τ ) e −iωτ dτ
−∞
∞
= 3 ∫ δ (τ ) e −iωτ dτ
−∞
∞
We know
∫ δ (τ )φ (τ ) = φ (0 )
−∞
Here φ (τ ) = e −iωτ ∴φ ( 0 ) = 1
∴ SXX (ω ) = 3.1 = 3
We know the spectral relation between input and output process is
S YY (ω ) = H (ω ) S (ω )
2
XX
1
But H (ω ) =
2
36 + ω 2
3
∴ SYY (ω ) = which is the power spectral density of Y (t )
36 + ω 2
Now the autocorrelation of Y (t ) is R (τ ) = F −1 ( S (ω ))
YY YY
⎛ 3 ⎞
R (τ ) = F − 1
YY ⎜ 36 + ω 2 ⎟
⎝ ⎠
We know F −1 ⎛ 2α ⎞ = e−α τ
⎝⎜ α 2 + ω 2 ⎟ ⎠ ⎞
∴ R (τ ) = 3
F −1 ⎛ 2.6 [Here α = 6]
YY ⎜ 62 + ω 2 ⎟
2.6 ⎝ ⎠
15
Unit.5. Linear System with Random Inputs
1 −6 τ
= e
4
∞
(ii) SYY (ω ) = ∫ R (τ ) e
YY
−iωτ
dτ
−∞
∞ 1 −6 τ −iτω
= ∫ 4 e e dτ
−∞
∞
1⎧ 0 ⎫
= ⎨ ∫ e( 6τ −iω )τ dτ + ∫ e−( 6+iω )τ dτ ⎬
4 ⎩−∞ 0 ⎭
+ ⎢⎡ e−( 6+iωτ ) ⎥ ⎤ ∞ ⎫⎬⎪
0
= 1 ⎧⎨⎢
⎪ ⎡ e( ) ⎤⎥
6−iω τ
4 6 − iω − ( 6 + iω )
⎩⎪ ⎣ ⎦ −∞ ⎣
1
⎦ 0 ⎪⎭
⎫
1 ⎧ 1 (1− 0) −
= ⎨ ( 0 −1)
⎬
4 ⎩ 6 − iω 6 + iω ⎭
1 ⎧⎨ 1 1 ⎫ ⎬
= +
4 ⎩ 6 − iω 6 + iω ⎭
1 ⎧⎨⎪ 6 + iω + 6 − iω ⎫⎪⎬
= 4 ( 6 − iω )( 6 + iω )
⎩⎪ ⎪⎭
1 ⎧ 12 ⎫ 3
= ⎨ 2⎬ = 2 .
4 ⎩ 36 + ω ⎭ 36 + ω
Problem 25. Show that the power spectrum SYY (ω ) of the output of a linear system with
If {X (t )} is a WSS and if y (t ) = ∫ h ( α )X (t − α ) dα
−∞
Consider Y (t ) = ∫ X (t − α ) h (α ) dα
−∞
∞
X ( t + τ ) Y ( t ) = ∫ X ( t + τ ) X ( t − α ) h (α ) d α
−∞
∞
E ⎡⎣ X ( t + τ ) Y ( t ) ⎤⎦ = ∫ E ⎣⎡ X ( t + τ ) X ( t − α ) ⎤⎦ h ( α ) dα
−∞
∞
RYX ( −τ ) = ∫ RXX (τ + α ) h (α ) dα
−∞
∞
RXY (τ ) = ∫ RXX (τ − β ) h ( − β ) d β
−∞
16
Unit.5. Linear System with Random Inputs
R XY (τ ) = R XX (τ ) *h ( −τ )
∞
Y ( t ) Y ( t − τ ) = ∫ X ( t − α ) Y ( t − τ ) h ( α ) dα
−∞
∞
∴E ⎡⎣ Y ( t ) Y ( t − τ ) ⎤⎦ = ∫ RXY (τ − α )h ( α ) d α
−∞
Problem 26. A system has an impulse response h ( t ) = e−β tU ( t ) , find the power spectral
density of the output Y (t ) corresponding to the input X (t ) .
Solution:
Given h ( t ) = e −β t , t ≥ 0
∞
H (ω ) = ∫ h (t ) e−iωtdt
−∞
∞
H (ω ) = ∫ e −β t e − iωt dt
0
∞
= ∫ e−t ( β +i ω ) dt
0
∞
⎡ e −t ( β +iω ) ⎤
=⎢ ⎥
⎣ − ( β + iω ) ⎦ 0
1
H (ω ) =
β + iω
1
H * (ω ) =
β − iω
H (ω ) = H (ω ) H * (ω )
2
⎛ 1 ⎞⎛ 1 ⎞
=
⎜⎝ ⎟⎜ ⎟
β + iω ⎠⎝β − iω ⎠
17
Unit.5. Linear System with Random Inputs
1
=
β +α2
2
(ω ) = H (ω ) (ω )
2
∴S YY S XX
SXX (ω )
SYY (ω ) = 2 2
β +α
Problem 27. If X (t ) is the input voltage to a circuit and Y (t ) is the output voltage,
{ X (t )} is a stationary random process with μx = 0 and RXX (τ ) = e−2 τ . Find μ y , SXX (ω )
1
and SYY (ω ) , if the system function is given by H (ω ) = 2 2 .
ω +2
Solution:
Given Mean ⎡ ⎣ X (t )⎤ ⎦ = μX = 0
∞
Y (t ) = ∫ h (α ) X (t − α ) dα
−∞
∞
E ⎡⎣ Y ( t ) ⎤⎦ = ∫ h (α ) E ⎡⎣ X ( t − α ) ⎤⎦ dα = 0
−∞
∞
SXX (ω ) = RXX (τ ) e −i ωτ dτ
∫
−∞
∞ ∞ ∞
S (ω ) = H ( ω ) S
(ω ) = ⎛⎜ 1 ⎞ 4
⎟ 2
⎝ ω + 4 ⎠ω + 4
YY XX 2
4
SYY (ω ) =
(ω )
2
2
+4
Problem 28. X (t ) is the input voltage to a circuit (system) and Y (t ) is the output
voltage. {X (t )} is a stationary random process with μx = 0 and RXX (τ ) = e−α τ . Find
R
μ y , Syy (ω ) & Ryy (τ ) if the power transfer function is H (ω ) =
R + iLω
Solution:
∞
Y (t ) = ∫ h (α ) X (t − α ) dα
−∞
18
Unit.5. Linear System with Random Inputs
E ⎡⎣ Y ( t ) ⎤⎦ = ∫ h (α ) E ⎡⎣ X ( t − α ) d α ⎤⎦
−∞
E ⎡⎣ Y ( t ) ⎤⎦ = 0
∞
SXX (ω ) = ∫R XX (τ ) e−iωτ dτ
−∞
0 ∞
∫ e e dτ + ∫ e 0 e dτ
ατ −iωτ −ατ −iωτ
= −∞
= ⎢⎡ e(α −iω )τ ⎥⎤ 0 + ⎢⎡ e−(α +iω )τ ⎤ ⎥∞ = + =
α − iω − ( α + iω ) α − i1 +1iω α 2 +2ωα 2
⎣ ⎦ −∞ ⎣ ⎦0 ω α
SYY (ω ) = SXX (ω ) H (ω )
2
2α R
=
α + ω R + L 2ω 2
22 2
Consider,
2α R2 A B
= 2 2 + 2 2 2
(α + ω )( R + L ω ) α + ω R + L ω
2 2 2 2 2
By partial fractions
⎛ ⎞
⎜ R2 ⎟
⎛ R2 2α ⎜ 2 ⎟
2α ⎜ 2 2 2 ⎟ ⎞ ⎜ α − R2 ⎟
R −Lα 2
SYY (ω ) = ⎝ ⎠+ ⎝ L ⎠
α2 + ω
2 2
R + L 2ω
2 2 2
⎛R⎞ ⎛ R2 ⎞
2α ⎜ ⎟ 2α ⎜ 2 ⎟
⎝ L ⎠ 1
. α 2 + ω2 + ⎝L ⎠2 . 1
= ⎛ R ⎞2 ⎛ R ⎞ ⎛ R ⎞
2
α 2 − ⎜ ⎟ ⎜ ⎟ + ω2
⎜ ⎟ −α
2
⎝L⎠ ⎝L ⎠ ⎝L ⎠
1 1
= λ. 2 2 + μ.
α +ω ⎛R⎞2
⎜ ⎟ +ω
2
⎝L⎠
λ ∞ eiωτ dω + μ ∞∫ eiωτ dω
RYY (τ ) =
2π
∫ α 2
+ ω 2
2 π ⎛ R ⎞
2
−∞ −∞
⎜ ⎟ +ω
2
⎝ ⎠L
By contour integration technique we know that
eiaz dz = e ab , a > 0
2 2
z +b b
19
Unit.5. Linear System with Random Inputs
⎛ R ⎞2 ⎛ R ⎞2
⎜L⎟α −⎛ ⎞ τ
R
⎜L⎟ L
∴ RYY (τ ) = ⎛ R⎝⎞2 ⎠ −α τ
e +
⎝ ⎠R 2 ⎜ ⎟
⎛ ⎞
e⎝ ⎠
⎜ ⎟ − α 2
α 2
− ⎜ ⎟
⎝L⎠ ⎝L⎠ .
Problem 29. X (t )is the i/P voltage to a circuit and Y (t ) is the O/P voltage. X (t ) is a
stationary random process with zero mean and autocorrelation RXX (τ ) = e−2 τ . Find the
1
mean of Y (t ) and its PSD if the system function H (ω ) = .
Jω + 2
Solution:
1
H (ω ) =
Jω + 2
1
⇒ H ( 0) =
2
E ⎡⎣ Y ( t ) ⎤⎦ = E ⎡⎣ X ( t ) ⎤⎦ .H ( 0 ) = 0
1
H (ω ) =
2
ω +4 2
4
S ( ω ) = F ⎡ R (τ ) ⎤ = F ⎡ e−2 τ ⎤ =
XX ⎣ XX ⎦
⎣ ⎦ ω2 + 4
4
SYY ( ω ) = H (ω ) S XX (ω ) =
2
( )
2
ω2 + 4
0 2
( ) ∫ ( 2 + τ ) ( 9 + 2eτ ) d τ + ∫ ( 2 − τ ) (9 + 2e−τ ) d τ
∴E Y 2 =
−2 0 0 2
⎡ 9τ 2 ⎤ ⎡ 9τ 2 ⎤
= ⎢ 18τ + 4e +
τ + 2 e (τ −1) ⎥
τ
+ ⎢ 18τ − 4eτ − 2 e −τ (τ +1) ⎥
⎣ 2 ⎦ −2 ⎣ 2 ⎦0
∴E (Y2 ) = 40.542
V ar ( Y ) = E (Y 2 ) − ⎡⎣ E ( Y ) ⎤⎦
2
= 40.542 − 36 = 4.542
1
Problem 30. Consider a system with transfer function . An input signal with
1+ iω
autocorrelation function mδ (τ ) + m 2 is fed as input to the system. Find the mean and
mean-square value of the output.
Solution:
1
Given, H (ω ) = and RXX (τ ) = m δ (τ ) + m 2
1+ iω
SXX ( ω ) = m + 2π m 2δ ( ω )
We know that, SYY (ω ) = H (ω ) S (ω )
2
XX
20
Unit.5. Linear System with Random Inputs
2
1
= ⎡ m + 2π m 2δ ( ω ) ⎤⎦
1+1iω ⎣
= ⎡ m + 2π m 2δ ( ω ) ⎤
1+ ω 2 ⎣ ⎦
RYY (τ ) is the Fourier inverse transform of SYY (ω ).
m −τ
So, RYY (τ ) = e + m2
2
lim
RXX (τ ) = X
2
We know that
τ →∞
2
So X = M 2
X =m
Also H ( 0 ) = 1
We know that Y = 1, m = m
m
Mean-square value of the output = Y = RYY ( 0 ) = + m2
2
2
Problem 31. If the input to a time-invariant, stable, linear system is a WSS process,
prove that the output will also be a WSS process.
Solution:
Let X (t ) be a WSS process for a linear time variant stable system with Y (t ) as the
output process.
∞
Then Y (t ) =
−∞
∫ h (u ) X (t − u ) du where h(t) is weighting function or unit impulse
response.
∞
∴ E ⎡⎣ Y ( t ) ⎤⎦ = ∫ E ⎡⎣ h ( u ) X ( t − u ) ⎤⎦ du
−∞
∞
= ∫ h (u ) E ⎣ ⎡ X (t − u )⎤ ⎦ du
−∞
∴ E ⎡⎣ Y ( t ) ⎤⎦ = ∫ h ( u ) μ X du
−∞
∞
= μX
∫ h (u ) du
−∞
∞
∴ E ⎡⎣ Y ( t ) ⎤⎦ is a constant.
21
Unit.5. Linear System with Random Inputs
Now RYY ( t, t + τ ) = E ⎡⎣ Y ( t ) Y ( t + τ ) ⎤⎦
⎡ ∞ ∞
⎤
= E ⎢ ∫ h ( u1 ) X ( t − u1 ) du1 ∫ h ( u2 ) X ( t + τ − u2 ) du2 ⎥
⎣ −∞ −∞ ⎦
⎡ ∞ ∞ ⎤
= E ⎢ ∫ ∫ h ( u1 ) h ( u2 ) X ( t − u1 ) X ( t + τ − u2 ) du1u2 ⎥
⎣ −∞ −∞ ⎦
∞∞
= ∫ ∫ h ( u1 ) h ( u2 ) E ⎡⎣ X ( t − u1 ) X ( t + τ − u2 )⎤⎦ du1u2
−∞ −∞
Since X (t ) is a WSS process, auto correlation function is only a function time difference.
∞∞
When this double integral is evaluated by integrating w.r. to u1 , u2 , the R.H.S is only a
function of τ .
∴ RYY ( t, t + τ ) is only a function of time difference τ .
Hence Y (t ) is a WSS process.
∞
a) RXY (τ ) = h (τ ) * RXX (τ )
b) RYX (τ ) = h ( −τ ) * RXX (τ )
c) Ryy (τ ) = h (τ ) * Rxy (τ )
Where * denotes the convolution and H * (ω ) is the complex conjugate of H (ω ) .
Solution:
Given X (t ) is WSS∴E ⎣ ⎡ X (t )⎤ ⎦ is constant and
RXX ( t, t + τ ) = RXX (τ )
∞
Y (t ) = ∫ h (u ) X (t − u ) du
−∞
Now RXY ( t, t + τ ) = E ⎡⎣ X ( t ) Y ( t + τ ) ⎤⎦
⎡ ∞ ⎤
= E ⎢ X ( t ) ∫ h ( u ) .X ( t + τ − u ) du ⎥
⎣∞ −∞ ⎦
⎡ ⎤ ∞
= E ⎢ ∫ h ( u ) X ( t ) X ( t + τ − u ) du ⎥ = ∫ h ( u ) .E ⎡⎣ X ( t ) X ( t + τ − u ) ⎤⎦ du
⎣ −∞ ⎦ −∞
Since X (t ) is a WSS Process,
E ⎡⎣ X ( t ) X ( t + τ − u ) ⎤⎦ = RXX (τ − u )
∞
∴ RXY ( t, t + τ ) = ∫ h ( u ) RXX (τ − u ) du
−∞
22
Unit.5. Linear System with Random Inputs
⇒ RXY (τ ) = RXX (τ )* h (τ )
(b). Now RYX (τ ) = RXY ( −τ )
= RXY ( −τ ) * h ( −τ ) [ from (i) ]
= RXX (τ ) * h ( −τ ) [Since RXX (τ ) is an even function of τ ]
(c). RYY ( t, t − τ ) = E ⎣⎡ Y ( t ) Y ( t − τ ) ⎤⎦
⎡∞ ⎤
= E ⎢ ∫ h ( u ) X ( t − u ) du Y ( t − τ ) ⎥
⎣ −∞∞ ⎦
⎡ ⎤
= E ⎢ ∫ X ( t − u ) Y ( t − τ ) h ( u ) du ⎥
⎣ −∞ ⎦
∞ ∞
= ∫ E ⎡⎣ X ( t − u ) Y ( t − τ ) ⎤⎦ h ( u ) du = ∫ RXY (τ − u ) h ( u ) du
−∞ −∞
= ∫ h (u )( t − u ) du
−∞
∴ E ⎣⎡ Y ( t ) ⎤⎦ = ∫ h ( u ) μ X du = μ X ∫ h ( u ) du
−∞ −∞
i.e. H (ω ) =
∫ h (t ) dt
−∞
∞ ∞
put ω = 0 ∴H (0) = ∫ h (t ) dt = ∫ h (u ) du
−∞ −∞
∴E ⎡⎣ Y ( t ) ⎤⎦ = μ X H ( 0 ) .
23
UNIT –I RANDOM VARIABLES
PART-A
Problem1. X and Y are independent random variables with variance 2 and 3. Find the variance
of 3X + 4Y .
Solution:
V (3X + 4Y ) = 9Var( X ) +16Var(Y ) + 24Cov( XY )
= 9× 2 +16 × 3 + 0 (∴ X & Y are independent cov( XY ) = 0 )
= 18 + 48 = 66.
Problem 2. A Continuous random variable X has a probability density function F ( x) = 3x2;
0 ≤ x ≤ 1. Find ‘a’ such that P ( x ≤ a ) = P ( x > a )
Solution:
We know that the total probability =1
Given P ( X ≤ a ) = P ( X > a ) = K (say)
Then K + K = 1
1
K=
2
1 1
ie P ( X ≤ a ) = & P ( X > a ) =
2 2
1
Consider P ( X ≤ a) =
2
a
1
i.e. f ( x) dx =
∫0 2
a 1
∫0 3x dx = 2
2
a
⎛ x3 ⎞ 1
3⎜ ⎟ =
⎝ 3 ⎠0 2
1
a3 =
2
1/ 3
⎛1⎞
a=⎜ ⎟ .
⎝2⎠
⎧Cxe− x ; if x > 0
Problem 3. A random variable X has the p.d.f f ( x) given by f ( x) = ⎨
⎩0 ; if x ≤ 0
Find the value of C and cumulative density function of X .
1
Solution:
∞
Since
∫ f ( x) dx = 1
−∞
∞
∫ Cxe−xdx = 1
0
( )( )
∞
C ⎡⎣ x −e − x − e− x ⎤ ⎦= 1
0
C =1
⎧xe − x ; x > 0
∴ f x( =) ⎨
⎩ 0 ;x ≤ 0
−t x
x x
⎡ −t −t ⎤ −x −x
= 1− (1+ x) e −x .
⎧1
⎪ ( x +1); −1 < x < 1
Problem 4. If a random variable X has the p.d.f f ( x ) = ⎨2 .
⎪
⎩ 0 ; otherwise
Find the mean and variance of X .
Solution:
1
11 11 2
Mean= ∫ xf ( x) dx = 2 ∫ (x x +1 ) dx =
2 ∫ x + x dx ( )
−1 −1 −1
1
2
1 ⎜⎛ x3 x ⎞ 1
= 2 3 + ⎟ =
⎝ 2 ⎠−1 3
2 1
(
μ ′ = 1 x f ( x)dx = 1 x + x dx = 1 ⎡ x
3 2 4
+ x 3
⎤⎥
1
)
⎢
2 ∫−1 ∫
2 −1 2⎣4 3 ⎦ −1
1 ⎡ 1 1 1 1⎤
= ⎢ + − +
2 ⎣ 4 3 4 3⎦
1 2 1
= .⎥ =
2 3 3
()
2
Variance = μ2′ − μ ′ 1
1 1 3 −1 2
= − = = .
3 9 9 9
⎧2e−2 x ; x ≥ 0
Problem 5. A random variable X has density function given by f ( x) = ⎨ .
⎩0 ; x < 0
Find m.g.f
Solution:
2
∞ ∞
( )
M X (t ) = E etx = etx f ( x ) dx = e 2e dx
∫ ∫
tx −2 x
0 0
∞
= 2∫ e( t −2 ) x dx
0
∞
⎡ (t −2)x ⎤ 2
= 2⎢ e ⎥ = ,t < 2 .
t −2 2−t
⎣ ⎦0
Problem 6. Criticise the following statement: “The mean of a Poisson distribution is 5 while the
standard deviation is 4”.
Solution: For a Poisson distribution mean and variance are same. Hence this statement is
not true.
Problem 7. Comment the following: “The mean of a binomial distribution is 3 and variance is 4
Solution:
In binomial distribution, mean >variance but Variance < Mean
Since Variance = 4 &Mean = 3, the given statement is wrong. ⎛ 1⎞ ⎛ 1⎞
Problem8. If X and Y are independent binomial variates B 5, and B 7,
⎜ 2⎟ ⎜ 2⎟
⎝ ⎠ ⎝ ⎠
find P[ X + Y = 3]
Solution:
1
X + Y is also a binomial variate with parameters n1 + n2 = 12 & p =
2
3 9
⎛1⎞ ⎛1⎞
∴ P [ X + Y = 3] = 12C3 ⎜ ⎟ ⎜ ⎟
⎝2 ⎠ ⎝ 2 ⎠
55
= 10
2
4
Problem 9. If X is uniformly distributed with Mean1and Variance , find P[ X > 0]
3
Solution:
If X is uniformly distributed over (a, b ) , then
E ( X ) = b + a and V ( X ) = ( b − a )
2
2 12
b+a
∴ =1⇒a+b=2
2
(b − a) 4
2
= ⇒ ( b − a ) = 16
2
⇒
12 3
⇒ a + b = 2 & b − a = 4 We get b = 3, a = −1
∴a = −1& b = 3 and probability density function of x is
3
⎧1
⎪ ; −1 < x < 3
f ( x) = ⎨ 4
⎪⎩0 ;Otherwise
0
1 1 0 1
P [ x < 0] = ∫ dx = [ x ]−1 = .
−
4 4 4
1
Problem 10. State the memoryless property of geometric distribution.
Solution:
If X has a geometric distribution, then for any two positive integer ' m ' and ' n '
P ⎡ X > m + n X > m⎤ = P [ X > n] .
⎣ ⎦
Problem 11. X is a normal variate with mean = 30 and S.D = 5
Find the following P [26 ≤ X ≤ 40]
Solution:
X N (30, 52 )
∴ μ = 30 & σ = 5
X −μ
Let Z = be the standard normal variate
σ ⎡ 26 − 30 40 − 30 ⎤
P [26 ≤ X ≤ 40] = P ≤Z≤
⎣⎢ 5 5 ⎥⎦
= P [−0.8 ≤ Z ≤ 2] = P [−0.8 ≤ Z ≤ 0] + P [0 ≤ Z ≤ 2]
= P [0 ≤ Z 0.8] + [ 0 ≤ z ≤ 2]
= 0.2881 + 0.4772 = 0.7653 .
⎡ 3⎤
Problem 12. If X is a N ( 2, 3) Find P Y ≥ where Y +1 = X .
⎢⎣
2⎥ ⎦
Solution:
⎡ 3⎤ ⎡ 3⎤
P Y ≥ = P X −1 ≥
⎣ ⎢ 2⎥ ⎦ ⎢⎣ 2⎥ ⎦
= P[ X ≥ 2.5] = P [Z ≥ 0.17]
= 0.5 − P [0 ≤ Z ≤ 0.17]
= 0.5 − 0.0675 = 0.4325
1
Problem 13. If the probability is that a man will hit a target what is the chance that he
4
will hit the target for the first time in the 7th trial?
Solution:
The required probability is
P[FFFFFFS ] = P ( F ) P ( F ) P ( F ) P ( F ) P ( F ) P ( F ) P ( S )
6
⎛ 3 ⎞ . ⎛ 1 ⎞ = 0.0445 .
=q p=⎜ ⎟ ⎜ ⎟
6
⎝4⎠ ⎝4⎠
4
Hence p = Probability of hitting target and q = 1− p .
Problem 14. A random variable X has an exponential distribution defined by p.d.f.
f ( x ) = e−x , 0 < x < ∞ . Find the density function of Y = 3X + 5 .
Solution:
dy dx 1
y = 3x + 5 ⇒ =3⇒ =
dx dy 3
P.d.f of y h ( y ) = f ( x) dx
Y X
dy
1
h
Y
( y) = e−x.
3
y −5 ⎞
−⎛
y −5 1 ⎜ ⎟
Using x = we get hY ( y ) = e ⎝ 3 ⎠ , y > 5( x > 0 ⇒ y > 5)
3 3
Problem 15. If X is a normal variable with zero mean and variance σ , Find the p.d.f of y = e−x
2
Solution:
Given Y = e−x
1
f ( x) =
2
1 x
−
e 2σ
2
σ 2π
1 1
h ( y ) = f ( x) dx (log y )
2
–
1 × .
= 2σ 2
e
dy σ 2π
Y X
y
PART-B
Problem 16. A random variable X has the following probability function:
Values of X ,
X : 0 1 2 3 4 5 6 7
P( X ) : 0 K 2K 2K 3K K2 2K 2 7K 2 + K
Find (i) K , (ii) Evaluate P ( X < 6 ) , P ( X ≥ 6) and P (0 < X < 5)
(iii). Determine the distribution function of X .
(iv). P (1.5 < X < 4.5 X > 2)
(v). E (3x − 4) ,Var(3x − 4)
Solution(i):
7
Since ∑ P ( X ) = 1,
x=0
K + 2K + 2K + 3K + K 2 + 2K 2 + 7K 2 + K = 1
10K2+9K−1=0
1
K= or K = −1
10
1
As P ( X ) cannot be negative K =
10
Solution(ii):
5
P ( X < 6) = P ( X = 0) + P ( X = 1) +... + P ( X = 5)
1 81
= + 2 + 2 + 3 + 1 + ... =
10 10 10 10 100 100
Now P ( X ≥ 6) = 1− P ( X < 6)
81 19
= 1− =
100 100
Now P (0 < X < 5) = P ( X = 1) + P ( X = 2) + P ( X = 3) = P ( X = 4)
= K + 2K + 2K + 3K
8 4
= 8K = = .
10 5
Solution(iii):
The distribution of X is given by FX ( x) defined by
F X ( x) = P ( X ≤ x)
X : 0
1 2 3 4 5 6 7
1 3 5 4 81 83
FX (x) : 0 1
10 10 10 5 100 100
⎧x
; x = 1, 2, 3, 4, 5
Problem 17. (a) If P ( x) = ⎪
⎨15
⎪⎩0 ; elsewhere
Find (i) P { X = 1or2} and (ii) P{1/ 2 < X < 5 / 2 x > 1}
(b) X is a continuous random variable with pdf given by
⎧ Kx in 0 ≤ x ≤ 2
⎪2K in 2 ≤ x ≤ 4
⎪
F(X)= ⎨
⎪6K − Kx in 4 ≤ x ≤ 6
⎪⎩0 elsewhere
Find the value of K and also the cdf FX ( x) .
Solution:
(a) i) P ( X = 1 or 2) = P ( X = 1) + P ( X = 2)
1 2 3 1
= + = =
15 15 ⎧⎛151 5 5⎞ ⎫
P < X < ∩ ( X > 1)
⎨⎜2 ⎬
⎛1 5
⎞ ⎩⎝ 2 ⎟⎠ ⎭
ii) P ⎜ < X < / x > 1⎟ =
⎝2 2 ⎠ P ( X > 1)
P{( X = 1or2) ∩ ( X > 1)}
=
P ( X > 1)
6
P ( X = 2)
=
1− P ( X = 1)
1
2 /15 2 /15 2 = .
= = =
1− (1/15) 14 /15 14 7
∞
Since
∫ F ( x) dx = 1
∞
2 4 6
⎟ + ( 2x ) + ∫ ⎜ 6x −
4
K ⎢⎜ ⎟ ⎥ =1
2
2
⎢ ⎣ ⎝ ⎠0 4⎝ 2 ⎠4 ⎥⎦
K ⎡⎣ 2 + 8 − 4 + 36 −18 − 24 + 8⎤⎦ = 1
1
8K = 1 K=
8
x
We know that FX ( x ) =
∫ f ( x) dx
−∞
x
If x < 0 , then FX ( x ) = ∫ f ( x) dx = 0
−∞
x
If x ∈(0, 2) , then FX ( x ) =
∫ f ( x) dx
−∞
0 x
F X ( x) = ∫ f ( x) dx + ∫ f ( x) dx
−∞ 0
0 x 0
1x
= ∫ 0dx + ∫ Kxdx = ∫ 0dx + ∫ xdx
−∞ 0 −∞
8 0
x
2 2
⎛x ⎞ x
=⎜ ⎟ = ,0≤x≤2
⎝ 16 ⎠0 16
x
If x ∈ ( 2, 4) , then FX ( x ) = ∫ f ( x) dx
−∞
0 2 x
F X ( x) = ∫ f ( x) dx + ∫ f ( x) dx + ∫ f ( x) dx
−∞ 0 2
0 2 x
7
1 x 1
= + −
4 4 2
x 4 x −1 , 2 ≤ x < 4
= − =
4 16 4
0 2 4 x
=1, x ≥ 6
⎧0 ;x≤0
⎪ 2
⎪x ;0 ≤ x ≤ 2
⎪16
( x) = ⎪⎨ ( x −1)
1
∴ FX ;2≤ x≤ 4
⎪4
⎪−1
( )
⎪16 20 −12x + x ; 4 ≤ x ≤ 6
2
⎪
⎩⎪ 1 ;x ≥ 6
Problem18. (a). A random variable X has density function
⎧ K
⎪ ,−∞ < x < ∞
f ( x) = ⎨1 + x2 . Determine K and the distribution functions. Evaluate the
⎩⎪ 0 , Otherwise
probability P ( x ≥ 0) .
⎧2x , 0 < x < 1
(b). A random variable X has the P.d.f f ( x ) = ⎨
⎛ 1⎞ ⎛1 1⎞ ⎩ 0 ,⎛Otherwise
3 1⎞
Find (i) P X < (ii) P <x< (iii) P X > / X >
⎜ 2⎟ ⎜4 2⎟ ⎜ 4 2⎟
⎝ ⎠ ⎝ ⎠ ⎝ ⎠
Solution (a):
8
∞
Since
∫ F ( x)dx = 1
−∞
∞
K
∫ 1+ x dx = 1 2
−∞
∞ dx
K∫ =1
1+ x 2
∞
∞
(
K tan−1 x )−∞
=1
⎛ π ⎛ π ⎞⎞
K − − =1
⎜ 2 ⎜ 2 ⎟⎟
⎝ ⎝ ⎠⎠
Kπ = 1
1
K=
π
x x
K
FX ( x ) = ∫ f ( x)dx = ∫ 1+ x dx 2
−∞ −∞
1
( tan −1 x )
x
=
1π ⎡ ⎛ π ⎞⎤
−∞
= tan −1 x − −
π1⎢⎣ ⎡ π ⎜ 2 ⎟⎥
⎝ ⎤ ⎠⎦
= + tan −1 x , −∞ < x < ∞
π ⎢⎣ 2 ∞ ⎥⎦
1 ∞ dx 1 −
P ( X ≥ 0) = ∫ =
π 01+ x2 π
(tan 1 x) 0
1 ⎛π ⎞ 1
= − tan−1 0 = .
π ⎜2 ⎟
2
Solution (b): ⎝ ⎠
⎛ 1/ 2 1/ 2 2 1/ 2 2 ×1 1
(i) P ⎜ x < ⎟⎞ = ∫ f ( x ) dx = ∫ 2xdx = 2 ⎜ ⎛ ⎟x ⎞ =
1 =
2 2 8 4
⎝ ⎠ 0 0 ⎝ ⎠0
⎛1 1 ⎞ 1/ 2 1/ 2 2 1/ 2
(ii) P ⎜ < x < ⎟ = ∫ f ( x ) dx = ∫ 2xdx = 2 ⎜ ⎛ x⎟ ⎞
4 2 2
⎝ ⎠ 1/ 4 1/ 4 ⎝ ⎠1/ 4
⎛1 1
= 2 − ⎞ ⎛1 1 ⎞ 3
= − = .
⎜ ⎟ ⎜ ⎟
⎝ 8 32 ⎠ 3⎝ 4 161 ⎠ 16
⎛ ⎞ ⎛ 3⎞
P⎜X > ∩X > P X>
⎛ 3 1 ⎞ 4 2⎟ ⎜ 4⎟
(iii) P ⎜ X > / X > ⎝ ⎠ ⎝ 1 ⎞⎠
2 ⎠⎟ = = P⎛X >
⎛ 1 ⎞
⎝ 4 P X>
⎜ 2 ⎟ ⎜ 2⎟
⎝ ⎠ ⎝ ⎠
9
1
⎛ 3⎞ 1 ⎛ x2 ⎞ 1 9 7
4 ∫ f ( x ) dx = ∫ 2xdx = 2 ⎜ 2 ⎟
P⎜ X > ⎟ = = 1− =
⎝ ⎠3/ 4 3/ 4 ⎝ 2 ⎠13/ 4 16 16
⎛ 1 ⎞ 1 1 ⎛x ⎞ 1 3
P⎜ X > ⎟ =
2 ∫ () ∫f x dx = 2xdx = 2 ⎜ ⎟
2
= 1 −
4
=
4
⎝ ⎠ 1/ 2 1/ 2 ⎝ ⎠1/ 2
⎛ 3 1⎞ 7 7 4 7
P ⎜ X > / X > ⎟ = 163 = 16× 3= 12 .
4 2
⎝ ⎠
4
⎧Ke −3x , x > 0
Problem 19.(a).If X has the probability density function f ( x) = ⎨
⎩0 , otherwise
find K , P[0.5 ≤ X ≤ 1]and the mean of X .
(b).Find the moment generating function for the distribution whose p.d.f is
f ( x ) = λ e−λ x , x > 0 and hence find its mean and variance.
Solution:
∞
Since
∫ f ( x) dx = 1
−∞
∞
∫ Ke−3xdx = 1
0
∞
⎡ e−3x ⎤
K⎢ ⎥ =1
⎣ − ⎦0
K 3
=1
3
K =3
e−3 − e−1.5 ⎡ −1.5 −3 ⎤
1 1
−3x
P (0.5 ≤ X ≤ 1) = ∫ f ( x ) dx = 3 ∫ e dx = 3 −3 =⎣e –e ⎦
0.5 0.5
∞ ∞
Mean of X = E ( x ) = xf ( x ) dx = 3 xe −3x dx
∫ 0
∫ 0
∞
⎡ ⎛ −e−3x ⎞ ⎛ e−3x ⎞⎤ 3×1 1
= 3⎢ x⎜ −1
⎟ ⎜ ⎟⎥ = =
⎣ ⎝ 3 ⎠ ⎝ 9 ⎠⎦ 0 9 3
1
Hence the mean of X = E ( X ) =
∞
3 ∞
M X (t ) = E etx =( ) ∫ etx f ( x) dx = ∫ λ e − λ x tx
e dx
0 0
∞
= λ ∫ e− x(λ −t )dx
0
10
∞
⎡ e− x(λ −t ) ⎤ λ
=λ⎢ ⎥ =
⎣ −(λ − t) ⎦ 0 λ − t
⎡ d ⎤
Mean = μ ′ = M (t ) ⎡⎢ λ 2⎥⎤ 1
1 ⎢ dt X = (λ − t) =λ
⎥⎣
⎦
t =0 ⎢⎣ ⎥⎦ t =0
⎡ d2 ⎤
μ′= ( ) = ⎡⎢ λ ( 2) 3 ⎤⎥ = 22
2 ⎢ dt 2 X t ⎥
M (λ − t) λ
⎣ ⎦ t =0 ⎢⎣ ⎦⎥ t =0
()
2
2 1 1
Variance = μ2′ − μ ′ = − = .
λ λ λ2
1 2 2
Problem 20. (a). If the continuous random variable X has ray Leigh density
F ( x ) = ⎜⎛ x2 e − 2α ⎟×U ( x ) find E ( x )nand deduce the values of E X and Var( X ) .
2
x ⎞
⎜α
2
⎟
( )
⎝ ⎠
⎧1 − x
⎪ e 2 ,x > 0
(b). Let the random variable X have the p.d.f f ( x) = ⎨2 .
⎪⎩0 , otherwise.
Find the moment generating function, mean & variance of X .
Solution:
⎧1 if x > 0
(a) Here U ( x ) = ⎨
⎩0 if x ≤ 0
∞
( )
E x n = ∫ xn f (x)dx
0
∞ − x2
= xn x2 e2α dx
∫0 α
2
x2
Put =t, x = 0, t = 0
2α 2
x
dx = dt x = α,t = ∞
α2
( )
∞
= ∫ 2α 2 t
n/2
e −t dt ⎡ x= 2α . t ⎤
⎣ ⎦
0
∞
= 2n / 2 α n ∫0t n / 2 e−t dt
E xn ( ) ⎛n ⎞
= 2n/ 2 α n Γ +1 − (1)
⎜2 ⎟
⎝ ⎠
Putting n =
1/ 21 in⎛(1)
E ( x ) = 2 α Γ ⎜ ⎟=
3 ⎞ we get ⎛ 1 ⎞
2α Γ ⎜ +1 ⎟
2 2
⎝ ⎠ ⎝ ⎠
11
1 ⎛1⎞
= 2α Γ ⎜ ⎟
2
2 ⎝ ⎠
α ⎛1⎞
π [ Γ = π
= 2 ⎜2 ⎟
⎝ ⎠
π
∴ E ( x) = α
2
Putting n = 2 in (1), we get
( )
E x 2 = 2α 2 Γ ( 2 ) = 2α 2 [ Γ ( 2 ) = 1]
( )
∴Var ( X ) = E X 2 − ⎡⎣ E ( X ) ⎤⎦
2
π
= 2α 2 − α 2
2
⎛ π ⎞ 2 ⎛ 4 −π ⎞ 2 .
= 2− α = α
⎜ 2 ⎟ ⎜ 2 ⎟
⎝ ∞ ⎠ ⎝ ∞ ⎠
( )
(b) M X (t ) = E etx = ∫ etx f ( x) dx = ∫ etx 12 e− x / 2dx
−∞ 0
∞
∞ −⎛ t 1 ⎞ x
⎡⎢ ⎝ ⎜⎛ t − 21 ⎟⎞⎠x ⎥ ⎤
1 ⎜ ⎟ 1 e 1 1
= ∫e ⎝
2
dx = ⎢
⎠ ⎥ = , if t < .
20 2 ⎢ ⎛ t − 1 ⎞⎥ 1− 2t 2
⎢ ⎜ 2 ⎟⎥
⎣⎝ ⎠⎦ 0
⎡ d ⎤ ⎡ 2 ⎤
E ( X ) = dt M X (t ) = =2
⎣ ⎦ ⎢ 2⎥
(1− 2t ) ⎥⎦ t =0
2 ⎡ d2 t =0 ⎣⎢ 8 ⎤
⎤ ⎡
( )
E X = ⎢ 2 M X (t )⎥ =⎢ 3⎥ =8
⎣ dt ⎦ t =0 ⎢ ⎣ (1− 2t ⎥⎦ t =0
)
( )
Var ( X ) = E X 2 − E ( X ) ⎤⎦ = 8 − 4 = 4 .
2
−∞
∫ f ( x) dx = 1
∞
−b( x−a)
y0 ∫ e dx = 1
0
12
∞
⎡ e−b( x−a) ⎤
y0 ⎢ ⎥ =1
⎣ −b 1 ⎦ 0
⎛ ⎞
y ⎜ ⎟= 1
0 ⎝
b⎠
y0 = b.
∞
−∞
∞
= b ∫ ( x − a ) e − b ( x − a ) dx
r
Put x − a = t , dx = dt , when x = a, t = 0 , x = ∞, t = ∞
∞
= b ∫ t r e − bt dt
0
Γ ( r +1) r!
=b (r +1)
=
b br
In particular r = 1
1
μ′ =
1
b
2
μ 2′ =
b2
1
Mean = a + μ ′ = a +
1
b
()
2
Variance = μ2′ − μ ′ 1
2 1 = 1 .
= −
b2 b2 b2
b) Given μ ′ =1 1, μ ′ =2 4, μ ′ = 10,
3
μ ′ = 45
4
()
2
Variance = μ2 = μ2′ − μ ′ 1
= 4 −1 = 3 .
()
3
μ 3 = μ ′ 3− 3μ ′μ ′2+ 21 μ ′ 1
= 10 − 3 ( 4 )(1) + 2 (1) = 0
3
( ) − 3( μ ′ )
2 4
μ 4 = μ ′ 4− 4μ ′ μ ′3+ 61 μ ′ μ ′ 2 1 1
13
= 45 − 4 (10 )(1) + 6 ( 4 )(1) − 3 (1)
2 4
μ4 = 26 .
Problem 22. (a). A continuous random variable X has the p.d.f
f ( x ) = kx2e−x , x ≥ 0. Find the rth moment of X about the origin. Hence find mean and
variance of X.
(b). Find the moment generating function of the random variable X, with probability
density function f ( x ) = 2 − x for 10≤≤xx<<21.Also find μ ′ , μ ′ .
⎪⎧ x for
⎨ 1 2
⎪0 otherwise
⎩
Solution:
∞
Since
∫ Kx2e−xdx = 1
0
∞
⎡ ⎛ e− x ⎞ ⎛ e− x ⎞ ⎛ e−x ⎞⎤
⎟ − 2x ⎜ ⎟+ 2⎜ ⎟⎥ = 1
2
K⎢x ⎜
⎣ ⎝ ⎠−1 ⎝ 1 ⎠ ⎝ −1 ⎠⎦ 0
2K = 1
1
K= .
2
∞
μr′ = ∫ x f ( x)dx
r
1∞
= x e dx
2 ∫0 r +2 − x
∞
1 −x (r +3)−1 ( r + 2)!
=
2 ∫0
e x dx =
2
3!
Putting n = 1 , μ ′ = = 3
1
2
′ 41
n = 2,μ = = 12
2
2
∴ Mean = μ ′ = 1
3
()
2
Variable= μ 2′ − μ ′ 1
i.e. μ 2 = 12 − ( 3) = 12 − 9
2
∴ μ2 = 3.
∞
(b) M X (t ) =
∫ etx f ( x)dx
−∞
1 2
= ∫ e xdx + ∫ etx ( 2 − x ) dx
tx
0 1
14
1
⎛ xetx etx ⎞ ⎡ etx ⎤
2
etx
= − 2 ⎟ + ⎢ ( 2 − x ) − (−1) 2 ⎥
⎜ t t t t
⎝ ⎠0 ⎣ ⎦1
t t 2t t t
e e 1 e e e
= − 2 + 2 + 2 − − 2
t t 2 t t t t
⎛ et −1 ⎞
=⎜ ⎟
⎝ t ⎠ 2
⎡ t t2 t3 ⎤
= ⎢ 1+ + + + ... −1⎥
⎣ 1! 2! 3! 2
⎦
⎡ t t2 t3 ⎤
= ⎢ 1+ + + + ...⎥
⎣ 2! 3! 4! ⎦
t
′
1μ = coeff . of
=1
1!
t2 7
μ2 ′ = coeff . of 2! = 6
.
x−θ
1 −
Problem 23. (a).The p.d.f of the r.v. X follows the probability law: f ( x) = e θ ,
2θ
−∞ < x < ∞ . Find the m.g.f of X and also find E ( X ) and V ( X ) .
(b).Find the moment generating function and rth moments for the distribution. Whose
p.d.f is f ( x) = Ke−x , 0 ≤ x ≤ ∞ . Find also standard deviation.
Solution:
M (t ) = E (etx ) = ∞ etx f ( x ) dx = ∞ 1
e
– θx−θ etxdx
X ∫
−∞
∫ 2θ
−∞
θ ( x−θ ) ∞ ( − x −θ )
= ∫ 1e θ etxdx + ∫ 1 e θ etxdx
−∞ 2θ ⎛ 1⎞ θ 2∞θ ⎛1 ⎞
−1 θ x t+ −x −t
M X (t ) = e ⎝⎜ θ ⎟⎠ dx + e ⎜θ ⎟
e
2θ
∫ 2θ ∫ θ
e ⎝ ⎠
dx
−∞
⎛ 1⎞
θ t+ −θ
⎛1
−1
⎞
⎜
−1 θ⎟ ⎜θ ⎟
⎝ ⎠ ⎠ ⎝
e e e e
= +
2θ ⎛ t + 1 ⎞ 2θ ⎛ 1 − t ⎞
⎜ θ⎟ ⎜θ ⎟
⎝θ t ⎠ ⎝ ⎠ θt
e eθ t e = θ t ⎡ − θ 2 ⎤ −1
= + =
e ⎣1 ( t )
2 (θ t +1) 2 (1− θ t ) 1− θ t
2 2 ⎦
⎡⎢ 1 θ 2t 2 ⎤
+ θ t + 2! + ...⎥⎡ + θ t + θ t + ...⎤⎦
2 2 4 4
=
⎣ 1
⎣ ⎦
3θ t
2 2
= 1+ θ t + + ...
15
2!
16
E ( X ) = μ ′1= coeff . of t in M X (t ) = θ
t2
μ 2′ = coeff . of in M X (t ) = 3θ
2
2!
( )
2
Var ( X ) = μ2′ − μ ′ 1 = 3θ 2 − θ 2 = 2θ 2 .
b)
Total Probability=1
∞
∴ ∫ ke − x dx = 1
0
∞
⎡ e−x ⎤
k⎢ ⎥ =1
⎣ − ⎦0
1 k =1
∞ ∞
e(t −1)xdx
M
X
( t ) = E ⎡⎣ etx ⎤
⎦
= etxe − x dx
∫ =
∫
0 0
∞
⎡ (t −1)x ⎤ 1
=⎢ e ⎥ = ,t <1
t −1 1− t
⎣ ⎦0
= (1− t ) = 1+ t + t 2 + ... + t r + ...∞
−1
μ′= t2
=
1 coeff . of r!
r!
When r = 1, μ 1′ = 1! = 1
r = 2 , μ ′2= 2! = 2
Variance = μ ′ 2− μ ′ 1= 2 −1 = 1
∴ Standard deviation=1.
Problem 24. (a). Define Binomial distribution Obtain its m.g.f., mean and variance.
(b). (i).Six dice are thrown 729 times. How many times do you expect atleast 3 dice
show 5 or 6 ?
(ii).Six coins are tossed 6400 times. Using the Poisson distribution, what is the
approximate probability of getting six heads x times?
Solution:
a) A random variable X said to follow binomial distribution if it assumes only non
negative values and its probability mass function is given by
P ( X = x ) = nCx p q , x = 0,1, 2,..., n and q = 1− p.
x n −x
M X ( t ) = E ⎡⎣ etx ⎤ ⎦ = ∑ etx P ( X = x )
x=0
n
= ∑ nC xx P xq n − xe t x
x=0
17
n x
= ∑ nCx ( )q
pet n− x
x=0
( )
n
M X (t) = q + pet
Mean of Binomial distribution
Mean = E ( X ) = M X ′ (0)
(
= ⎡ n q + pet ) pet ⎤
n−1
= np Since q + p = 1
⎢⎣ ⎥⎦ t =0
( )
E X 2 = M X′ (0)
(
= ⎡ n ( n −1) q + pet ) ( pet ) ( ) ⎤
n−2 2 n−1
+ npet q + pet
⎢⎣ ⎥⎦ t =0
( )
E X 2 = n (n −1) p2 + np
= n2 p2 + np (1− p ) = n2 p2 + npq
( )
Variance = E X 2 − ⎡⎣ E [ X ]⎤⎦ = npq
2
18
1
Probability of getting six =
6
1 1
∴p= & q = 1−
6 6
Let x : No of throws for getting the number 6.By geometric distribution
P [ X = x] = qx−1 p, x = 1, 2, 3....
Since 6 can be got either in first, second……throws.
To find P [ X > 5] = 1− P [ X ≤ 5]
5 5 x −1 1
⎛ ⎞
= 1− ∑ ⎜ ⎟ .
x =1 ⎝ 6 ⎠ 6
⎡ ⎛ 1 ⎞ ⎛ 5 ⎞⎛ 1 ⎞ ⎛ 5 ⎞2 ⎛ 1 ⎞ ⎛ 5 ⎞3 ⎛ 1 ⎞ ⎛ 5 ⎞4 ⎛ 1 ⎞⎤
= 1− ⎢ ⎜ ⎟ + ⎜ ⎟⎜ ⎟ + ⎜ ⎟ ⎜ ⎟ + ⎜ ⎟ ⎜ ⎟ + ⎜ ⎟ ⎜ ⎟⎥
⎣ ⎢ ⎝ 6 ⎠ ⎝ 65 ⎠⎝ 6 ⎠ ⎝ 6 ⎠ ⎝ 6 ⎠ ⎝ 6 ⎠ ⎝ 6 ⎠ ⎝ 6 ⎠ ⎝ 6 ⎠⎥ ⎦
1 ⎡ ⎛5⎞ ⎤
⎢ 1− ⎜ ⎟ ⎥
6⎢ ⎣ ⎝6 ⎠ ⎥ ⎦ ⎛ 5 ⎞ 5 = 0.4019
= 1− = ⎜ ⎟
5 ⎝6 ⎠
1−
6
b) Here p = 0.8, q = 1 − p = 0.2
P[ X = r] = qr−1 p, r = 0,1, 2...
(i) The probability that the target would be hit on the 6th attempt = P [ X = 6]
= ( 0.2 ) ( 0.8) = 0.00026
5
(ii) The probability that it takes him less than 5 shots = P[ X < 5]
4
4 r −1
r −1
=
∑q
r =1
p = 0.8∑(0.2)
r =1
19
∞
∴P X > t ∞
⎡ −λx ⎤ − λt
= λ −λx
[ ] ∫t e dx = ⎣ −e ⎦ t = e
P [ x > s + t ∩ x > s]
∴ P [ X > s + t / x > s] =
P[ x > s]
P[ X > s + t] e − λ ( s +t ) − λt
= = =
P [ X > s] e− λs
e
= P[x > t ]
b) Let X denote the time to failure of the component then X has exponential distribution
with Mean = 1000 hours.
1 1
∴ = 10, 000 ⇒ λ =
λ 10, 000
⎪ 1 e − 10,000 , x ≥ 0
⎧ x
∫ f ( x) dx
e−1 − e−1.5
= 10,000
∞
= −1
e
∫ f ( x) dx
10,000
0.3679 − 0.2231
= = 0.3936 .
0.3679
(ii) Probability that the component will operate for another 5000 hours given that
it is in operational 15,000 hours = P[ X > 20, 000 / X > 15, 000]
= P [ x > 5000] [By memoryless prop]
∞
∫ f ( x) dx = e = 0.6065
−0.5
=
5000
27. (a). The Daily consumption of milk in a city in excess of 20,000 gallons is
1
approximately distributed as a Gamma variate with parameters α = 2 and λ = .
10, 000
The city has a daily stock of 30,000 gallons. What is the probability that the stock is in
sufficient on a particular day?
20
(b). The lifetime (in hours) of a certain kxpiece of equipment is a continuous r.v. having
⎧xe − , 0 < x < ∞
range 0 < x < ∞ and p.d.f.is f ( x) = ⎨ . Determine the constant K and
⎩0 , otherwise
evaluate the probability that the life time exceeds 2 hours.
Solution:
a) Let X be the r.v denoting the daily consumption of milk (is gallons) in a city
Then Y = X − 20, 000 has Gamma distribution with p.d.f.
y
1 –
f ( y) = y2−1e 10,000
,y≥0
(10, 000 ) Γ ( 2 )
2
y
–
ye 10,000
f (y ) = , y ≥ 0.
(10, 000)2
∴the daily stock of the city is 30,000 gallons, the required probability that the stock is
insufficient on a particular day is given by
P [ X > 30, 000] = P [Y > 10, 000]
y
∞ ∞ –
10,000
= ye
∫ g ( y ) dy = ∫ (10, 000) 2
dy
10,000 10,000
y dy
put Z = then dz =
10, 000 10, 000
∞
To find K , ∫ f ( x) dx = 1
0
∞
∫e
−kx
0
x2−1dx = 1
Γ( 2)
=1⇒ K 2 =1 ∴K =1
2
K
⎧xe − x , 0 < x < ∞
∴ f x( =) ⎨
⎩0 , Otherwise
P[Life time exceeds 2 hours] = P[ X > 2]
∞
= ∫ f ( x ) dx
2
21
∞
= ∫ xe−xdx
2
( ) ( )
∞
= ⎣⎡ x −e− x − e − x ⎦⎤
2
= 2e + e = 3e = 0.4060
−2 −2 − 2
Problem 28. (a). State and prove the additive property of normal distribution.
t2
(μ 2
,σ 2
2
) ,… (μ n
,σ n
2
) then X + X +... + X
1 2 n
also a normal random variable with mean
⎞
⎛n μ , ∑σ .n 2
⎜∑ i ⎟ i
⎝ i=1 i =1 ⎠
Proof:
We know that. M X + X +...+ X 1 2 n
(t ) = M X (t ) M X (t )...M X (t )
1 2 n
t2σ 2
μi t + i
But M X ( t ) = e 2
, i = 1, 2 ... n
i
t 2σ 2
t2 σ 2
t2σ 2
μ1 t + 1 μ2 t + 2
μn t + n
M X +X 1 2 +...+ X n
(t ) = e 2
e 2 ...e 2
(σ 2
+σ 22 +...+ σ 2
)t 2
(μ )t +
1 n
+ μ +...+μ
=e
1 2 n
2
n
n ∑σ i
2 2
t
∑μ t+ i=1
=e i=1
i 2
⎝ ⎠
σ 2π −∞
x−μ
Put z = then dz = σdx, − ∞ < Z < ∞
σ
∞ 2
t(σ z + μ )− z
1
∴ MX (t ) = ∫ e 2
dz
2π −∞ ⎛ 2 ⎞
∞ − z −tσ z
e μt ⎜2 ⎟
= ∫e ⎝ ⎠ dz
2π −∞
22
1 σ ⎛t
2 2 ⎞ σ 2t 2 ∞ 1
μt ∞ − ( z −tσ )
2
+⎜ μt − ( z −tσ )
2
2
= e ∫e dz = e e ∫e
2 ⎝ 2⎟ ⎠ 2
dz
2π −∞ 2π −∞
∞ 1
− ( z −tσ )
2
1
the total area under normal curve is unity, we have ∫e 2
dz = 1
2π −∞
μt + σ 2t2
Hence M X (t ) = e 2
∴For standard normal variable N (0,1)
t2
M X (t ) = e 2
Problem 29. (a). The average percentage of marks of candidates in an examination is 45
will a standard deviation of 10 the minimum for a pass is 50%.If 1000 candidates appear
for the examination, how many can be expected marks. If it is required, that double that
number should pass, what should be the average percentage of marks?
(b). Given that X is normally distribution with mean 10 and
probability P [ X > 12] = 0.1587 . What is the probability that X will fall in the
interval (9,11) .
Solution:
a) Let X be marks of the candidates
Then X N (42,102 )
X − 42
Let z =
10
P [ X > 50] = P [Z > 0.8]
= 0.5 − P [0 < z < 0.8] = 0.5 − 0.2881 = 0.2119
Since 1000 students write the test, nearly 212 students would pass the
examination.
If double that number should pass, then the no of passes should be 424.
We have to find z1 , such that P [Z > z1 ] = 0.424
∴ P [0 < z < z1] = 0.5 − 0.424 = 0.076
From tables, z = 0.19
50 − x1
∴z = ⇒ x = 50 −10z = 50 −1.9 = 48.1
1 1 1
10
The average mark should be 48 nearly.
b) Given X is normally distributed with mean μ = 10.
x−μ
Let z = be the standard normal variate.
σ
12 −10 2
For X = 12, z = ⇒z=
σ σ
2
Put z1 =
σ
Then P [ X > 12] = 0.1587
23
P [Z > Z1 ] = 0.1587
∴0.5 − p [0 < z < z1] = 0.1587
⇒ P [ 0 < z < z1 ] = 0.3413
From area table P[ 0 < z < 1] = 0.3413
2
∴ Z1 = 1 ⇒ = 1
σ
To find P [9 < x < 11]
1 1
For X = 9, z = − and X = 11, z =
2 2
∴ P [9 < X < 11] = P [−0.5 < z < 0.5] = 2P [0 < z < 0.5] = 2 × 0.1915 = 0.3830
Problem. (a). In a normal distribution,31 % of the items are under 45 and 8% are over
64.Find the mean and standard deviation of the distribution.
(b). For a certain distribution the first moment about 10 is 40 and that the 4th moment
about 50 is 48, what are the parameters of the distribution.
Solution:
a) Let μ be the mean and σ be the standard deviation.
Then P [ X ≤ 45] = 0.31 and P [ X ≥ 64] = 0.08
45 − μ
When X = 45 , Z = = −z
σ 1
z1
∴ z1 = 0.495
45 − μ = −0.495σ ---(1)
64 − μ
When X = 64 , Z = =z
σ
2
z2
∴ z2 = 1.405
64 − μ = 1.405σ ---(2)
Solving (1) & (2) We get μ = 10 (approx) & σ = 50 (approx)
b) Let μ be mean and σ 2 the variance then μ ′1 = 40 about A=10
∴Mean A + μ ′ =1 10 + 40 10+40
⇒ μ = 50
Also μ 4= 48 ⇒ 3σ 4 = 48 ⇒ σ 2 = 4
∴ The parameters are Mean = μ = 50 and S.D = σ = 2 .
24